Для связи в whatsapp +905441085890

Дифференциальное уравнение в математике с примерами решения и образцами выполнения

Оглавление:

При решении различных задач математики, физики, химии и других наук часто используют математические модели в виде уравнений, связывающих основные параметры изучаемого процесса. И поскольку скорость изменения величин можно рассматривать как производную некоторой функции, то ряд уравнений содержат и искомую функцию, и производную этой функции.

Так, зависимость массы Понятие дифференциального уравнения вещества, вступившего в химическую реакцию, от времени Понятие дифференциального уравнения описывается уравнением: Понятие дифференциального уравнения или Понятие дифференциального уравнения, где Понятие дифференциального уравнения — коэффициент пропорциональности. «Закон размножения бактерий» (зависимость массы бактерий Понятие дифференциального уравнения от времени Понятие дифференциального уравнения) также описывается уравнением: Понятие дифференциального уравнения или Понятие дифференциального уравнения, где Понятие дифференциального уравнения.

Закон радиоактивного распада вещества ( Понятие дифференциального уравнения ), закон изменения температуры тела в зависимости от времени (Понятие дифференциального уравнения), закон изменения давления воздуха в зависимости от высоты над уровнем моря (Понятие дифференциального уравнения) — все это примеры использования подобных уравнений в практике. Уже приведенные примеры указывают на их исключительную роль при решении разнообразных задач.

Особенностью рассмотренных выше уравнений является то, что неизвестной оказывается нс одно число или пара чисел, а функция. Причем неизвестная функция находится под знаком производной. Уравнения, содержащие производные или дифференциал искомой функции называют дифференциальным и уравнениями.

Например, уравнения Понятие дифференциального уравнения, Понятие дифференциального уравнения, Понятие дифференциального уравнения, Понятие дифференциального уравнения являются дифференциальными уравнениями.

Наивысший порядок производной неизвестной функции, входящей в уравнение, называют порядком дифференциального уравнения.

Так, Понятие дифференциального уравнения, Понятие дифференциального уравнения, Понятие дифференциального уравнения — дифференциальные уравнения первого порядка, Понятие дифференциального уравнения — дифференциальное уравнение второго порядка.

Решением дифференциального уравнения называется функция, подстановка которой в уравнение обращает его в тождество.

Пример:

Докажите, что функция Понятие дифференциального уравнения является решением дифференциального уравнения Понятие дифференциального уравнения.

Решение:

Найдем производную функции Понятие дифференциального уравнения:Понятие дифференциального уравнения. Подставим известное Понятие дифференциального уравнения и найденное Понятие дифференциального уравнения в дифференциальное уравнение Понятие дифференциального уравнения:

Понятие дифференциального уравнения

Получили, что Понятие дифференциального уравнения — верное равенство, следовательно, функция Понятие дифференциального уравнения является решением дифференциального уравнения Понятие дифференциального уравнения.

Любое дифференциальное уравнение имеет не одно, а множество решений, отличающихся друг от друга на константу Понятие дифференциального уравнения. Такое множество решений получило название общего решения дифференциального уравнения. Геометрически его можно изобразить в виде семейства интегральных кривых.

При решении задач часто необходимо из всей совокупности решений дифференциального уравнения выделить одно, отвечающее конкретным требованиям. Для этого задают начальные условия: Понятие дифференциального уравнения при Понятие дифференциального уравнения. Геометрически это означает, что нужно выделить отдельную интегральную кривую, проходящую через точку Понятие дифференциального уравнения.

Задача нахождения решения дифференциального уравнения, удовлетворяющего условию Понятие дифференциального уравнения, называется задачей Коши, а полученное решение называется частным решением дифференциального уравнения.

Эта лекция взята с главной страницы на которой находится курс лекций с теорией и примерами решения по всем разделам высшей математики:

Предмет высшая математика

Другие лекции по высшей математике, возможно вам пригодятся:

Разложение в ряд Фурье периодических функций с периодом 2п.
Разложение в ряд Фурье чётных и нечётных функций, функций произвольного периода.
Простейшие дифференциальные уравнения первого порядка.
Дифференциальные уравнения с разделенными и разделяющимися переменными.

Дифференциальное уравнение

Решение различных задач сводится к отысканию неизвестной функции из уравнения, содержащего независимую переменную, искомую функцию и производные (или дифференциалы) этой функции. Такое уравнение называется дифференциальным.

Рассмотрим задачу, приводящую к дифференциальному уравнению.

Пример:

Составить уравнение кривой, обладающей тем свойством, что отрезок любой касательной, заключенный между осями координат, делится пополам в точке касания.

Решение:

Пусть Дифференциальные уравнения — искомое уравнение, Дифференциальные уравнения — произвольная точка кривой, определяемой этим уравнением; предположим, для определенности, что кривая расположена в первой четверти (рис. 115).

Дифференциальные уравнения

По условию задачи имеем Дифференциальные уравнения, а следовательно, Дифференциальные уравнения Из рис. 115 видно, что

Дифференциальные уравнения

Учитывая, что Дифференциальные уравнения есть угловой коэффициент касательной, который в точке Дифференциальные уравнения равен Дифференциальные уравнения. получаем дифференциальное уравнение искомой кривой:

Дифференциальные уравнения

Из школьного курса известны и другие задачи, приводящие к дифференциальным уравнениям. Наша ближайшая цель — научиться решать некоторые типы дифференциальных уравнений. Поэтому задачи на составление и решение дифференциальных уравнений мы рассмотрим ниже, а теперь переходим к введению
основных понятий и определений.

Определение:

Дифференциальным уравнением называется уравнение, содержащее независимую переменную Дифференциальные уравнения, искомую функцию Дифференциальные уравнения и ее производные Дифференциальные уравнения

Символически дифференциальное уравнение записывается так:

Дифференциальные уравнения

Например, уравнения Дифференциальные уравненияДифференциальные уравнения являются дифференциальными уравнениями.

Определение:

Порядком дифференциального уравнения называется порядок наивысшей производной, входящей в данное уравнение.

Например, Дифференциальные уравнения — уравнение первого порядка, Дифференциальные уравнения — уравнение третьего порядка.

Уравнение (2) является уравнением Дифференциальные уравнения порядка, записанным в общем виде.

Уравнение первого порядка в общем виде записывается следующим образом:

Дифференциальные уравнения

Разрешая уравнение (3), если это возможно, относительно производной Дифференциальные уравнения получим

Дифференциальные уравнения

Уравнение (4) называется уравнением первого порядка, разрешенным относительно производной.

Определение:

Решением дифференциального уравнения называется всякая функция Дифференциальные уравнения, которая обращает данное уравнение в тождество.

Так, например, решением уравнения (1) является
всякая функция вида

Дифференциальные уравнения

где С — постоянная.

В самом деле, заменив в уравнении (1) Дифференциальные уравнения его значением из равенства (5), получим:

Дифференциальные уравнения

При различных числовых значениях постоянной С равенство (5) определяет различные решения
уравнения (1). Например, нетрудно убедиться, что функции Дифференциальные уравнения являются решениями уравнения (1).

Таким образом, для дифференциального уравнения (1) мы рассмотрели как общее решение Дифференциальные уравнения где С — произвольная постоянная, так и частные его решения, например, Дифференциальные уравненияи другие, которые получаются из общего решения при различных числовых значениях постоянной С.

Определение:

Общим решением дифференциального уравнения Дифференциальные уравненияпорядка называется функция.

Дифференциальные уравнения

зависящая от п произвольных постоянных Дифференциальные уравненияДифференциальные уравнения и удовлетворяющая данному уравнению при любых значениях этих постоянных.

Так, общим решением дифференциального уравнения первого порядка называется функция

Дифференциальные уравнения

содержащая только одну постоянную и удовлетворяющая данному уравнению при любом фиксированном значении постоянной С.

Определение:

Частным решением дифференциального уравнения называется решение, полученное из общего решения при фиксированных значениях произвольных постоянных.

Чтобы из бесконечного числа решений дифференциального уравнения, определяемых его общим решением, выделить одно частное решение, требуется ввести начальные условия.

Задать начальные условия дифференциального уравнения Дифференциальные уравненияпорядка означает задать некоторое фиксированное значение аргумента Дифференциальные уравнения и соответствующие значения функции Дифференциальные уравнения и ее производных
Дифференциальные уравненияДифференциальные уравнения

Задача нахождения частного решения Дифференциальные уравнения удовлетворяющего начальным условиям, называется задачей Коши. В случае дифференциального уравнения первого порядка задача Коши ставится так: найти частное решение Дифференциальные уравнения уравнения Дифференциальные уравнения, удовлетворяющее условию Дифференциальные уравнения, или Дифференциальные уравнения .

Пусть, например, требуется найти частное решение уравнения (1), удовлетворяющего условию Дифференциальные уравнения. Общим решением уравнения (1) является функция (5). Подставляя в общее решение начальные условия Дифференциальные уравнения, Дифференциальные уравнения, найдем

Дифференциальные уравнения

Таким образом, искомым частным решением
уравнения (1) является функция

Дифференциальные уравнения

С геометрической точки зрения общее решение дифференциального уравнения определяет семейство кривых, называемых интегральными кривыми, а частное решение определяет лишь
одну-единственную интегральную кривую. Так, общее решение Дифференциальные уравнения уравнения Дифференциальные уравнения определяет семейство равносторонних гипербол, асимптотами которых являются оси координат (рис. 116), а также прямую Дифференциальные уравнения (при Дифференциальные уравнения ). Частное решение Дифференциальные уравненияопределяет лишь одну гиперболу, проходящую через точку (2; 3).

Пример:

Доказать, что функция

Дифференциальные уравнения

является общим решением уравнения

Дифференциальные уравнения

Найти частное решение, удовлетворяющее начальному
условию

Дифференциальные уравнения

Решение:

Заменив в уравнении (10) Дифференциальные уравнения его значением из равенства (9), получим

Дифференциальные уравнения

Следовательно, функция (9) является общим решением
уравнения (10).

Дифференциальные уравнения

Подставляя в (9) начальные условия Дифференциальные уравнения, найдем

Дифференциальные уравнения

Таким образом, искомым частным решением будет функция Дифференциальные уравнения,

Дифференциальные уравнения первого порядка с разделяющимися переменными

Определение:

Дифференциальное уравнение первого порядка Дифференциальные уравнения называется уравнением с разделяющимися переменными, если его можно представить в виде

Дифференциальные уравнения

Предположим, что Дифференциальные уравнения. Тогда уравнение (1) можно переписать так

Дифференциальные уравнения

Уравнение вида (2) называется уравнением с разделенными переменными.

Интегрируя почленно уравнение (2), получим общее решение уравнения (1):

Дифференциальные уравнения

При решении дифференциальных уравнений с разделяющимися переменными полезно придерживаться следующей схемы:

1) разделить переменные;
2) интегрируя уравнение с разделенными переменными, найти общее решение данного уравнения,
3) найти частное решение, удовлетворяющее начальным условиям (если они заданы).

Заметим, что если а является корнем уравнения Дифференциальные уравнения, то, очевидно, функция Дифференциальные уравнения является решением уравнения (1). Поэтому, чтобы получить все решения уравнения (1), надо к полученному общему решению добавить еще решения вида Дифференциальные уравнения, где а — корень уравнения Дифференциальные уравнения

Пример:

Найти частное решение уравнения Дифференциальные уравнения, если Дифференциальные уравненияпри Дифференциальные уравнения.

Решение:

Перепишем данное уравнение в виде

Дифференциальные уравнения

Интегрируя обе части последнего равенства, найдем:

Дифференциальные уравнения

Подставив начальные условия Дифференциальные уравнения, найдем C:

Дифференциальные уравнения

Следовательно, искомое частное решение будет

Дифференциальные уравнения

Пример:

Найти общее решение уравнения

Дифференциальные уравнения

Решение:

Разделим переменные. Для этого преобразуем данное уравнение следующим образом:

Дифференциальные уравнения

(полагаем здесь Дифференциальные уравнения ).

Проинтегрируем обе части последнего равенства:

Дифференциальные уравнения

Для удобства потенцирования представим Дифференциальные уравнения в виде Дифференциальные уравнения и постоянную интегрирования Дифференциальные уравнения в виде Дифференциальные уравнения. Имеем:

Дифференциальные уравнения

Потенцируя, получим

Дифференциальные уравнения

В процессе решения мы предположили Дифференциальные уравнения. Однако Дифференциальные уравнения
— решение данного уравнения (в этом легко убедиться проверкой). Следовательно, сняв в (3) ограничение Дифференциальные уравнения, мы получим, что

Дифференциальные уравнения

— общее решение данного уравнения (решение Дифференциальные уравнения получается отсюда как частное решение именно при Дифференциальные уравнения).

Пример:

Тело, имеющее в начальный момент времени Дифференциальные уравнениятемпературу Дифференциальные уравнения, охлаждается в воздушной среде до температуры Дифференциальные уравнения в течение времени Дифференциальные уравнения мин. Найти время, за которое тело охладится до температуры 30°, если известно, что температура воздуха 20°, а скорость охлаждения тела пропорциональна разности между температурой тела и температурой воздуха.

Решение:

Обозначим температуру тела в любой момент времени t через Дифференциальные уравнения. Так как скорость охлаждения тела Дифференциальные уравнения пропорциональна разности между температурой тела Т и температурой воздуха 20°, то получаем дифференциальное уравнение

Дифференциальные уравнения

Здесь К — коэффициент пропорциональности. Уравнение (4) является дифференциальным уравнением первого порядка с разделяющимися переменными, поэтому решаем его по указанной выше схеме.

Разделив переменные, получим

Дифференциальные уравнения

Интегрируя, находим:

Дифференциальные уравнения

или

Дифференциальные уравнения

Равенство (5) является общим решением уравнения (4). Найдем частное решение, удовлетворяющее условию Дифференциальные уравнения:

Дифференциальные уравнения

Итак, частным решением является функция

Дифференциальные уравнения

Найдем числовое значение постоянной Дифференциальные уравнения. Для этого воспользуемся условием, что Дифференциальные уравнения:

Дифференциальные уравнения

Таким образом, частное решение (6) можно записать так:

Дифференциальные уравнения

В задаче требуется определить время, за которое тело охладится до температуры 30°. Положив в равенстве Дифференциальные уравнения найдем:

Дифференциальные уравнения

Итак, тело охладится до температуры 30° в течение одного часа.

Однородные дифференциальные уравнения первого порядка

Определение:

Функция Дифференциальные уравнения называется однородной функцией Дифференциальные уравнения измерения Дифференциальные уравнения , если при любом t (кроме, быть может, Дифференциальные уравнения ) имеет место тождество

Дифференциальные уравнения

Например, Дифференциальные уравнения — однородная функция третьего измерения, так как

Дифференциальные уравнения

Аналогично доказывается, что функции

Дифференциальные уравнения

являются однородными функциями соответственно первого, нулевого и второго измерений.

Определение:

Дифференциальное уравнение первого порядка Дифференциальные уравнения называется однородным, если его можно представить в виде

Дифференциальные уравнения

где Дифференциальные уравнения — однородные функции одинакового измерения.

Однородное дифференциальное уравнение приводится к дифференциальному уравнению с разделяющимися переменными подстановкой

Дифференциальные уравнения

где Дифференциальные уравнения — новая неизвестная функция.

Пример:

Найти общее решение уравнения

Дифференциальные уравнения

Решение:

В данном уравнении функции Дифференциальные уравнения — однородные второго измерения, следовательно, данное уравнение является однородным.

Положим Дифференциальные уравнения, откуда Дифференциальные уравнения. Подставляем эти выражения Дифференциальные уравнения в данное уравнение:

Дифференциальные уравнения

Разделяем переменные:

Дифференциальные уравнения

Интегрируем почленно это уравнение:

Дифференциальные уравнения

Возвращаясь к прежней функции Дифференциальные уравнения находим искомое общее решение:

Дифференциальные уравнения

Пример:

Найти частное решение уравнения

Дифференциальные уравнения

если Дифференциальные уравнения при Дифференциальные уравнения

Решение:

Записав данное уравнение в виде Дифференциальные уравнениялегко можно убедиться в том, что оно однородно.

Положим Дифференциальные уравнения, откуда Дифференциальные уравнения Подставляем значения у и dy в последнее уравнение:

Дифференциальные уравнения

Интегрируя, получаем

Дифференциальные уравнения

откуда

Дифференциальные уравнения

Подставив в найденное общее решение начальные условия, найдем

Дифференциальные уравнения

Итак, искомое частное решение будет

Дифференциальные уравнения

Пример:

Составить уравнение кривой, проходящей через точку Дифференциальные уравнения, если известно, что угловой коэффициент касательной в каждой ее точке равен Дифференциальные уравнения

Решение:

На основании геометрического смысла производной

Дифференциальные уравнения

В полученном уравнении Дифференциальные уравненияследовательно, оно однородное. Положим Дифференциальные уравнения откуда Дифференциальные уравнения Тогда уравнение принимает вид

Дифференциальные уравнения

Разделяем переменные:

Дифференциальные уравнения

Интегрируя это уравнение, найдем:

Дифференциальные уравнения

Возвращаясь к прежней функции у, будем иметь:

Дифференциальные уравнения

Подставляя координаты точки А в найденное общее решение, получим

Дифференциальные уравнения

Итак, искомое уравнение кривой имеет вид

Дифференциальные уравнения

Линейные дифференциальные уравнения первого порядка

Определение:

Дифференциальное уравнение первого порядка Дифференциальные уравнения называется линейным, если его можно представить в виде

Дифференциальные уравнения

где Дифференциальные уравнения и Дифференциальные уравнения — заданные функции от Дифференциальные уравнения (в частности, постоянные величины).

Линейное уравнение приводится к уравнению с разделяющимися переменными с помощью некоторой подстановки Дифференциальные уравнения, где одна из функций и, Дифференциальные уравнения подбирается определенным образом, а другая — новая неизвестная функция. Поясним сказанное на конкретном примере.

Пример:

Найти общее решение уравнения

Дифференциальные уравнения

Решение:

Разделив все члены данного уравнения на Дифференциальные уравнения, приведем его к виду (1):

Дифференциальные уравнения

Здесь Дифференциальные уравнения Положим Дифференциальные уравнения,

откуда Дифференциальные уравнения

Подставим эти значения Дифференциальные уравнения в уравнение (2):

Дифференциальные уравнения

Сгруппируем члены, содержащие, например, Дифференциальные уравнения и вынесем Дифференциальные уравнения за скобку:

Дифференциальные уравнения

Выберем функцию и так, чтобы выражение в скобках обращалось в нуль, т. е. чтобы

Дифференциальные уравнения

Тогда уравнение (3) примет вид

Дифференциальные уравнения

Итак, исходное уравнение мы можем привести к виду (5) заменой Дифференциальные уравнения, где и — любое решение уравнения (4).

Решаем уравнение (4) как уравнение с разделяющимися переменными:

Дифференциальные уравнения

Интегрируя, получаем:

Дифференциальные уравнения

откуда

Дифференциальные уравнения

Подставив значение функции и в уравнение (5), найдем:

Дифференциальные уравнения

откуда

Дифференциальные уравнения

Заменив в подстановке Дифференциальные уравнения функции и и Дифференциальные уравнения их выражениями из равенств (6) и (7), получим искомое общее решение данного уравнения:

Дифференциальные уравнения

или

Дифференциальные уравнения

Пример:

Найти частное решение уравнения Дифференциальные уравнения если Дифференциальные уравнения при Дифференциальные уравнения

Решение:

Запишем данное уравнение в виде Дифференциальные уравнения

Положим Дифференциальные уравнения, откуда Дифференциальные уравнения. Подставляем значения Дифференциальные уравнения и Дифференциальные уравнения в последнее уравнение:

Дифференциальные уравнения

Сгруппируем члены, содержащие Дифференциальные уравнения и вынесем Дифференциальные уравнения за скобки:

Дифференциальные уравнения

Найдем функцию и такую, что

Дифференциальные уравнения

Тогда уравнение (8) примет вид

Дифференциальные уравнения

Решаем уравнение (9) как уравнение с разделяющимися переменными

Дифференциальные уравнения

откуда, после интегрирования, т. е.

Дифференциальные уравнения

Подставив значение функции и в уравнение (10), найдем:

Дифференциальные уравнения

Отсюда

Дифференциальные уравнения

Итак, общим решением данного уравнения является функция

Дифференциальные уравнения

Находим частное решение, удовлетворяющее начальному условию Дифференциальные уравнения при Дифференциальные уравнения:

Дифференциальные уравнения

Следовательно, искомым частным решением является функция

Дифференциальные уравнения

Дифференциальные уравнения второго порядка

Дифференциальное уравнение второго порядка в общем случае записывается в виде

Дифференциальные уравнения

или, если это возможно, в разрешенном относительно Дифференциальные уравнения виде

Дифференциальные уравнения

Определение 1. Общим решением уравнения (2) называется функция

Определение:

Общим решением уравнения (2) называется функция Дифференциальные уравнения, содержащая две произвольные постоянные Дифференциальные уравнения и Дифференциальные уравнения и удовлетворяющая условиям:

1) при любых значениях постоянных Дифференциальные уравнения и Дифференциальные уравнения функция Дифференциальные уравнения является решением уравнения (2)

2) каковы бы ни были начальные условия

Дифференциальные уравнения

существуют единственные значения Дифференциальные уравнения и Дифференциальные уравнения такие, что функция Дифференциальные уравнения является решением
уравнения (2) и удовлетворяет начальным условиям (3).

Определение:

Частным решением уравнения (2) называется всякое решение Дифференциальные уравнения, получающееся из общего решения Дифференциальные уравнения при фиксированных значениях Дифференциальные уравнения

Простейшее уравнение второго порядка имеет вид

Дифференциальные уравнения

Уравнения этого вида решаются двукратным интегрированием. Полагаем Дифференциальные уравнения тогда Дифференциальные уравнения уравнение (4) примет вид Дифференциальные уравнения или Дифференциальные уравнения Отсюда

Дифференциальные уравнения

где Дифференциальные уравнения — одна из первообразных для функции Дифференциальные уравнения Так как Дифференциальные уравнения то

Дифференциальные уравнения

Отсюда, интегрируя еще раз, находим общее решение уравнения (4)

Дифференциальные уравнения

Пример:

Найти общее решение уравнения Дифференциальные уравнения.

Решение:

Положим Дифференциальные уравнения; тогда Дифференциальные уравнения а следовательно,

Дифференциальные уравнения

Интегрируя это уравнение, находим:

Дифференциальные уравнения

Интегрируя второй раз, находим искомое общее решение:

Дифференциальные уравнения

Пример:

Водитель трамвая, выключая постепенно реостат, увеличивает мощность вагонного двигателя так, что сила тяги возрастает на 1200 H за каждую секунду. Найти уравнение движения трамвая, если вначале сила тяги была равной нулю. Сила тяжести вагона Дифференциальные уравнения, сопротивление трения постоянно и равно 2000 H; начальная скорость равна нулю; начало движения вагона не совпадает с моментом выключения реостата.

Решение:

Будем считать, что центр тяжести вагона перемещается по горизонтальной прямой. Начало координат поместим в начальном положении центра тяжести вагона.

Проектируя внешние силы, приложенные к вагону, на ось абсцисс, получим два слагаемых: силу тяги, равную 1200 t (t — время, прошедшее с момента включения реостата), и силу сопротивления, равную 2000 H.

Согласно второму закону динамики дифференциальное уравнение движения вагона имеет вид

Дифференциальные уравнения

Это дифференциальное уравнение вида (4). Так как начало движения не совпадает с моментом выключения реостата, то время Дифференциальные уравнения, соответствующее началу движения, можно определить из условия равенства силы тяги и силы сопротивления:

Дифференциальные уравнения

Для удобства вычислений положим

Дифференциальные уравнения

откуда

Дифференциальные уравнения

Тогда уравнение (5) можно записать в виде

Дифференциальные уравнения

Интегрируя дифференциальное уравнение (7), получаем

Дифференциальные уравнения

Подставляя начальные условия Дифференциальные уравнения при Дифференциальные уравнения, находим Дифференциальные уравнения; следовательно,

Дифференциальные уравнения

Интегрируя уравнение (8), получим

Дифференциальные уравнения

Так как Дифференциальные уравнения при Дифференциальные уравнения Следовательно, уравнение движения вагона примет вид

Дифференциальные уравнения

или Дифференциальные уравнения

Линейные однородные дифференциальные уравнения второго порядка с постоянными коэффициентами. Определение и некоторые свойства

Определение 1. Линейным однородным дифференциальным уравнением второго порядка с постоянными коэффициентами называется уравнение вида

Дифференциальные уравнения

где p и q — постоянные величины.

Определение:

Два частных решения Дифференциальные уравнения и Дифференциальные уравнения уравнения (1) образуют фундаментальную систему решений, если для любого x

Дифференциальные уравнения

Определитель Дифференциальные уравнения называется определителем Вронского или вронскианом.

Пример:

Известно, что функции Дифференциальные уравнения, Дифференциальные уравненияявляются частными решениями уравнения

Дифференциальные уравнения

Докажем, что решения Дифференциальные уравнения и Дифференциальные уравнения образуют фундаментальную систему решений, а Дифференциальные уравнения и Дифференциальные уравнения — не образуют.

В самом деле, найдя вронскианы указанных пар решений, получим:

Дифференциальные уравнения

Теорема:

О структуре общего решения. Если два частных решения Дифференциальные уравнения линейного однородного дифференциального уравнения второго порядка с постоянными коэффициентами образуют фундаментальную систему, то общее решение этого уравнения имеет вид

Дифференциальные уравнения

где Дифференциальные уравненияпроизвольные постоянные. Выражение Дифференциальные уравненияназывается линейной комбинацией функций Дифференциальные уравнения.

Доказательство:

Прежде всего докажем, что функция (3) является решением уравнения (1). Для этого подставим в уравнение (1) вместо у линейную комбинацию (3) и докажем, что оно превращается в тождество:

Дифференциальные уравнения

или

Дифференциальные уравнения

Так как Дифференциальные уравнения и Дифференциальные уравнения являются решениями уравнения (1),

то

Дифференциальные уравнения

Следовательно, равенство (4) при любых значениях Дифференциальные уравнения и Дифференциальные уравнения является тождеством: Дифференциальные уравнения. А это значит, что функция (3) является решением уравнения (1).

Теперь докажем, что она является его общим решением. Для этого достаточно показать, что из него можно выделить единственное частное решение, удовлетворяющее заданным начальным условиям:

Дифференциальные уравнения

Пусть Дифференциальные уравнения — какое-либо частное решение уравнения (1), удовлетворяющее начальным условиям (5). Покажем, что оно может быть получено из общего решения (3) надлежащим выбором постоянных Дифференциальные уравнения и Дифференциальные уравнения

В самом деле, подставляя начальные условия в равенства

Дифференциальные уравнения

получим

Дифференциальные уравнения

Равенства (6) представляют собой систему уравнений с неизвестными Дифференциальные уравнения и Дифференциальные уравнения, причем определитель этой системы

Дифференциальные уравнения

является определителем Вронского для функций Дифференциальные уравнения и Дифференциальные уравнения при Дифференциальные уравнения. Так как, по условию, частные решения Дифференциальные уравнения и Дифференциальные уравненияобразуют фундаментальную систему, то Дифференциальные уравнения при любом действительном значении Дифференциальные уравнения. Поэтому система (6) имеет единственное решение:

Дифференциальные уравнения

Таким образом, существует частное решение

Дифференциальные уравнения

удовлетворяющее начальным условиям (5), причем в
силу его единственности оно совпадает с решением Дифференциальные уравнения.

Теорема доказана.

Решение линейных однородных дифференциальных уравнений второго порядка с постоянными коэффициентами

Было установлено, что для нахождения общего решения уравнения (1) достаточно найти два его частных решения, образующих фундаментальную систему.

Будем искать частные решения уравнения (1) в виде

Дифференциальные уравнения

тогда

Дифференциальные уравнения

Подставляя выражения для Дифференциальные уравненияв уравнение (1), получим

Дифференциальные уравнения

Так как

Дифференциальные уравнения

Уравнение (8) называется характеристическим уравнением линейного однородного дифференциального уравнения второго порядка с постоянными коэффициентами.

Решив характеристическое уравнение, найдем его корни Дифференциальные уравнения и Дифференциальные уравнения а следовательно, и частные решения уравнения (1):

Дифференциальные уравнения

При решении характеристического уравнения возможны следующие три случая:

Случай — 1. Корни характеристического уравнения действительные и различные: Дифференциальные уравнения.

В этом случае имеем два частных решения уравнения (1): Дифференциальные уравнения. Эти решения образуют фундаментальную систему решений, так как вронскиан

Дифференциальные уравнения

Следовательно, общее решение уравнения (1) имеет вид

Дифференциальные уравнения

Пример:

Найти частное решение уравнения

Дифференциальные уравнения

удовлетворяющее начальным условиям Дифференциальные уравнения

Решение:

Составляем характеристическое уравнение Дифференциальные уравнения откуда Дифференциальные уравнения

Подставляя найденные значения Дифференциальные уравнения в формулу (10), получим общее решение

Дифференциальные уравнения

Дифференцируя общее решение, получим

Дифференциальные уравнения

Согласно заданным начальным условиям имеем

Дифференциальные уравнения

или

Дифференциальные уравнения

откуда Дифференциальные уравнения Таким образом, искомым частным решением является функция

Дифференциальные уравнения

Случай 2. Корни характеристического уравнения
действительные и равные
: Дифференциальные уравнения

В этом случае непосредственно находим лишь одно
частное решение:

Дифференциальные уравнения

Предоставляем читателю возможность самостоятельно доказать, что вторым частным решением Дифференциальные уравнения, образующим вместе с первым фундаментальную систему, является функция

Дифференциальные уравнения

Таким образом, в этом случае общее решение уравнения (1) имеет вид

Дифференциальные уравнения

Пример:

Найти общее решение уравнения

Дифференциальные уравнения

Решение:

:Характеристическое уравнение

Дифференциальные уравнения

имеет действительные и равные корни Дифференциальные уравнения

Поэтому согласно формуле (11) искомое общее решение имеет вид

Дифференциальные уравнения

Случай 3. Корни характеристического уравнения
комплексные
: Дифференциальные уравнения

В этом случае частные решения имеют вид:

Дифференциальные уравнения

Но согласно формуле Эйлера (см. § 8 гл. 9)

Дифференциальные уравнения

Поэтому

Дифференциальные уравнения

Чтобы заменить комплексные решения действительными, рассмотрим функции:

Дифференциальные уравнения

Так как Дифференциальные уравнения и Дифференциальные уравнения являются линейными комбинациями частных решений Дифференциальные уравнения и Дифференциальные уравнения т. е. имеют вид (3), то они сами являются решениями уравнения (1), причем можно показать, что они образуют фундаментальную систему решений (определитель Вронского для этих решений отличен от нуля). Таким образом, общее
решение уравнения (1) имеет вид

Дифференциальные уравнения

или

Дифференциальные уравнения

Пример:

Найти общее решение уравнения

Дифференциальные уравнения

Решение:

Корни характеристического уравнения

Дифференциальные уравнения

комплексные: Дифференциальные уравнения

Здесь Дифференциальные уравнения, поэтому общим решением является функция

Дифференциальные уравнения

Пример:

Найти частное решение уравнения

Дифференциальные уравнения

удовлетворяющее начальным условиям Дифференциальные уравнения и Дифференциальные уравнения

Решение. Составляем характеристическое уравнение Дифференциальные уравнения и находим его корни: Дифференциальные уравнения

Следовательно, общим решением является функция

Дифференциальные уравнения

Дифференцируя общее решение, найдем

Дифференциальные уравнения

Постоянные Дифференциальные уравнения и Дифференциальные уравнения находим из начальных условий:

Дифференциальные уравнения

Отсюда Дифференциальные уравнения

Итак, искомым частным решением является функция

Дифференциальные уравнения

Решение задач на составление дифференциальных уравнений

Составление дифференциальных уравнений

Мы уже отмечали, что многие задачи физики, техники, биологии и социальных наук решаются при помощи дифференциальных уравнений. При этом сначала составляется дифференциальное уравнение, которое затем решается, во многих частных случаях, по одному из указанных выше способов, в зависимости от его типа. Составление дифференциальных уравнений по условию задачи напоминает составление алгебраических уравнений. При решении задач на составление дифференциальных уравнений широко используемся геометрический и физический смысл производной, а также известные законы естественных и социальных наук.

Рассмотрим конкретный пример.

Пример:

Конденсатор, емкость которого Q, включается в цепь с напряжением Е и сопротивлением R. Определить заряд q(t) конденсатора в момент времени t, если в начальный момент времени он был равен нулю.

Решение:

Если в момент времени t заряд конденсатора равен q(t), то к этому моменту времени ток i равен Дифференциальные уравнения, а электродвижущая сила Е равна разности между напряжением цепи U и напряжением конденсатора q/Q т. е.

Дифференциальные уравнения

Согласно закону Ома Дифференциальные уравнения откуда

Дифференциальные уравнения

или

Дифференциальные уравнения

Уравнение (1) является линейным дифференциальным уравнением первого порядка. Для его решения введем подстановку Дифференциальные уравнения, откуда

Дифференциальные уравнения

Подставляя значения q и Дифференциальные уравнения в уравнение (1), группируя члены, содержащие Дифференциальные уравнения и вынося его за скобки, получим

Дифференциальные уравнения

Найдем функцию и, удовлетворяющую условию

Дифференциальные уравнения

тогда уравнение (2) примет вид

Дифференциальные уравнения

Из уравнения (3) находим

Дифференциальные уравнения

откуда, интегрируя, получаем

Дифференциальные уравнения

или

Дифференциальные уравнения

Подставим полученную функцию и в уравнение (4):

Дифференциальные уравнения

откуда

Дифференциальные уравнения

или

Дифференциальные уравнения

Таким образом,

Дифференциальные уравнения

или

Дифференциальные уравнения

Постоянную С найдем из условия Дифференциальные уравнения при Дифференциальные уравнения:

Дифференциальные уравнения

Итак, в любой момент времени t заряд конденсатора определяется по формуле

Дифференциальные уравнения

Дифференциальное уравнение показательного роста

Ряд задач на составление дифференциальных
уравнений приводит к уравнениям вида

Дифференциальные уравнения

где Дифференциальные уравнения — постоянная величина.

Уравнение (6) называется уравнением показательного роста Его смысл состоит в том, что скорость изменения функции пропорциональна самой функции.

Перепишем уравнение (6) в виде

Дифференциальные уравнения

Разделяя переменные и интегрируя, находим последовательно:

Дифференциальные уравнения

или

Дифференциальные уравнения

Пример:

Катер движется в спокойной воде со скоростью Дифференциальные уравнения Определить скорость катера через 2 мин после выключения двигателя, если за 40 с она уменьшилась до Дифференциальные уравнения Сопротивление воды пропорционально скорости движения катера. Решение. Пусть скорость движения катера в момент времени t равна Дифференциальные уравнения. Тогда на движущийся катер действует сила сопротивления воды Дифференциальные уравнения Но согласно закону Ньютона Дифференциальные уравнения а следовательно, Дифференциальные уравнения, или

Дифференциальные уравнения

Уравнение (8) является дифференциальным уравнением показательного роста, поэтому его общим решением будет

Дифференциальные уравнения

Постоянную С найдем изначального условия Дифференциальные уравнения

Итак, скорость движения катера после выключения двигателя определяется формулой

Дифференциальные уравнения

Найдем значение постоянной Дифференциальные уравнения. Для этого воспользуемся условием, что при Дифференциальные уравнения

Дифференциальные уравнения

Положив в равенстве Дифференциальные уравненияДифференциальные уравнениянайдем искомую скорость:

Дифференциальные уравнения

Пример:

Скорость распада радия в момент времени t пропорциональна его количеству m(t). Пусть в начальный момент времени масса радия Дифференциальные уравненияДифференциальные уравнения. Сколько радия останется через 300 лет, если известно, что период T полураспада радия (промежуток времени, через который первоначальная масса радия уменьшается в два раза) равен 1550 лет.

Решение:

Из условия задачи имеем

Дифференциальные уравнения

где Дифференциальные уравнения. Знак минус показывает, что масса радия убывает, а следовательно, скорость распада Дифференциальные уравнения отрицательна.

Очевидно, что общим решением уравнения (11) будет функция

Дифференциальные уравнения

Согласно условию Дифференциальные уравнения; имеем: Дифференциальные уравнения

Следовательно,

Дифференциальные уравнения

Коэффициент Дифференциальные уравнения найдем из условия, что

Дифференциальные уравнения

Таким образом

Дифференциальные уравнения

Положив Дифференциальные уравнения найдем количество радия, оставшегося не распавшимся через 300 лет:

Дифференциальные уравнения

Дифференциальное уравнение гармонических колебаний

Пусть шарик массы m прикреплен двумя пружинами так, как показано на рис. 117. а. В положении равновесия координата
центра шарика равна нулю. Сместим шарик в направлении оси Дифференциальные уравнения

Дифференциальные уравнения

Тогда согласно закону Гука на шарик действует сила, пропорциональная смещению x:

Дифференциальные уравнения

где Дифференциальные уравнения. Знак минус указывает, что восстанавливающая сила направлена в сторону, противоположную направлению смещения. По второму закону Ньютона имеем

Дифференциальные уравнения

Из равенств (13) и (14) следует

Дифференциальные уравнения

Так как ускорение а прямолинейного движения есть вторая производная координаты (пути) по времени Дифференциальные уравнения, то равенство (15) можно переписать в виде

Дифференциальные уравнения

Положив Дифференциальные уравнения , получим

Дифференциальные уравнения

Уравнение (16) является линейным однородным дифференциальным уравнением второго порядка с постоянными коэффициентами.

Если физическая величина изменяется во времени в соответствии с уравнением (16), то говорят, что она совершает гармонические колебания. Поэтому уравнение (16) называется дифференциальным уравнением гармонических колебаний.

Решим уравнение (16). Для этого составим
характеристическое уравнение и найдем его корни:

Дифференциальные уравнения

Следовательно, общим решением уравнения (16) будет функция

Дифференциальные уравнения

Вместо произвольных постоянных Дифференциальные уравнения и Дифференциальные уравнения введем новые произвольные постоянные А и Дифференциальные уравнения, которые связаны с постоянными Дифференциальные уравнения и Дифференциальные уравнения соотношениями

Дифференциальные уравнения

Тогда равенство (17) примет вид

Дифференциальные уравнения

или

Дифференциальные уравнения

Равенство (19) является уравнением гармонических колебаний. Величина А представляет собой наибольшее отклонение тела от положения равновесия и называется амплитудой колебания.

Из равенств (18) следует

Дифференциальные уравнения

Так как sin и — периодическая функция с наименьшим периодом Дифференциальные уравнения, то наименьший период гармонического колебания найдем из равенства

Дифференциальные уравнения

откуда

Дифференциальные уравнения

Величина Дифференциальные уравнения называется частотой колебания. Наконец, величину Дифференциальные уравнения называют начальной фазой гармонического колебания и находят по формуле Дифференциальные уравнения, которая следует из равенств (18).

Дифференциальные уравнения в высшей математике

Дифференциальным уравнением называется уравнение, содержащее производные от искомой функции или ее дифференциалы. ‘Гак, например, Дифференциальные уравнения или Дифференциальные уравнениядифференциальные уравнения.

Решить дифференциальное уравнение, значит найти такую функцию от х, которая удовлетворяет данному уравнению, т. е. обращает это уравнение в тождество при подстановке ее в уравнение вместо у.

Уравнение, содержащее производные или дифференциалы не выше первого порядка, называется дифференциальным уравнением первого порядка.

Дифференциальные уравнения имеют большое применение в геометрии, механике, физике и других дисциплинах, а также в технике.

Пример решения дифференциального уравнения мы имели , отыскивая уравнение кривой по заданному угловому коэффициенту касательной. В результате решения дифференциального уравнения

Дифференциальные уравнения

мы получили уравнение

Дифференциальные уравнения

которое носит название общего решения дифференциального уравнения. Подставив вместо х и у соответственно 1 и 3, мы нашли уравнение

Дифференциальные уравнения

называемое частным решением дифференциального уравнения *)

*) Общее решение дифференциального уравнения всегда содержит произвольное постоянное С, в частном же решении это постоянное заменено определенным числом.

Данные значения х = 1 и у = 3 называются начальными условиями. Начальные условия задаются для того, чтобы из общего решения дифференциального уравнения получить его частное решение.

Правильность решения дифференциального уравнения легко проверить, подставив выражение (2) в уравнение (1). В результате получим:

Дифференциальные уравнения

или

Дифференциальные уравнения

т. е.

Дифференциальные уравнения

что говорит о правильности решения уравнения (1).

Дифференциальные уравнения первого порядка с разделяющимися переменными

Если каждая часть дифференциального уравнения представляет собою произведение некоторого выражения, зависящего от одной переменной, на дифференциал этой переменной, то говорят, что переменные в уравнении разделены; например, хdх = уdу. В этом случае уравнение можно интегрировать почленно.

Уравнения, в которых переменные разделяются, называются дифференциальными уравнениями с разделяющимися переменными.

Для того чтобы решить дифференциальное уравнение с разделяющимися переменными, нужно произвести разделение переменных, а затем взять интеграл от обеих частей уравнения.

Рассмотрим несколько примеров.

Пример:

Решить уравнение хdх = уdу если при х = 5 у= 10.

Решение:

Для разделения переменных обе части уравнения поделим на произведение ху, получим:

Дифференциальные уравнения

Интегрируя обе части последнего уравнения, найдем:

Дифференциальные уравнения

или

Дифференциальные уравнения

В правой части прибавлено постоянное в виде ln С для облегчения потенцирования. Освобождаясь от символа логарифма, т. е. потенцируя, получим общее решение:

Дифференциальные уравнения

Для определения постоянного С подставим в полученное решение начальные условия х = 5 и y =10:

Дифференциальные уравнения

откуда

Дифференциальные уравнения

Следовательно, искомое частное решение будет:

Дифференциальные уравнения

Таким образом, из всех прямых (семейства прямых), проходящих через начало координат, мы выделили одну, на которой лежит точка с координатами (5; 10).

Пример:

Решить уравнение Дифференциальные уравнения если при

х = 0 у = 4.

Решение:

После разделения переменных получим:

Дифференциальные уравнения

отсюда

Дифференциальные уравнения

или

Дифференциальные уравнения

Находим значение С из условия x = 0 и у = 4; сделав подстановку, получим:

Дифференциальные уравнения

откуда

Дифференциальные уравнения

Итак,

Дифференциальные уравнения

Для потенцирования нужно и правую часть последнего равенства написать со знаком логарифма. Пользуясь определением логарифма, будем иметь:

Дифференциальные уравнения

следовательно, решение (1) можно переписать:

Дифференциальные уравнения

отсюда, потенцируя, получаем: Дифференциальные уравнения или

Дифференциальные уравнения

Дифференциальные уравнения вида du dx

Уравнение, содержащее производные или дифференциалы второго порядка, называется дифференциальным уравнением второго порядка. Простейшее из этих уравнений имеет вид Дифференциальные уравнения оно решается двукратным интегрированием.

Пример:

Решить уравнение Дифференциальные уравнения

Решение:

Согласно определению второй производной можно написать

Дифференциальные уравнения

Положим теперь

Дифференциальные уравнения

тогда

Дифференциальные уравнения

и данное уравнение перепишется следующим образом:

Дифференциальные уравнения

или

Дифференциальные уравнения

Интегрируя последнее уравнение, найдем:

Дифференциальные уравнения

и

Дифференциальные уравнения

Но

Дифференциальные уравнения

поэтому

Дифференциальные уравнения

или

Дифференциальные уравнения

Снова интегрируем и получаем:

Дифференциальные уравнения

Мы нашли общее решение данного уравнения.

Чтобы получить частное решение его, необходимо найти числовые значения постоянных С1 и С2; для этого нужно иметь начальные условия. Пусть кривая, определяемая частным

решением, проходит, например, через точки с координатами (0; 1) иДифференциальные уравнения ; тогда, подставив значения х и у в уравнение (1), получим следующую систему уравнений относительно С1 и С2:

Дифференциальные уравнения

откуда

Дифференциальные уравнения

Следовательно, частное решение данного дифференциального уравнения при указанных начальных условиях будет:

Дифференциальные уравнения

Для проверки правильности решения найдем вторую производную этой функции:

Дифференциальные уравнения

Мы пришли к исходному уравнению, что говорит о том, что оно решено правильно.

В разобранном примере начальные условия были даны в виде координат двух точек кривой, уравнение которой является частным решением дифференциального уравнения. Однако начальные условия могут быть даны и в иной форме, например, при

Дифференциальные уравнения

Это значит, что мы задаем точку кривой и направление касательной в этой точке.

Пример:

Ускорение прямолинейного движения тела равно Дифференциальные уравнения Выразить путь s тела как функцию времени t.

Решение:

Согласно механическому смыслу второй производной функции имеем:

Дифференциальные уравнения

Обозначив

Дифференциальные уравнения

напишем

Дифференциальные уравнения

откуда

Дифференциальные уравнения

и

Дифференциальные уравнения

Заменив р его выражением, получим:

Дифференциальные уравнения

или

Дифференциальные уравнения

отсюда

Дифференциальные уравнения

Для получения частного решения нужны начальные условия. Пусть при t = 0 s = 0 и Дифференциальные уравнения (предполагаем, что в начальный момент движения путь s и скорость Дифференциальные уравнения равны нулю). Заменив t, s и Дифференциальные уравненияуравнениях (2) и (3) нулями, получим:

Дифференциальные уравнения

Таким образом, искомая зависимость будет

Дифференциальные уравнения

Обыкновенные дифференциальные уравнения

В математике дифференциальные уравнения занимают особое место. Математическое исследование самых разнообразных явлений, происходящих в природе, часто приводит к решению таких уравнений, поскольку сами законы, которым подчиняется то или иное явление, записываются в виде дифференциальных уравнений.

Дифференциальные уравнения — это уравнения, в которые неизвестная функция входит под знаком производной. Основная задача теории дифференциальных уравнений — изучение функций, являющихся решениями таких уравнений.

Дифференциальные уравнения можно разделить на обыкновенные дифференциальные уравнения, в которых неизвестные функции являются функциями одной переменной, и на дифференциальные уравнения в частных производных, в которых неизвестные функции являются функциями двух и большего числа переменных.

Теория дифференциальных уравнений в частных производных более сложная и рассматривается в более полных или специальных курсах математики. Элементы теории обыкновенных дифференциальных уравнений изложены в данной главе. В дальнейшем, говоря о дифференциальных уравнениях, будем иметь в виду только обыкновенные дифференциальные уравнения.

Определение дифференциального уравнения первого порядка

Изучение теории дифференциальных уравнений начнем с наиболее простого уравнения — уравнения первого порядка.

Определение:

Уравнение вида Обыкновенные дифференциальные уравнения
где х — независимая переменная; у — искомая функция; у’ — ее производная, называется дифференциальным уравнением первого порядка.

Если уравнение (1) можно разрешить относительно у’, то оно принимает вид
Обыкновенные дифференциальные уравнения
и называется уравнением первого порядка, разрешенным относительно производной. Будем рассматривать именно такие уравнения.

В некоторых случаях уравнение (2) удобно записать в виде Обыкновенные дифференциальные уравнения или в видеОбыкновенные дифференциальные уравнения являющемся частным случаем более общего уравненияОбыкновенные дифференциальные уравнения
где Р (х, у) и Q (х, у) — известные функции. Уравнение в симметричной форме (3) удобно тем, что переменные х и у в нем равноправны, т. е. каждую из них можно рассматривать как функцию другой.

Приведем примеры дифференциальных уравнений вида (2) и (3): Обыкновенные дифференциальные уравнения

Решение уравнения. Задача Коши

Определение:

Решением дифференциального уравнения первого прядка называется функция Обыкновенные дифференциальные уравнения, которая при подстановке в уравнение обращает его в тождество.

Так, например, функция Обыкновенные дифференциальные уравнения является решением уравнения Обыкновенные дифференциальные уравнения т. е. при подстановке в уравнение обращает его в тождество: Обыкновенные дифференциальные уравнения

График решения дифференциального уравнения называется интегральной кривой.

Ответ на вопрос о том, при каких условиях уравнение (2) имеет решение, дает теорема Коши, которая называется теоремой о существовании и единственности решения дифференциального уравнения (2) и является основной в теории дифференциальных уравнений.

Теорема:

Теорема Коши. Если функция f (х, у) и ее частная производная Обыкновенные дифференциальные уравнения определены и непрерывны в некоторой области G плоскости Оху, то какова бы ни была внутренняя точка Обыкновенные дифференциальные уравнения области G, в некоторой окрестности этой точки существует единственное решение уравнения y’=f(x, у), удовлетворяющее условиям:
Обыкновенные дифференциальные уравнения

Теорема Коши дает возможность по виду дифференциального уравнения (2) решать вопрос о существовании и единственности его решения. Это особенно важно в тех случаях, когда заранее неизвестно, имеет ли данное уравнение решение.

Геометрически теорема утверждает, что через каждую внутреннюю точку Обыкновенные дифференциальные уравнения области G проходит единственная интегральная кривая. Очевидно, что в области G уравнение (2) имеет бесконечное число различных решений.

Условия (4), в силу которых функция Обыкновенные дифференциальные уравнения принимает заданное значение Обыкновенные дифференциальные уравнения в заданной точке Обыкновенные дифференциальные уравнения, называют начальными условиями решения и записывают обычно так: Обыкновенные дифференциальные уравнения

Отыскание решения уравнения (2), удовлетворяющего начальным условиям (5), — одна из важнейших задач теории дифференциальных уравнений. Эта задача называется задачей Коши. С геометрической точки зрения решить задачу Коши — значит из множества интегральных кривых выделить ту, которая проходит через заданную точку Обыкновенные дифференциальные уравнения плоскости Оху.

Точки плоскости, через которые либо проходит более одной интегральной кривой, либо не проходит ни одной интегральной кривой, называются особыми точками данного уравнения.

Общее и частное решение уравнения

Дадим два основных определения.

Определение:

Общим решением уравнения (2) в некоторой области G плоскости Оху называется функция Обыкновенные дифференциальные уравнения зависящая от х и произвольной постоянной С, если она является решением уравнения (2) при любом значении постоянной С, и если при любых начальных условиях (5) таких, что Обыкновенные дифференциальные уравнения существует единственное значение постоянной Обыкновенные дифференциальные уравнения такое, что функция Обыкновенные дифференциальные уравнения удовлетворяет данным начальным условиям Обыкновенные дифференциальные уравнения

Определение:

Частным решением уравнения (2) в области G называется функция Обыкновенные дифференциальные уравнения, которая получается из общего решения Обыкновенные дифференциальные уравнения при определенном значении постоянной Обыкновенные дифференциальные уравнения

Геометрически общее решение Обыкновенные дифференциальные уравнения представляет собой семейство интегральных кривых на плоскости Оху, зависящее от одной произвольной постоянной С, а частное решение Обыкновенные дифференциальные уравнения — одну интегральную кривую этого семейства, проходящую через заданную точку Обыкновенные дифференциальные уравнения.

Иногда начальные условия (5) называют условиями Коши, а частным решением называют решение какой-нибудь задачи Коши.

Пример:

Рассмотрим уравнение Обыкновенные дифференциальные уравнения
Данное уравнение является дифференциальным уравнением первого порядка. Оно удовлетворяет всем условиям теоремы Коши, так как функции Обыкновенные дифференциальные уравнения определены и непрерывны на всей плоскости Оху. Легко проверить, что функция Обыкновенные дифференциальные уравнения где С — произвольная постоянная, является общим решением данного уравнения во всей плоскости Оху.

Геометрически это общее решение представляет собой семейство кубических парабол. При различных значениях постоянной С получаем различные решения данного уравнения. Например, если Обыкновенные дифференциальные уравнения

Для решения какой-нибудь задачи Коши, т. е. отыскания частного решения, зададим произвольные начальные условия: Обыкновенные дифференциальные уравнения Подставляя эти значения в общее решение Обыкновенные дифференциальные уравнения вместо х и у, получаем Обыкновенные дифференциальные уравнения откуда Обыкновенные дифференциальные уравнения Таким образом, найдено частное решение Обыкновенные дифференциальные уравнения Геометрически это означает, что из семейства кубических парабол Обыкновенные дифференциальные уравнения выбрана одна, проходящая через заданную точку Обыкновенные дифференциальные уравнения (рис. 221).

Пример:

Рассмотрим уравнение Обыкновенные дифференциальные уравнения
Данное уравнение является дифференциальным уравнением первого порядка. Функции Обыкновенные дифференциальные уравнения непрерывны при Обыкновенные дифференциальные уравнения Следовательно, во всей плоскости Оху, кроме оси Оу, это уравнение удовлетворяет условиям теоремы Коши.

Нетрудно проверить, что общим решением данного уравнения в областях у>0 и у<0 является функция у=С/х, где С — произвольная постоянная. При различных значениях постоянной С получаем различные решения.

Найдем частное решение, удовлетворяющее, например, начальным условиям Обыкновенные дифференциальные уравнения. Имеем 1=С/1. Отсюда С=1 и искомое частное решение у=1/х.

Геометрически общее решение данного уравнения представляет собой семейство гипербол у=С/х, каждая из которых изображает частное решение данного уравнения. Задавая начальные условия Обыкновенные дифференциальные уравнения, выделяем из всего семейства ту гиперболу, которая проходит через точку (1; 1) плоскости Оху (рис. 222).

Заметим, что через точки, лежащие на оси Оу, не проходит ни одна интегральная кривая, т. е. это особые точки данного уравнения.

Геометрический смысл уравнения

Пусть дано дифференциальное уравнение первого порядка y’=f(x, у) и пусть функция Обыкновенные дифференциальные уравнения — его решение. График решения представляет собой непрерывную интегральную кривую, через каждую точку которой можно провести касательную. Из уравнения следует, что угловой коэффициент у’ касательной к интегральной кривой в каждой ее точке (х; у) равен значению в этой точке правой части уравнения f (x, у). Таким образом, уравнение y’=f(x, у) устанавливает зависимость между координатами точки (х; у) и угловым коэффициентом у’ касательной к графику интегральной кривой в той же точке. Зная х и у, можно указать направление касательной к этой интегральной кривой в точке (х, у).Обыкновенные дифференциальные уравнения

Сопоставим каждой точке (х; у) интегральной кривой направленный отрезок, угловой коэффициент которого равен f (х, у). Получим так называемое поле направлений данного уравнения, раскрывающее геометрический смысл дифференциального уравнения первого порядка.

Итак, с геометрической точки зрения уравнение y’=f(x, у) определяет на плоскости Оху поле направлений, а решение этого уравнения — интегральная кривая, направление касательной к которой в каждой точке совпадает с направлением поля в этой точке.

Построив на плоскости поле направлений данного дифференциального уравнения, можно приближенно построить интегральные кривые.

Пример:

Рассмотрим уравнение у’=у/х.

Функция f (x, у)=у/х не определена при х=0, следовательно, поле направлений для данного уравнения можно построить на всей плоскости, кроме оси Оу.

В каждой точке Обыкновенные дифференциальные уравнения угловой коэффициент у’ касательной к интегральной кривой равен у/х и совпадает с угловым коэффициентом прямой, проходящей через начало координат и эту точку. На рис. 223 изображено поле направлений данного уравнения. Очевидно, что интегральными кривыми являются прямые у=Сх
(С—произвольная постоянная).
Обыкновенные дифференциальные уравнения

Рассмотрим теперь методы нахождения решений дифференциальных уравнений первого порядка. Отметим, что общего метода нахождения решений не существует. Обычно Ч рассматривают отдельные типы уравнений, и для каждого из них находят свой способ нахождения решения.

Уравнения с разделяющимися переменными

Определение:

Уравнение вида Обыкновенные дифференциальные уравнения
где Обыкновенные дифференциальные уравнения — непрерывные функции, называется дифференциальным уравнением с разделяющимися переменными.

Для отыскания решения уравнения (6) нужно, как говорят, разделить в нем переменные. Для этого заменим в (6) у’ на Обыкновенные дифференциальные уравнения, разделим обе части уравнения на Обыкновенные дифференциальные уравнения (предполагаем Обыкновенные дифференциальные уравнения) и умножим на dх. Тогда уравнение (6) принимает вид Обыкновенные дифференциальные уравнения

В этом уравнении переменная х входит только в правую часть, а переменная у — только в левую (т. е. переменные разделены).

Предполагая, что функция Обыкновенные дифференциальные уравнения является решением уравнения, и подставляя ее в (7), получаем тождество. Интегрируя тождество, получаем Обыкновенные дифференциальные уравнения
где Обыкновенные дифференциальные уравнения — произвольная постоянная.

Соотношение (8) определяет неявным образом общее решение уравнения (6).

Пример:

Решить уравнение Обыкновенные дифференциальные уравнения (сравните с примером 3). Решение. Данное уравнение вида (6), где Обыкновенные дифференциальные уравнения и Обыкновенные дифференциальные уравнения Разделяя переменные, получаем: Обыкновенные дифференциальные уравнения. Интегрируя, имеемОбыкновенные дифференциальные уравнения

Потенцируя, находим: Обыкновенные дифференциальные уравнения что эквивалентно уравнению Обыкновенные дифференциальные уравнения Полагая Обыкновенные дифференциальные уравнения окончательно получаем Обыкновенные дифференциальные уравнения
— общее решение данного уравнения, где С — произвольная постоянная, которая может принимать как положительные, так и отрицательные значения, но Обыкновенные дифференциальные уравнения. Заметим, что у=0 также решение уравнения (оно было потеряно при делении на у). Это решение можно включить в (9), если считать, что постоянная С принимает и значение С=0. Геометрически общее решение (9) представляет собой семейство прямых, проходящих через начало координат.

Пусть требуется выделить из общего решения (9) частное решение, удовлетворяющее следующим начальным условиям: Обыкновенные дифференциальные уравнения Подставляя эти значения в общее решение (9) вместо х и у, получаем Обыкновенные дифференциальные уравнения, откуда С=2. Таким образом, искомое частное решение у=2х.

Линейные уравнения

Определение:

Уравнение вида Обыкновенные дифференциальные уравнения
где р(х) и f (х) — непрерывные функции, называется линейным дифференциальным уравнением первого порядка.

Название уравнения объясняется тем, что неизвестная функция у и ее производная у’ входят в уравнение линейно, т. е. в первой степени.

Если Обыкновенные дифференциальные уравнения то уравнение (10) называется линейным однородным уравнением. Если Обыкновенные дифференциальные уравнения то уравнение (10) называется линейным неоднородным уравнением.

Для нахождения общего решения уравнения (10) может быть применен метод вариации постоянной.

В этом методе сначала находят общее решение линейного однородного уравнения
Обыкновенные дифференциальные уравнения
соответствующего данному неоднородному уравнению (10). Уравнение (11) является уравнением с разделяющимися переменными. Разделяя переменные и интегрируя, имеемОбыкновенные дифференциальные уравнения
Отсюда, потенцируя, находим общее решение уравнения (11): Обыкновенные дифференциальные уравнения
где Обыкновенные дифференциальные уравнения — произвольная постоянная.

Теперь найдем общее решение уравнения (10) в виде (12), где С будем считать не постоянной, а новой неизвестной функцией от х (в этом смысл метода!), т. е. в виде
Обыкновенные дифференциальные уравнения

Чтобы найти функцию С(х) и, тем самым, решение в виде (13), подставим функцию (13) в уравнение (10). Получим Обыкновенные дифференциальные уравнения

Итак, чтобы функция (13) являлась решением уравнения (10), функция С (х) должна удовлетворять уравнению (14). Интегрируя его, находим
Обыкновенные дифференциальные уравнения

где Обыкновенные дифференциальные уравнения — произвольная постоянная. Подставляя найденное выражение для С(х) в соотношение (13), получаем общее решение линейного уравнения (10):Обыкновенные дифференциальные уравнения

При решении конкретных примеров проще повторять каждый раз все приведенные выше выкладки, чем использовать громоздкую формулу (15).

Пример:

Найти общее решение уравнения Обыкновенные дифференциальные уравнения
Решение. Данное уравнение является линейным. Здесь Обыкновенные дифференциальные уравнения Решаем сначала соответствующее однородное уравнение у’+3у=0. Разделяя переменные Обыкновенные дифференциальные уравнения и интегрируя, находим Обыкновенные дифференциальные уравнения

Ищем общее решение данного уравнения в виде Обыкновенные дифференциальные уравнения Дифференцируя, имеем Обыкновенные дифференциальные уравнения Подставляя в данное уравнение выражения для у и у’, получаем Обыкновенные дифференциальные уравнения
откуда Обыкновенные дифференциальные уравнения где Обыкновенные дифференциальные уравнения — произвольная постоянная. Следовательно, общее решение данного уравнения имеет вид Обыкновенные дифференциальные уравнения

Уравнения в полных дифференциалах

Определение:

Уравнение вида
Обыкновенные дифференциальные уравнения
где левая часть представляет собой полный дифференциал некоторой функции F (х, у) в некоторой области G, называется уравнением в полных дифференциалах.

Если уравнение (16) является уравнением в полных дифференциалах, то его можно записать следующим образом:
Обыкновенные дифференциальные уравнения
где F (х, у) — такая функция, что Обыкновенные дифференциальные уравнения Отсюда следует, что общее решение уравнения (16) в неявном виде определяется уравнениемОбыкновенные дифференциальные уравнения

где С — произвольная постоянная. Действительно, если Обыкновенные дифференциальные уравнения — решение уравнения (16), то Обыкновенные дифференциальные уравнения и наоборот, для любой функции Обыкновенные дифференциальные уравнения,обращающей в тождество уравнение F(x, у)=С, получаем Обыкновенные дифференциальные уравнения — решение уравнения (16).

Таким образом, нахождение решения уравнения (16) сводится к отысканию такой функции F(x, у), дифференциал которой равен Обыкновенные дифференциальные уравнения

Как известно (см. теорему 13.7), для того, чтобы выражение Обыкновенные дифференциальные уравнения было полным дифференциалом некоторой функции F(х, у), необходимо и достаточно, чтобы Обыкновенные дифференциальные уравнения

Допустим, что условие (17) выполнено. Тогда существует функция
F(x, у) такая, что Обыкновенные дифференциальные уравнения Отсюда
Обыкновенные дифференциальные уравнения

Интегрируя соотношение Обыкновенные дифференциальные уравнения по х, находим Обыкновенные дифференциальные уравнения
где С (y) — произвольная функция от у. Теперь подберем функцию С (y) так, чтобы выполнялось второе из соотношений (18). Для этого продифференцируем правую часть равенства (19) по у и производную приравняем Q (х, у):
Обыкновенные дифференциальные уравнения

Из полученного уравнения (20) определяем С'(y) и, интегрируя, находим С(у). Подставляя найденную функцию С(у) в соотношение (19), получаем искомую функцию F(х, у).

Чтобы выделить из общего решения частное, удовлетворяющее начальным условиям Обыкновенные дифференциальные уравнения надо в общем решении F(x; y)=C х и у заменить начальными значениями. Тогда Обыкновенные дифференциальные уравнения — искомое частное решение.

Пример:

Найти общее решение уравнения Обыкновенные дифференциальные уравнения и выделить из него частное решение, удовлетворяющее начальным условиям Обыкновенные дифференциальные уравнения

Решение:

Здесь Обыкновенные дифференциальные уравнения Так как
Обыкновенные дифференциальные уравнения
то выражение Обыкновенные дифференциальные уравнения является полным дифференциалом некоторой функции F(x,y). Имеем Обыкновенные дифференциальные уравнения

Найдем функцию С (у), используя формулу (20): Обыкновенные дифференциальные уравненияПодставляя найденное С(у) в (21), получаем Обыкновенные дифференциальные уравнения

Данное уравнение принимает вид Обыкновенные дифференциальные уравнения а его общее решение определяется уравнением Обыкновенные дифференциальные уравнения

Полагая Обыкновенные дифференциальные уравнения — произвольная постоянная), получаем окончательное уравнение, определяющее неявно общее решение исходного дифференциального уравнения Обыкновенные дифференциальные уравнения

Найдем теперь значение постоянной Обыкновенные дифференциальные уравнения, при котором частное решение удовлетворяет заданным начальным условиям. Имеем: Обыкновенные дифференциальные уравнения откуда Обыкновенные дифференциальные уравнения, и искомое частное решение определяется уравнением Обыкновенные дифференциальные уравнения

Приближенное решение дифференциальных уравнений первого порядка методом Эйлера

Мы рассмотрели несколько способов нахождения точных решений дифференциальных уравнений первого порядка. Если ни один из них не приводит к цели или требует сложных вычислений, то прибегают к приближенным методам решений уравнений. Познакомимся с простейшим из них — методом Эйлера.

Суть этого метода состоит в том, что искомая интегральная кривая, являющаяся графиком частного решения, приближенно заменяется ломаной.

Пусть даны дифференциальное уравнение

Обыкновенные дифференциальные уравнения

и начальные условия Обыкновенные дифференциальные уравнения

Найдем приближенно решение
уравнения на отрезке Обыкновенные дифференциальные уравнения, удовлетворяющее заданным начальным условиям.

Разобьем отрезок Обыкновенные дифференциальные уравнения точками Обыкновенные дифференциальные уравнения на n равных частей.

Пусть Обыкновенные дифференциальные уравненияОбыкновенные дифференциальные уравнения Обозначим через Обыкновенные дифференциальные уравнения приближенные значения искомого решения в точках Обыкновенные дифференциальные уравнения. Проведем через точки разбиения х, прямые (рис. 224), параллельные оси Оу, и последовательно проделаем следующие однотипные операции.

Обыкновенные дифференциальные уравнения

Подставим значения Обыкновенные дифференциальные уравнения в правую часть уравнения Обыкновенные дифференциальные уравнения и вычислим угловой коэффициент касательной к интегральной кривой в точке Обыкновенные дифференциальные уравнения. Для нахождения приближенного значения у, искомого решения заменяем на отрезке Обыкновенные дифференциальные уравнения интегральную кривую отрезком ее касательной в точке Обыкновенные дифференциальные уравнения. При этом получаем

Обыкновенные дифференциальные уравнения

откуда, так как Обыкновенные дифференциальные уравнения известны, находим

Обыкновенные дифференциальные уравнения

Подставляя значения х, и у, в правую часть уравнения Обыкновенные дифференциальные уравнения, вычисляем угловой коэффициент Обыкновенные дифференциальные уравнения касательной к интегральной кривой в точке Обыкновенные дифференциальные уравнения. Далее, заменяя на отрезке Обыкновенные дифференциальные уравнения интегральную кривую отрезком касательной, находим приближенное значение решения Обыкновенные дифференциальные уравнения в точке Обыкновенные дифференциальные уравнения:

Обыкновенные дифференциальные уравнения

В этом равенстве известными являются Обыкновенные дифференциальные уравнения выражается через них.

Аналогично находим

Обыкновенные дифференциальные уравнения

Таким образом, приближенно построена искомая интегральная кривая в виде ломаной и получены приближенные значения Обыкновенные дифференциальные уравненияискомого решения в точках Обыкновенные дифференциальные уравнения. При этом значения Обыкновенные дифференциальные уравнения вычисляются по формуле

Обыкновенные дифференциальные уравнения

Формула (22) и является основной расчетной формулой метода Эйлера. Ее точность тем выше, чем меньше разность Обыкновенные дифференциальные уравнения

Степень точности метода Эйлера, вообще говоря, невелика. Существуют гораздо более точные методы приближенного решения дифференциальных уравнений.

Пример:

Найти приближенное решение уравнения Обыкновенные дифференциальные уравнения на отрезке [0, 1], удовлетворяющее начальным условиям Обыкновенные дифференциальные уравнения, Обыкновенные дифференциальные уравнения, и вычислить у при x=1.

Решение. Разделим отрезок [0, 1] на 10 равных частей точками Обыкновенные дифференциальные уравнения. Обозначим через Обыкновенные дифференциальные уравнения приближенные значения решения, которые будем искать по формуле (22). Имеем:

Обыкновенные дифференциальные уравнения

Аналогично находятся остальные значения Обыкновенные дифференциальные уравнения, причем результаты вычисления удобно расположить в таблице, заполняя последовательно одну строку за другой.

Обыкновенные дифференциальные уравнения

Второй столбец таблицы содержит приближенные значения искомого решения данного уравнения на [0, 1], удовлетворяющего заданным начальным условиям.

Приближенное значение функции у при Обыкновенные дифференциальные уравнения.

Чтобы сравнить приближенный результат с точным, найдем точное решение данного уравнения при тех же начальных условиях. Так как уравнение линейное, то используем метод вариации постоянной. Находим общее решение однородного уравнения

Обыкновенные дифференциальные уравнения

Варьируя постоянную Обыкновенные дифференциальные уравнения и подставляя в данное уравнение, получаем: Обыкновенные дифференциальные уравненияОбыкновенные дифференциальные уравнения

Обыкновенные дифференциальные уравнения

общее решение данного уравнения. Подставляя вместо х и у начальные значения Обыкновенные дифференциальные уравнения. находим Обыкновенные дифференциальные уравнения= 2. Итак, точное решение данного уравнения, удовлетворяющее заданным начальным условиям, Обыкновенные дифференциальные уравнения. Точное решение при х=1 таково: Обыкновенные дифференциальные уравнения. Сравнивая с приближенным значением, видим, что абсолютная погрешность составляет 0,2293.

Некоторые применения дифференциальных уравнений первого порядка

К дифференциальным уравнениям первого порядка приводят различные физические задачи. Основную трудность при их решении представляет составление дифференциальных уравнений. Здесь не существует универсального метода. Каждая задача требует индивидуального подхода, основанного на понимании законов физики, и умения переводить физические задачи на математический язык.

Рассмотрим несколько таких задач.

Задача о прожекторе. Определить форму зеркала, представляющего собой поверхность вращения и обладающего тем свойством, что все лучи, выходящие из источника света, помещенного в точке О на оси вращения, отражаются зеркалом параллельно этой оси.

Для решения задачи рассмотрим плоское сечение зеркала, проходящее через ось вращения. Поместим источник света в начале координат, и пусть ось вращения совпадает с осью Ох (рис. 225). Обозначим через а угол, образованный осью Ох и касательной 4S в произвольной точке сечения М (х; у). Наша цель — найти форму сечения, т. е. зависимость координаты у от координаты х: у=у(х). Ломаная ОМТ изображает путь луча, исходящего из источника света в точке О и отражающегося в точке Мот поверхности зеркала параллельно оси Ох. Проведем нормаль MN и опустим из точки М на ось Ох перпендикуляр MP. Так как Обыкновенные дифференциальные уравнения (угол падения равен углу отражения), то Обыкновенные дифференциальные уравнения. Следовательно, Обыкновенные дифференциальные уравнения равнобедренный и OM = ON. Кроме того, по построению Обыкновенные дифференциальные уравнения. Используя равенства ON=PN-РО, Обыкновенные дифференциальные уравнения, РО= —х, ON= ОМ =Обыкновенные дифференциальные уравнения, приходим к соотношению

Обыкновенные дифференциальные уравнения

Отсюда, учитывая геометрический смысл производной Обыкновенные дифференциальные уравнения, для определения зависимости у от х получаем дифференциальное уравнение первого порядка

Обыкновенные дифференциальные уравнения

Преобразуем это уравнение следующим образом. Умножим обе его части на 2 dx:

Обыкновенные дифференциальные уравнения

Используя подстановку Обыкновенные дифференциальные уравнения, получаем уравнение с разделяющимися переменными

Обыкновенные дифференциальные уравнения

которое можно преобразовать к виду

Обыкновенные дифференциальные уравнения

Заменяя переменную z ее выражением через х и у, получаем

Обыкновенные дифференциальные уравнения

Возводя в квадрат обе части этого уравнения, получаем

Обыкновенные дифференциальные уравнения

Таким образом, искомая кривая — парабола с параметром р=С и вершиной, лежащей на отрицательной полуоси Ох на расстоянии С/2 от начала координат (рис. 226). Следовательно, искомая отражательная поверхность — параболоид вращения.

Задача о радиоактивном распаде. Экспериментальным путем установлено, что скорость распада радиоактивного вещества, т. е. скорость изменения его массы в зависимости от времени, прямо пропорциональна его количеству. Установим закон изменения массы т радиоактивного вещества в зависимости от времени t, считая, что начальная масса вещества при t=0 была Обыкновенные дифференциальные уравнения.

Обыкновенные дифференциальные уравнения

Пусть в момент времени t масса вещества есть m, в момент времени Обыкновенные дифференциальные уравнения масса составляет Обыкновенные дифференциальные уравнения. За время Обыкновенные дифференциальные уравнения распадается масса Обыкновенные дифференциальные уравнения. Отношение Обыкновенные дифференциальные уравнения -средняя скорость распада за время Обыкновенные дифференциальные уравнения, Обыкновенные дифференциальные уравнения-мгновенная скорость распада в момент времени t.

Согласно условию

Обыкновенные дифференциальные уравнения

где к — коэффициент пропорциональности (знак минус взят потому, что масса вещества убывает с течением времени, а производная убывающей функции отрицательна). Получено дифференциальное уравнение первого порядка, из которого надо найти зависимость массы m от времени t.

Решая уравнение, находим

Обыкновенные дифференциальные уравнения

откуда

Обыкновенные дифференциальные уравнения

Формула (24) дает зависимость массы вещества как функции времени. В данной задаче постоянная С имеет определенное значение, а именно при t=0 получаем: Обыкновенные дифференциальные уравнения. Подставляя это значение С в формулу (24), получаем искомую зависимость массы радиоактивного вещества от времени:

Обыкновенные дифференциальные уравнения

Равенство (24) представляет собой общее решение дифференциального уравнения, а равенство (25) — частное решение, отвечающее начальным условиям данной задачи.

Коэффициент k определяется экспериментально. Например, для радия Обыкновенные дифференциальные уравнения 0,000447. Промежуток времени Т, за который распадается половина первоначальной массы радиоактивного вещества, называют периодом полураспада этого вещества. Подставляя в формулу (25) вместо m значение Обыкновенные дифференциальные уравнения, вместо k — значение 0,000447, получаем уравнение для определения периода полураспада Т радия:

Обыкновенные дифференциальные уравнения

откуда

Обыкновенные дифференциальные уравнения

Задача о законе «естественного роста». Закон «естественного роста» — это закон, согласно которому скорость «роста» вещества прямо пропорциональна его количеству. Найдем формулу для определения изменения количества вещества у в зависимости от времени t, считая, что в начальный момент t=0 количество вещества было равно Обыкновенные дифференциальные уравнения.

Здесь независимой переменной является время t, а искомой величиной — количество вещества в любой момент времени. Скорость «роста» вещества есть скорость изменения величины у в зависимости от переменной t.

Используя, как и в предыдущей задаче, физический смысл производной, можно записать закон «естественного роста» следующим образом:

Обыкновенные дифференциальные уравнения

где k>0 — коэффициент пропорци9нальности. Уравнение (26), отличающееся только знаком правой части от уравнения (23), описывает многие процессы «размножения».

Решение уравнения (26), удовлетворяющее заданным начальным условиям Обыкновенные дифференциальные уравнения, имеет вид

Обыкновенные дифференциальные уравнения

Формула (27) и выражает закон «естественного роста». Согласно этому закону, например, происходит «размножение» нейтронов в ядерных реакциях, размножение бактерий, рост кристаллов и т. п.

Дифференциальные уравнения второго порядка

Определение. Уравнение вида

Обыкновенные дифференциальные уравнения

где х — независимая переменная; у — искомая функция; у’ и у» — ее производные, называется дифференциальным уравнением второго порядка.

Обычно изучают уравнения, которые могут быть записаны в виде, разрешенном относительно второй производной:

Обыкновенные дифференциальные уравнения

Так же как и для дифференциального уравнения первого порядка, решением уравнения (1) называется функция Обыкновенные дифференциальные уравненияОбыкновенные дифференциальные уравнения, которая при подстановке в уравнение обращает его в тождество. График решения называется также интегральной кривой.

Для уравнения второго порядка имеет место теорема существования и единственности решения (теорема Коши), аналогичная соответствующей теореме для уравнения первого порядка.

Теорема:

Теорема Коши. Если функция Обыкновенные дифференциальные уравнения и ее частные производные Обыкновенные дифференциальные уравнения определены и непрерывны в некоторой области G пространства переменных Обыкновенные дифференциальные уравнения, то какова бы ни была внутренняя точка Обыкновенные дифференциальные уравнения области G, в некоторой окрестности точки Обыкновенные дифференциальные уравнения существует единственное решение уравнения Обыкновенные дифференциальные уравнения, удовлетворяющее условиям

Обыкновенные дифференциальные уравнения

Геометрически это означает, что через заданную точку Обыкновенные дифференциальные уравнения плоскости проходит единственная интегральная кривая с заданным угловым коэффициентом у0 касательной в этой точке.

Условия (2) называют начальными условиями решения и часто записывают в виде

Обыкновенные дифференциальные уравнения

Как и для уравнения первого порядка, задачу отыскания решения по заданным начальным условиям называют задачей Коши.

Дадим теперь определения общего и частного решений уравнения (1), удовлетворяющих условиям теоремы Коши.

Функция Обыкновенные дифференциальные уравнения, зависящая от х и двух произвольных постоянных Обыкновенные дифференциальные уравнения, называется общим решением уравнения (1) в некоторой области G, если она является решением уравнения (1) при любых значениях постоянных Обыкновенные дифференциальные уравнения и если при любых начальных условиях (3) существуют единственные значения постоянных Обыкновенные дифференциальные уравнения такие, что функция Обыкновенные дифференциальные уравнения удовлетворяет данным начальным условиям.

Любая функция Обыкновенные дифференциальные уравнения, получающаяся из общего решения Обыкновенные дифференциальные уравнения уравнения (1) при определенных значениях постоянных Обыкновенные дифференциальные уравнения называется частным решением.

Рассмотрим, например, уравнение у»=2. Это уравнение второго порядка. Так как функции Обыкновенные дифференциальные уравнения и Обыкновенные дифференциальные уравненияопределены и непрерывны во всем пространстве переменных (х; у; у’), то оно удовлетворяет во всем пространстве требованиям теоремы Коши.

Общее решение данного уравнения найдем двукратным последовательным интегрированием. Последовательно интегрируя, находим сначала первую производную Обыкновенные дифференциальные уравнения а затем и общее решение:

Обыкновенные дифференциальные уравнения

где Обыкновенные дифференциальные уравнения—произвольные постоянные. Геометрически общее решение представляет собой семейство парабол, причем так как оно зависит от двух произвольных постоянных, то через каждую точку плоскости проходит бесконечное множество парабол, имеющих различные касательные в этой точке. Поэтому для выделения одной параболы из полученного семейства кроме точки Обыкновенные дифференциальные уравнения, через которую проходит парабола, нужно задать еще угловой коэффициент у о касательной к параболе в этой точке.

Найдем, например, частное решение данного уравнения при начальных условиях

Обыкновенные дифференциальные уравнения

Подставляя эти значения в выражения для общего решения Обыкновенные дифференциальные уравнения и его производной Обыкновенные дифференциальные уравнения, для определения Обыкновенные дифференциальные уравнения получаем систему уравнений

Обыкновенные дифференциальные уравнения

откуда находим Обыкновенные дифференциальные уравнения. Следовательно, искомым частным решением является функция

Обыкновенные дифференциальные уравнения

график которой — парабола, проходящая через точку (1; 1) с угловым коэффициентом в этой точке, равным единице.

Уравнения второго порядка, допускающие понижение порядка

Рассмотрим три частных случая, когда решение уравнения (1) с помощью замены переменной сводится к решению уравнения первого порядка. Такое преобразование уравнения (1) называется понижением порядка.

1) Уравнение вида у» = f(х). Уравнение не содержит у и у’. Введем новую функцию z(x), полагая z(x) = y’. Тогда z'(x) = y», и уравнение превращается в уравнение первого порядка: z'(x) = f(x) с искомой функцией z(x). Решая его, находим:

Обыкновенные дифференциальные уравнения

Отсюда, интегрируя еще раз, получаем искомое решение:

Обыкновенные дифференциальные уравнения

где Обыкновенные дифференциальные уравнения — произвольные постоянные.

Пример:

Найти общее решение уравнения у» = х.

Решение:

Полагая z(x)=y’, получаем уравнение первого порядка z'(х) = х. Интегрируя его, найдем Обыкновенные дифференциальные уравнения. Заменяя z(x) на у’ и интегрируя еще раз, находим искомое общее решение:

Обыкновенные дифференциальные уравнения

2)Уравнение вида y» = f(x, у’). Уравнение не содержит у. Положим, как и в предыдущем случае, z(x)=y’; тогда z’ (х)=у», и уравнение преобразуется в уравнение первого порядка относительно Обыкновенные дифференциальные уравнения. Решая его, найдем Обыкновенные дифференциальные уравнения. Так как Обыкновенные дифференциальные уравнения. Отсюда, интегрируя еще раз, получаем искомое решение

Обыкновенные дифференциальные уравнения

где Обыкновенные дифференциальные уравнения — произвольные постоянные.

Пример:

Найти общее решение уравнения Обыкновенные дифференциальные уравнения

Решение:

Полагая z(х) = у’, получаем линейное уравнение первого порядка Обыкновенные дифференциальные уравнения. Решая его, найдем Обыкновенные дифференциальные уравнения

Тогда Обыкновенные дифференциальные уравнения искомое решение.

3)Уравнение вида у»=f(y, у’). Уравнение не содержит х. Вводим новую функцию z(y), полагая y’ = z. Тогда

Обыкновенные дифференциальные уравнения

Подставляя в уравнение выражения для у’ и у», получаем уравнение первого порядка относительно z как функции от у:

Обыкновенные дифференциальные уравнения

Решая его, найдем Обыкновенные дифференциальные уравнения. Так как Обыкновенные дифференциальные уравнения, то Обыкновенные дифференциальные уравнения.

Отсюда Обыкновенные дифференциальные уравнения. Получено уравнение с разделяющимися переменными, из которого находим общее решение данного уравнения:

Обыкновенные дифференциальные уравнения

где Обыкновенные дифференциальные уравнения— произвольные постоянные.

Пример:

Найти общее решение уравнения Обыкновенные дифференциальные уравнения

Решение:

Полагая y’=z(y) и учитывая, что Обыкновенные дифференциальные уравнения получаем Обыкновенные дифференциальные уравнения. Это уравнение первого порядка с разделяющимися переменными. Приводя его к виду Обыкновенные дифференциальные уравнения и интегрируя, имеем Обыкновенные дифференциальные уравнения, откуда Обыкновенные дифференциальные уравнения. Учитывая, что Обыкновенные дифференциальные уравнения находим: Обыкновенные дифференциальные уравнения, откуда получаем искомое решение

Обыкновенные дифференциальные уравнения

При сокращении на z было потеряно решение уравнения z=y’=0, т. е. у=С=const. В данном случае оно содержится в общем решении, так как получается из него при Обыкновенные дифференциальные уравнения=0 (за исключением решения у=0).

Дифференциальные уравнения высших порядков

Ограничимся только основными определениями и общими замечаниями, относящимися к дифференциальным уравнениям n-го порядка.

Дифференциальное уравнение n-го порядка имеет вид

Обыкновенные дифференциальные уравнения

или, если оно разрешено относительно старшей производной,

Обыкновенные дифференциальные уравнения

Решением уравнения (4), как и уравнений первого и второго порядков, называется функция Обыкновенные дифференциальные уравнения, которая при подстановке в уравнение обращает его в тождество.

Теорема существования и единственности решения уравнения (4) аналогична соответствующим теоремам, приведенным ранее для случаев n=1 и n=2.

Общее решение уравнения (4) зависит от х и n произвольных постоянных и может быть записано в виде

Обыкновенные дифференциальные уравнения

Решения, получающиеся из общего при определенных значениях постоянных Обыкновенные дифференциальные уравнения, называются частными решениями уравнения (4). Чтобы выделить частное решение уравнения из общего (4), можно задать начальные условия

Обыкновенные дифференциальные уравнения

Отыскание решения уравнения (4), удовлетворяющего заданным начальным условиям (5), называется решением задачи Коши для этого уравнения.

Простейшим уравнением вида (4) является уравнение, в котором правая часть зависит только от х, т. е. уравнение вида

Обыкновенные дифференциальные уравнения

Это уравнение легко решается. Действительно, интегрируя последовательно n раз, получаем

Обыкновенные дифференциальные уравнения

где Обыкновенные дифференциальные уравнения — произвольные постоянные. Функция (7) и является общим решением уравнения (6).

Пример:

Найти общее решение уравнения третьего порядка у»‘=е’ и выделить из него частное решение, удовлетворяющее следующим начальным условиям:

Обыкновенные дифференциальные уравнения

Решение:

Последовательно интегрируя, находим Обыкновенные дифференциальные уравненияОбыкновенные дифференциальные уравнения. Интегрируя еще раз, получаем общее решение данного уравнения:

Обыкновенные дифференциальные уравнения

Подставляя в выражения для у, у’, у» начальные условия, имеем: Обыкновенные дифференциальные уравнения откуда находим Обыкновенные дифференциальные уравнения. Итак, Обыкновенные дифференциальные уравнения — искомое частное решение.

Линейные дифференциальные уравнения второго порядка

Линейные дифференциальные уравнения второго порядка в теории дифференциальных уравнений занимают важное место не только потому, что представляют собой простой и хорошо изученный тип уравнений, но и потому, что многие практические задачи физики, механики, техники и особенно электротехники приводят к решению этих уравнений.

Определение:

Уравнение вида

Обыкновенные дифференциальные уравнения

где у — искомая функция, а р (х), q (х) и f (х) — непрерывные функции на некотором интервале (а, b), называется линейным дифференциальным уравнением второго порядка.

Если f(x)Обыкновенные дифференциальные уравнения0, то уравнение (!) называется линейным однородным уравнением. Если же f (х)Обыкновенные дифференциальные уравнения0, то уравнение (1) называется линейным неоднородным уравнением.

Разрешая уравнение (1) относительно у» : Обыкновенные дифференциальные уравненияОбыкновенные дифференциальные уравнения, видим, что оно является частным случаем уравнения Обыкновенные дифференциальные уравнения и удовлетворяет условиям теоремы существования и единственности решения. Действительно, функция Обыкновенные дифференциальные уравненияОбыкновенные дифференциальные уравнения— непрерывная как функция трех переменных х, у, и у’ (она зависит от у и у’ линейно, а функции р (х), q (х) и f(х) непрерывны по условию), частные производные Обыкновенные дифференциальные уравнения также являются непрерывными функциями трех переменных х, у и у’ (от у и у’ р (х) и q (х) не зависят, а как функции х непрерывны по условию). Поэтому при любых начальных условиях

Обыкновенные дифференциальные уравненияОбыкновенные дифференциальные уравнения

где Обыкновенные дифференциальные уравнения, уравнение (1) имеет единственное решение задачи Коши.

Изучение линейных дифференциальных уравнений мы начнем с однородных уравнений.

Линейные однородные дифференциальные уравнения второго порядка

Рассмотрим некоторые свойства решении линейных однородных уравнении.

Теорема:

Если функции Обыкновенные дифференциальные уравнения — решения уравнения

Обыкновенные дифференциальные уравнения

то функция Обыкновенные дифференциальные уравнения при любых значениях постоянных Обыкновенные дифференциальные уравнения также является решением уравнения (2).

Доказательство:

Продифференцировав дважды функцию Обыкновенные дифференциальные уравнения и подставив выражения для у, у’ и у в левую часть уравнения (2), получим

Обыкновенные дифференциальные уравнения

Так как функции Обыкновенные дифференциальные уравнения по условию являются решениями уравнения (2), то выражения в квадратных скобках тождественно равны нулю, а это значит, что функция Обыкновенные дифференциальные уравнения — решение уравнения (2).

Итак, функция вида Обыкновенные дифференциальные уравнения с произвольными постоянными Обыкновенные дифференциальные уравнения является решением уравнения (2). Естественно возникает вопрос, не является ли это решение общим решением уравнения (2). Докажем, что при некоторых условиях функция Обыкновенные дифференциальные уравнения является общим решением уравнения (2). Предварительно введем понятия линейной зависимости и линейной независимости функций Обыкновенные дифференциальные уравнения.

Функции Обыкновенные дифференциальные уравнения называются линейно зависимыми на (а, b), если существуют такие числа Обыкновенные дифференциальные уравнения, из которых хотя бы одно отлично от нуля, что для любого Обыкновенные дифференциальные уравнения имеет место равенство

Обыкновенные дифференциальные уравнения

Очевидно, что если функции Обыкновенные дифференциальные уравнения линейно зависимы, то они пропорциональны. Действительно, если Обыкновенные дифференциальные уравненияпричем Обыкновенные дифференциальные уравнения, то Обыкновенные дифференциальные уравнения. Верно и обратное.

Функции Обыкновенные дифференциальные уравнения называются линейно независимыми на (a, b), если не существует таких чисел Обыкновенные дифференциальные уравнения, из которых хоть одно отлично от нуля, что для любого Обыкновенные дифференциальные уравнения имеет место равенство (3).

Другими словами, равенство (3) выполняется сразу для всех Обыкновенные дифференциальные уравнения, если только Обыкновенные дифференциальные уравнения.

Очевидно, что если функции Обыкновенные дифференциальные уравнения линейно независимы, то их отношение Обыкновенные дифференциальные уравнения, т. е. они не пропорциональны.

Так, например, функции Обыкновенные дифференциальные уравнения линейно независимы на любом интервале (а, b), поскольку Обыкновенные дифференциальные уравнения, а функции Обыкновенные дифференциальные уравнения линейно зависимы на любом
промежутке, так как Обыкновенные дифференциальные уравнения

Предположим теперь, что функции Обыкновенные дифференциальные уравнения являются решениями уравнения (2). Вопрос о том, являются ли они линейно зависимыми или линейно назависимыми, решают с помощью определителя Вронского:

Обыкновенные дифференциальные уравнения

Определитель Вронского (или вронскиан) является функцией, определенной на (a, b), и обозначается Обыкновенные дифференциальные уравнения или просто Обыкновенные дифференциальные уравнения.

Теорема:

Если функции Обыкновенные дифференциальные уравнения линейно зависимы на (а, b), то определитель Вронского, составленный из них, равен нулю на этом интервале.

Доказательство:

Так как по условию функции Обыкновенные дифференциальные уравнения линейно зависимы, то по определению существуют числа Обыкновенные дифференциальные уравнения, из которых хотя бы одно отлично от нуля, такие, что имеет место равенство (3): Обыкновенные дифференциальные уравнения. Пусть, например, Обыкновенные дифференциальные уравнения. Тогда из равенства (3) следует, что

Обыкновенные дифференциальные уравнения

Подставляя выражения для Обыкновенные дифференциальные уравнения в определитель Вронского, получаем

Обыкновенные дифференциальные уравнения

Теорема:

Если решения Обыкновенные дифференциальные уравнения уравнения (2) линейно независимы на (а, b), то определитель Вронского, составленный из них, отличен от нуля на этом интервале.

Доказательство:

Допустим обратное, т. е. предположим, что существует точка Обыкновенные дифференциальные уравнения, в которой определитель Вронского Обыкновенные дифференциальные уравнения. Составим систему уравнений

Обыкновенные дифференциальные уравнения

в которой Обыкновенные дифференциальные уравнения— неизвестные числа. Так как определитель этой системы Обыкновенные дифференциальные уравнения, то система (5) имеет (гл. 10, § 3, случай 2) не нулевое решение относительно Обыкновенные дифференциальные уравнения (т. е. хотя бы одно из этих чисел отлично от нуля). Рассмотрим функцию

Обыкновенные дифференциальные уравнения

где Обыкновенные дифференциальные уравнения— ненулевое решение системы (5).

По теореме 15.3 эта функция является решением уравнения (2). Кроме того, поскольку Обыкновенные дифференциальные уравнения— решение системы (5), функция у(х) удовлетворяет нулевым начальным условиям

Обыкновенные дифференциальные уравнения

Таким начальным условиям, очевидно, удовлетворяет решение у(х) = 0. По теореме существования и единственности решение у(х)=0 является единственным решением уравнения (2) с начальными условиями (6). Следовательно, Обыкновенные дифференциальные уравнения на интервале (а, b), а это означает, что функции у, (х) и у2(х) линейно зависимы на (а, b), что противоречит условию теоремы. Таким образом, Обыкновенные дифференциальные уравнения для всех Обыкновенные дифференциальные уравнения.

Итак, установлено, что если функции Обыкновенные дифференциальные уравнения являются на (а, b) решениями линейного однородного уравнения (2), то составленный из них определитель Вронского на (а, b) либо равен нулю Обыкновенные дифференциальные уравнения линейно зависимы), либо отличен от нуля Обыкновенные дифференциальные уравнения линейно независимы).

Установим теперь, при каких условиях функция Обыкновенные дифференциальные уравненияОбыкновенные дифференциальные уравнения является общим решением линейного однородного уравнения (2).

Теорема:

Если функции Обыкновенные дифференциальные уравнения — линейно независимые на (а, b) решения уравнения (2), то функция

Обыкновенные дифференциальные уравнения

где Обыкновенные дифференциальные уравнения — произвольные постоянные, является общим решением уравнения (2).

Доказательство:

Напомним, что в силу теоремы 15.3 функция Обыкновенные дифференциальные уравнения при любых значениях постоянных Обыкновенные дифференциальные уравнения является решением уравнения (2). Для того чтобы доказать, что эта функция — общее решение уравнения (2), достаточно установить, что из него можно выделить частное решение, удовлетворяющее любым заданным начальным условиям.

Пусть Обыкновенные дифференциальные уравнения и

Обыкновенные дифференциальные уравнения

— произвольные начальные условия. Покажем, что постоянные Обыкновенные дифференциальные уравнения можно подобрать так, что решение (7) при этих значениях постоянных является частным решением, удовлетворяющим заданным начальным условиям (8).

Составим систему уравнений

Обыкновенные дифференциальные уравнения

в которой Обыкновенные дифференциальные уравнения — неизвестные числа. Определитель этой системы есть определитель Вронского Обыкновенные дифференциальные уравнения. Так как по условию функции Обыкновенные дифференциальные уравнения— линейно независимы на (а, b), то в силу теоремы 15.5 Обыкновенные дифференциальные уравнения. Поэтому система (9) имеет единственное решение, которое обозначим Обыкновенные дифференциальные уравнения. Подставляя Обыкновенные дифференциальные уравнения в равенство (7), получаем искомое частное решение уравнения (2): у=Обыкновенные дифференциальные уравнения, удовлетворяющее условиям (8). Это и означает, что решение (7) является общим решением уравнения (2).

Из доказанной теоремы следует, что для отыскания общего решения уравнения (2) достаточно найти два линейно независимых частных решения и составить выражения (7) с произвольными постоянными Обыкновенные дифференциальные уравнения.

Пример:

Найти общее решение уравнения у»—у= 0. Решение. Имеем линейное однородное уравнение. Легко заметить, что его частными решениями являются Обыкновенные дифференциальные уравненияОбыкновенные дифференциальные уравнения. Так как определитель Вронского

Обыкновенные дифференциальные уравнения

отличен от нуля, то эти решения линейно независимы на всей числовой прямой. Следовательно, общее решение данного уравнения можно записать в виде Обыкновенные дифференциальные уравнения где Обыкновенные дифференциальные уравнения— произвольные постоянные.

В заключение покажем, как найти общее решение уравнения (2), если известно только одно частное решение этого уравнения. Пусть Обыкновенные дифференциальные уравнения — частное решение уравнения (2). Введем новую функцию z, полагая Обыкновенные дифференциальные уравнения. Тогда Обыкновенные дифференциальные уравненияОбыкновенные дифференциальные уравнения. Подставляя выражения для у, у’ и у» в уравнение (2) и группируя слагаемые, получаем

Обыкновенные дифференциальные уравнения

Так как Обыкновенные дифференциальные уравнения—решение уравнения (2), то выражение в первых квадратных скобках равно нулю и уравнение принимает вид

Обыкновенные дифференциальные уравнения

Порядок этого уравнения можно понизить, полагая Обыкновенные дифференциальные уравнения, где Обыкновенные дифференциальные уравнения — новая искомая функция:

Обыкновенные дифференциальные уравнения

Получено уравнение первого порядка относительно функции и с разделяющимися переменными. Решая его, находим

Обыкновенные дифференциальные уравнения

где Обыкновенные дифференциальные уравнения — произвольная постоянная. Возвращаясь к переменной z и умножая выражение для z на Обыкновенные дифференциальные уравнения получаем общее решение уравнения (2):

Обыкновенные дифференциальные уравнения

где Обыкновенные дифференциальные уравнения — произвольные постоянные.

В качестве примера найдите самостоятельно общее решение уравнения у»—у=0 (см. пример 1), взяв одно из его частных решений за известное.

Линейные неоднородные дифференциальные уравнения второго порядка

Рассмотрим теперь основные свойства решений линейного неоднородного дифференциального уравнения второго порядка (1):

Обыкновенные дифференциальные уравнения

Имеет место следующая теорема.

Теорема:

Общее решение уравнения (1) есть сумма любого его частного решения и общего решения соответствующего однородного уравнения.

Доказательство:

Пусть у (х) — частное решение уравнения (1), a Обыкновенные дифференциальные уравнения — общее решение соответствующего однородного уравнения (2), где Обыкновенные дифференциальные уравнения — произвольные постоянные. Покажем, что функция

Обыкновенные дифференциальные уравнения

— решение уравнения (1). Для этого найдем Обыкновенные дифференциальные уравненияОбыкновенные дифференциальные уравнения и подставим их в уравнение (1):

Обыкновенные дифференциальные уравнения

Отсюда следует, что функция Обыкновенные дифференциальные уравнения действительно решение уравнения (1).

Покажем теперь, что функция (10) — общее решение уравнения (1). Для этого возьмем любое решение у уравнения (1) и рассмотрим разность Обыкновенные дифференциальные уравнения. Эта разность является решением однородного уравнения (2). Действительно,

Обыкновенные дифференциальные уравнения

Это означает, что разность Обыкновенные дифференциальные уравнения может быть записана в виде

Обыкновенные дифференциальные уравнения

где Обыкновенные дифференциальные уравнения — определение значения постоянных Обыкновенные дифференциальные уравнения. Итак, любое решение у уравнения (1) получается из формулы (10) при соответствующем подборе произвольных постоянных Обыкновенные дифференциальные уравнения, т. е. функция (10) является общим решением уравнения (1).

Таким образом, чтобы найти общее решение линейного неоднородного уравнения, нужно найти общее решение соответствующего однородного уравнения и какое-нибудь частное решение неоднородного уравнения. В общем случае задача отыскания частного решения является сложной. Покажем, как можно найти частное решение неоднородного уравнения методом вариации произвольных постоянных, если известно общее решение соответствующего однородного уравнения.

Пусть Обыкновенные дифференциальные уравнения — общее решение однородного уравнения (2). Будем искать частное решение неоднородного уравнения (1) в виде

Обыкновенные дифференциальные уравнения

рассматривая Обыкновенные дифференциальные уравнения как некоторые искомые функции от х. Продифференцируем последнее равенство

Обыкновенные дифференциальные уравнения

Подберем функции Обыкновенные дифференциальные уравнения так, чтобы выполнялось равенство

Обыкновенные дифференциальные уравнения

Тогда равенство (12) принимает вид

Обыкновенные дифференциальные уравнения

Дифференцируя это равенство, найдем у»:

Обыкновенные дифференциальные уравнения

Подставляя выражения для у, у’ и у» в уравнение (1) и группируя слагаемые, получаем

Обыкновенные дифференциальные уравнения

Выражения в квадратных скобках равны нулю, так как Обыкновенные дифференциальные уравнения — решения однородного уравнения. Поэтому последнее равенство принимает вид

Обыкновенные дифференциальные уравнения

Таким образом, функция (11) является решением уравнения (1), если функции Обыкновенные дифференциальные уравнения удовлетворяют уравнениям (13) и (14). Объединяя их, получаем систему уравнений

Обыкновенные дифференциальные уравнения

в которой Обыкновенные дифференциальные уравнения — неизвестны, а Обыкновенные дифференциальные уравнения и f (х) — неизвестны. Так как определителем этой системы является определитель Вронского

Обыкновенные дифференциальные уравнения

составленный из линейно независимых решений Обыкновенные дифференциальные уравнения однородного уравнения (2), то он по теореме 15.5 не равен нулю, а значит, система (15) имеет единственное решение относительно Обыкновенные дифференциальные уравнения. Решая эту систему, получаем

Обыкновенные дифференциальные уравнения

где Обыкновенные дифференциальные уравнения—известные функции, откуда, интегрируя, найдем Обыкновенные дифференциальные уравнения. Подставляя полученные выражения для Обыкновенные дифференциальные уравнения в равенство (11), получаем искомое частное решение уравнения (1).

Пример:

Найти частное решение уравнения у»—у=х. Решение. В примере 1 п. 2 было найдено общее решение Обыкновенные дифференциальные уравнения соответствующего однородного уравнения у»—у=0. Поэтому частное решение неоднородного уравнения будем искать в виде

Обыкновенные дифференциальные уравнения

Система (15) для нахождения Обыкновенные дифференциальные уравнения в данном случае имеет вид

Обыкновенные дифференциальные уравнения

Складывая эти уравнения, найдем Обыкновенные дифференциальные уравнения. Отсюда, интегрируя, получаем

Обыкновенные дифференциальные уравнения

Произвольную постоянную не пишем, так как ищем какое-нибудь частное решение. Подставляя выражение Обыкновенные дифференциальные уравнения в первое из уравнений системы, найдем Обыкновенные дифференциальные уравненияоткуда интегрируя, получаем

Обыкновенные дифференциальные уравнения

Подставляя найденные выражения Обыкновенные дифференциальные уравнения в равенство (16), получаем частное решение у данного неоднородного уравнения:

Обыкновенные дифференциальные уравнения

Заметим, что, найдя частное решение неоднородного уравнения и зная общее решение соответствующего однородного уравнения, на основании равенства (10) можно записать общее решение данного неоднородного уравнения:

Обыкновенные дифференциальные уравнения

где Обыкновенные дифференциальные уравнения — произвольные постоянные.

Линейные дифференциальные уравнения второго порядка с постоянными коэффициентами

Рассмотрим важный частный случай линейных дифференциальных уравнений второго порядка — случай, когда функции р (х) и q (х) являются постоянными величинами. Такие уравнения называются линейными уравнениями с постоянными коэффициентами.

Линейные однородные дифференциальные уравнения второго порядка с постоянными коэффициентами

Рассмотрим линейное однородное уравнение второго порядка

Обыкновенные дифференциальные уравнения

где р и q — вещественные числа.

Теорема:

Если число к — вещественный корень уравнения

Обыкновенные дифференциальные уравнения

то функция Обыкновенные дифференциальные уравнения является решением уравнения (1).

2) Если числа Обыкновенные дифференциальные уравнения— комплексные корни уравнения (2), то функции Обыкновенные дифференциальные уравнения являются решениями уравнения (1).

Доказательство:

1) Пусть Обыкновенные дифференциальные уравнения. Тогда Обыкновенные дифференциальные уравнения Подставляя у, у’ и у» в уравнение (1), получаем

Обыкновенные дифференциальные уравнения

Так как Обыкновенные дифференциальные уравнения, то, сокращая на Обыкновенные дифференциальные уравнения имеем

Обыкновенные дифференциальные уравнения

Следовательно, если к является корнем уравнения (2), то функция Обыкновенные дифференциальные уравнения—решение уравнения (1).

2)Аналогично случаю 1) можно проверить, что функции Обыкновенные дифференциальные уравненияОбыкновенные дифференциальные уравнения удовлетворяют уравнению (1).

Уравнение (2) называется характеристическим уравнением данного уравнения (1).

Характеристическое уравнение (2) является квадратным уравнением и, следовательно, имеет два корня. Обозначим их Обыкновенные дифференциальные уравнения.

Теорема :

1) Если корни характеристического уравнения вещественные и различные Обыкновенные дифференциальные уравнения, то общее решение уравнения (1) имеет вид

Обыкновенные дифференциальные уравнения

2) если корни характеристического уравнения вещественные и равные Обыкновенные дифференциальные уравнения, то общее решение имеет вид

Обыкновенные дифференциальные уравнения

3)если корни характеристического уравнения комплексные Обыкновенные дифференциальные уравненияОбыкновенные дифференциальные уравнения, то общее решение имеет вид

Обыкновенные дифференциальные уравнения

Доказательство:

1) Пусть корни Обыкновенные дифференциальные уравнения различны. По теореме 15.8 функции Обыкновенные дифференциальные уравнения —частные решения уравнения (1). Эти решения линейно независимы, так как Обыкновенные дифференциальные уравненияОбыкновенные дифференциальные уравнения. Следовательно, по теореме 15.6 общее решение уравнения (1) имеет вид

Обыкновенные дифференциальные уравнения

2)Пусть корни Обыкновенные дифференциальные уравнения равны:Обыкновенные дифференциальные уравнения. По теореме 15.8 функция Обыкновенные дифференциальные уравнения —частное решение уравнения (1). Найдем второе частное решение, линейно независимое с первым. Будем искать его в виде Обыкновенные дифференциальные уравнения где z (х) — новая неизвестная функция. Дифференцируя, имеем

Обыкновенные дифференциальные уравнения

Подставляя Обыкновенные дифференциальные уравнения в левую часть уравнения (1), получаем

Обыкновенные дифференциальные уравнения

По условию, Обыкновенные дифференциальные уравнения. Кроме того, Обыкновенные дифференциальные уравнения, поэтому Обыкновенные дифференциальные уравнения. Следовательно, для того чтобы найти функцию z(x), надо решить уравнение z»=0. Последовательно интегрируя, получаем: Обыкновенные дифференциальные уравнения, где Обыкновенные дифференциальные уравнения—произвольные постоянные. Полагая Обыкновенные дифференциальные уравнения, найдем z(х) = х. Таким образом, Обыкновенные дифференциальные уравнения — второе частное решение уравнения (1). Решения линейно независимы, так как Обыкновенные дифференциальные уравнения и по теореме 15.6 общее решение уравнения (1) имеет вид

Обыкновенные дифференциальные уравнения

3)Пусть Обыкновенные дифференциальные уравнения—комплексно-сопряженные корни, т. е. Обыкновенные дифференциальные уравненияОбыкновенные дифференциальные уравнения. Тогда по теореме 15.8 функции Обыкновенные дифференциальные уравнения являются частными решениями уравнения (1). Эти решения линейно независимы, так как Обыкновенные дифференциальные уравненияОбыкновенные дифференциальные уравнения . Поэтому общее решение уравнения (1) имеет вид

Обыкновенные дифференциальные уравнения

Пример:

Найти общее решение уравнения Обыкновенные дифференциальные уравнения.

Решение:

Характеристическое уравнение имеет вид Обыкновенные дифференциальные уравненияОбыкновенные дифференциальные уравнения; его корни Обыкновенные дифференциальные уравнения вещественные и различные.

Соответствующие частные решения уравнения Обыкновенные дифференциальные уравнения. Общее решение уравнения имеет вид Обыкновенные дифференциальные уравнения

Пример:

Найти общее решение уравнения.

Решение:

Характеристическое уравнение имеет вид Обыкновенные дифференциальные уравненияОбыкновенные дифференциальные уравнения; его корни Обыкновенные дифференциальные уравнения вещественные и равные. Соответствующие частные решения уравнения Обыкновенные дифференциальные уравнения. Общее решение уравнения имеет вид Обыкновенные дифференциальные уравненияОбыкновенные дифференциальные уравнения

Пример:

Найти общее решение уравнения Обыкновенные дифференциальные уравнения

Решение:

Характеристическое уравнение имеет вид Обыкновенные дифференциальные уравненияОбыкновенные дифференциальные уравнения; его корни Обыкновенные дифференциальные уравнения комплексные.

Соответствующие частные решения уравнения Обыкновенные дифференциальные уравненияОбыкновенные дифференциальные уравнения. Общее решение уравнения имеет вид Обыкновенные дифференциальные уравненияОбыкновенные дифференциальные уравнения.

Линейные неоднородные дифференциальные уравнения второго порядка с постоянными коэффициентами

Рассмотрим линейное неоднородное уравнение второго порядка

Обыкновенные дифференциальные уравнения

где p и q — вещественные числа; f (х) — непрерывная функция.

Как известно, общее решение такого уравнения представляет собой сумму частного решения неоднородного уравнения и общего решения соответствующего однородного уравнения. Общее решение однородного уравнения мы находить умеем, поэтому остается рассмотреть вопрос о нахождении частного решения. Для нахождения частного решения можно применять метод вариации произвольных постоянных. Однако если в правой части уравнения (3) — многочлен, либо показательная функция, либо тригонометрическая функция Обыкновенные дифференциальные уравнения, либо линейная комбинация перечисленных функций, то частное решение может быть найдено методом неопределенных коэффициентов, не содержащим процесса интегрирования.

Рассмотрим различные виды правых частей уравнения (3).

1) Правая часть имеет вид

Обыкновенные дифференциальные уравнения

где Обыкновенные дифференциальные уравнения — многочлен степени п. Тогда частное решение у можно искать в виде

Обыкновенные дифференциальные уравнения

где Qn (х) — многочлен той же степени, что и Обыкновенные дифференциальные уравнения, а r — число корней характеристического уравнения, равных нулю.

Пример:

Найти общее решение уравнения Обыкновенные дифференциальные уравненияОбыкновенные дифференциальные уравнения

Решение:

Общее решение соответствующего однородного уравнения имеет вид Обыкновенные дифференциальные уравнения (см. пример 2). Так как правая часть уравнения — многочлен первой степени и ни один из корней характеристического уравнения Обыкновенные дифференциальные уравнения не равен нулю Обыкновенные дифференциальные уравнения, то частное решение ищем в виде

Обыкновенные дифференциальные уравнения

где А и В — неизвестные коэффициенты. Дифференцируя дважды Обыкновенные дифференциальные уравнения и подставляя у, у’ и у» в данное уравнение, найдем

Обыкновенные дифференциальные уравнения

Приравнивая коэффициенты при одинаковых степенях х в обеих частях равенства: A= 1, — 2A+В=1, находим: A=1, В=3.

Итак, частное решение данного уравнения имеет вид Обыкновенные дифференциальные уравнения, а его общее решение

Обыкновенные дифференциальные уравнения

2)Правая часть имеет вид

Обыкновенные дифференциальные уравнения

где Обыкновенные дифференциальные уравнения— многочлен степени п. Тогда частное решение у следует искать в виде

Обыкновенные дифференциальные уравнения

где Q(x)— многочлен той же степени, что и Обыкновенные дифференциальные уравнения, а г — число корней характеристического уравнения равных а. Если а=0, то f (х) = Обыкновенные дифференциальные уравнения, т. е. имеет место случай 1).

Пример:

Найти общее решение уравнения Обыкновенные дифференциальные уравненияОбыкновенные дифференциальные уравнения

Решение:

Характеристическое уравнение Обыкновенные дифференциальные уравнения имеет корни Обыкновенные дифференциальные уравнения. Значит, общее решение соответствующего однородного уравнения имеет вид Обыкновенные дифференциальные уравнения. В правой части этого уравнения — произведение многочлена первой степени на показательную функцию Обыкновенные дифференциальные уравнения при а=1. Так как среди корней характеристического уравнения имеется только один корень Обыкновенные дифференциальные уравненияОбыкновенные дифференциальные уравнения, то Обыкновенные дифференциальные уравнения. В данном случае Обыкновенные дифференциальные уравнения= х—многочлен первой степени. Поэтому частное решение данного уравнения ищем в виде

Обыкновенные дифференциальные уравнения

Дифференцируя и подставляя в уравнение, получаем

Обыкновенные дифференциальные уравнения

Приравнивая коэффициенты при одинаковых степенях х в обеих частях равенства: Обыкновенные дифференциальные уравнения, находим: Обыкновенные дифференциальные уравненияОбыкновенные дифференциальные уравнения. Подставляя найденные значения А и В в выражение для у, получаем частное решение данного уравнения Обыкновенные дифференциальные уравненияОбыкновенные дифференциальные уравнения; общее решение имеет вид

Обыкновенные дифференциальные уравнения

3)Правая часть имеет вид

Обыкновенные дифференциальные уравнения

где Обыкновенные дифференциальные уравнения — известные числа. Тогда частное решение у надо искать в виде

Обыкновенные дифференциальные уравнения

где А и В — неизвестные коэффициенты, а r — число корней характеристического уравнения, равных Обыкновенные дифференциальные уравнения.

Пример:

Найти общее решение уравнения Обыкновенные дифференциальные уравнения

Решение:

Характеристическое уравнение Обыкновенные дифференциальные уравнения имеет корни Обыкновенные дифференциальные уравнения. Поэтому общее решение соответствующего однородного уравнения Обыкновенные дифференциальные уравнения. В правой части равенства — тригонометрическая функция sin х, т. е. а=0, b=1, B=1. Так как Обыкновенные дифференциальные уравнения— корень характеристического уравнения, то r= 1 и частное решение надо искать в виде

Обыкновенные дифференциальные уравнения

Дифференцируя и подставляя в уравнение, получаем

Обыкновенные дифференциальные уравнения

откуда Обыкновенные дифференциальные уравнения. Таким образом, частное решение Обыкновенные дифференциальные уравненияОбыкновенные дифференциальные уравнения общее решение уравнения

Обыкновенные дифференциальные уравнения

Пример:

Найти общее решение уравнения Обыкновенные дифференциальные уравнения

Решение:

Данное уравнение отличается от предыдущего только тем, что Обыкновенные дифференциальные уравнения= 2. Так как Обыкновенные дифференциальные уравнения не является корнем характеристического уравнения, то r=0 и частное решение следует искать в виде

Обыкновенные дифференциальные уравнения

Дифференцируя и подставляя в уравнение, получаем

Обыкновенные дифференциальные уравнения

откуда А=0, В =-1/3, т. е. частное решение Обыкновенные дифференциальные уравнения, общее решение уравнения

Обыкновенные дифференциальные уравнения

4) Правая часть имеет вид

Обыкновенные дифференциальные уравнения

где Обыкновенные дифференциальные уравнения — многочлен степени n, а Обыкновенные дифференциальные уравнения — многочлен степени m. Тогда частное решение следует искать в виде

Обыкновенные дифференциальные уравнения

где Обыкновенные дифференциальные уравнения — многочлены степени Обыкновенные дифференциальные уравнения — число корней характеристического уравнения, равных Обыкновенные дифференциальные уравнения.

Пример:

Найти общее решение уравнения Обыкновенные дифференциальные уравнения.

Решение:

Здесь характеристическое уравнение Обыкновенные дифференциальные уравнения имеет корни Обыкновенные дифференциальные уравнения. Общее решение однородного уравнения таково: Обыкновенные дифференциальные уравнения. В правой части уравнения — произведение многочлена нулевой степени, показательной и тригонометрической функций, так что Обыкновенные дифференциальные уравнения. Число Обыкновенные дифференциальные уравнения не является корнем характеристического уравнения, поэтому r=0, и частное решение ищем в виде

Обыкновенные дифференциальные уравнения

Дифференцируя и подставляя в уравнение, получаем

Обыкновенные дифференциальные уравнения

Приравнивая коэффициенты при cos х и sin х, находим

Обыкновенные дифференциальные уравнения

откуда Обыкновенные дифференциальные уравнения. Таким образом, частное решение Обыкновенные дифференциальные уравнения, а общее решение уравнения

Обыкновенные дифференциальные уравнения

Пример:

По данным корням характеристического уравнения и правой части f(х) записать частное решение у линейного неоднородного уравнения:

Обыкновенные дифференциальные уравнения

Решение:

а) Имеем:Обыкновенные дифференциальные уравнения

Так как число Обыкновенные дифференциальные уравнения — корень характеристического уравнения, то r=1. Поэтому Обыкновенные дифференциальные уравнения;

б)имеем: Обыкновенные дифференциальные уравнения. Число Обыкновенные дифференциальные уравнения не является корнем характеристического уравнения, поэтому r=0. Следовательно Обыкновенные дифференциальные уравненияОбыкновенные дифференциальные уравнения;

в)имеем: Обыкновенные дифференциальные уравнения. Так как число Обыкновенные дифференциальные уравнения не является корнем характеристического уравнения, то r=0. Поэтому Обыкновенные дифференциальные уравненияОбыкновенные дифференциальные уравнения;

г)имеем:Обыкновенные дифференциальные уравнения. Число Обыкновенные дифференциальные уравнения — корень характеристического уравнения, поэтому r=1. Следовательно, Обыкновенные дифференциальные уравнения;

д)имеем: Обыкновенные дифференциальные уравнения. Число Обыкновенные дифференциальные уравнения— корень характеристического уравнения, значит, r=1. Следовательно, Обыкновенные дифференциальные уравненияОбыкновенные дифференциальные уравнения

В заключение докажем теорему, которую часто применяют при решении линейных неоднородных уравнений, в правой части которых сумма нескольких слагаемых.

Теорема:

Если Обыкновенные дифференциальные уравнения — решение уравнения

Обыкновенные дифференциальные уравнения

а Обыкновенные дифференциальные уравнения — решение уравнения

Обыкновенные дифференциальные уравнения

то сумма Обыкновенные дифференциальные уравнения является решением уравнения

Обыкновенные дифференциальные уравнения

Доказательство:

Составим сумму Обыкновенные дифференциальные уравнения и подставим ее в левую часть уравнения (6). Получим

Обыкновенные дифференциальные уравнения

так как по условию выражение в первой скобке равно Обыкновенные дифференциальные уравнения, а выражение во второй скобке равно Обыкновенные дифференциальные уравнения. Следовательно, Обыкновенные дифференциальные уравнения — решение уравнения (6).

Пример:

Найти общее решение уравнения Обыкновенные дифференциальные уравненияОбыкновенные дифференциальные уравнения

Решение:

Характеристическое уравнение Обыкновенные дифференциальные уравнения имеет корни Обыкновенные дифференциальные уравнения, поэтому общее решение соответствующего однородного уравнения Обыкновенные дифференциальные уравнения.

Так как правая часть уравнения состоит из суммы двух функций sin х и Обыкновенные дифференциальные уравнения, то в соответствии с теоремой 15.10 частное решение данного уравнения можно искать в виде

Обыкновенные дифференциальные уравнения

где Обыкновенные дифференциальные уравнения— частное решение уравнения Обыкновенные дифференциальные уравнения — частное решение уравнения Обыкновенные дифференциальные уравнения.

Сначала найдем частное решение Обыкновенные дифференциальные уравнения. Так как число Обыкновенные дифференциальные уравнения не является корнем характеристического уравнения (r=0), то частное решение у, будем искать в виде Обыкновенные дифференциальные уравнения. Подставляя Обыкновенные дифференциальные уравнения в уравнение Обыкновенные дифференциальные уравнения и сравнивая коэффициенты при sin х и cos х, получаем Обыкновенные дифференциальные уравнения, откуда Обыкновенные дифференциальные уравнения и, следовательно, Обыкновенные дифференциальные уравнения.

Теперь найдем частное решение Обыкновенные дифференциальные уравнения. Будем его искать в виде Обыкновенные дифференциальные уравнения так как число а= -1 не является корнем характеристического уравнения. Подставляя Обыкновенные дифференциальные уравнения в уравнение Обыкновенные дифференциальные уравнения, имеем А=1/4. Следовательно, Обыкновенные дифференциальные уравнения.

Таким образом, частное решение данного уравнения имеет вид

Обыкновенные дифференциальные уравнения

а общее решение этого уравнения

Обыкновенные дифференциальные уравнения

Применение линейных дифференциальных уравнений к изучению колебательных явлений

Линейные дифференциальные уравнения являются мощным аппаратом в решении задач о колебаниях, занимающих значительное место в современной технике и физике. Познакомимся с одной из них — с задачей о колебании груза, подвешенного на вертикальной пружине.

Постановка задачи. Пусть груз массой m, подвешенный на пружине, движется по вертикальной прямой. Если пружину с грузом оттянуть или сжать, то груз начнет совершать колебания около положения равновесия. Установим закон движения груза, т. е. найдем формулу, выражающую отклонение груза от положения равновесия в любой момент времени t.

Совместим начало координат с положением равновесия груза а ось Оу направим вертикально вверх. Обозначим через /в.расстояние от конца нерастянутой пружины без груза до положения равно-, весия груза, а через у — отклонение груза от положения равновесия в момент времени t (рис. 227). На груз действует сила, равная сумме следующих трех сил: 1) силы . тяжести груза mg, направленной вниз; 2) силы сопротивления среды, направленной в сторону, противоположную движению груза, и по величине пропорциональной скорости движения груза, т. е. равной Обыкновенные дифференциальные уравнения, где Обыкновенные дифференциальные уравнения—коэффициент пропорциональности; 3) упругой силы пружины, направленной вверх (т. е. в положительном направлении оси Оу), величина которой, по закону Гука, пропорциональна деформации, т. е. равна Обыкновенные дифференциальные уравнения, где с — коэффициент пропорциональности, называемый коэффициентом жесткости пружины (масса пружины не учитывается).

Обыкновенные дифференциальные уравнения

Согласно второму закону Ньютона получаем следующее уравнение движения груза:

Обыкновенные дифференциальные уравнения

Так как в положении равновесия {у=0) вес груза mg уравновешивается упругой силой пружины, то Обыкновенные дифференциальные уравнения. Поэтому

Обыкновенные дифференциальные уравнения

Получено дифференциальное уравнение, которое называется уравнением свободных колебаний груза, подвешенного на пружине.

Если на груз действует внешняя сила, направленная вертикально (вдоль оси Оу), величина которой F (t) зависит от времени t, то уравнение (1) принимает вид

Обыкновенные дифференциальные уравнения

Уравнение (2) называется уравнением вынужденных колебаний груза, подвешенного на пружине.

Разделив все члены уравнения (2) на m и обозначая Обыкновенные дифференциальные уравнения,Обыкновенные дифференциальные уравнения получаем окончательный вид уравнения вынужденных колебаний:

Обыкновенные дифференциальные уравнения

Уравнение (3) представляет собой линейное неоднородное дифференциальное уравнение второго порядка с постоянными коэффициентами.

Перейдем теперь к исследованию колебаний, применяя известные решения линейных уравнений с постоянными коэффициентами.

Свободные колебания. Пусть отсутствуют внешняя сила f(t) и сопротивление среды (Обыкновенные дифференциальные уравнения=0). Тогда уравнение (3) принимает вид

Обыкновенные дифференциальные уравнения

Это линейное однородное уравнение. Характеристическое уравнение Обыкновенные дифференциальные уравнения имеет корни Обыкновенные дифференциальные уравнения и общее решение уравнения определяется формулой

Обыкновенные дифференциальные уравнения

где Обыкновенные дифференциальные уравнения — произвольные постоянные. Для удобства дальнейших рассуждений заменим произвольные постоянные Обыкновенные дифференциальные уравненияпостоянными A>0 и ф, полагая Обыкновенные дифференциальные уравнения (отсюда А =Обыкновенные дифференциальные уравнения). Тогда

Обыкновенные дифференциальные уравнения

и общее решение можно записать в виде

Обыкновенные дифференциальные уравнения

Формула (4) выражает закон движения груза, подвешенного на пружине, т. е. отклонение у груза от положения равновесия в любой момент времени t. Согласно этой формуле груз совершает, как говорят, свободные гармонические колебания около положения равновесия. Величина А называется амплитудой колебаний, Обыкновенные дифференциальные уравнения— частотой колебаний и ф — начальной фазой.

Для того чтобы выделить из общего решения частное, необходимо задать начальные условия движения. Пусть в начальный момент времени t=0 отклонение и скорость груза известны:

Обыкновенные дифференциальные уравнения

Тогда, дифференцируя, получаем

Обыкновенные дифференциальные уравнения

подставляя начальные условия (5) в (4) и (6), имеем

Обыкновенные дифференциальные уравнения

Отсюда, выражая произвольные постоянные А и ф и подставляя их значения в (4), получаем искомое частное решение, удовлетворяющее начальным условиям (5).

Из формул (7), в частности, следует, что постоянные А и ф зависят от частоты колебаний о и начальных условий движения. Частота же колебаний не зависит от начальных условий, а зависит от отношения коэффициента жесткости пружины к массе груза Обыкновенные дифференциальные уравнения.

Пусть теперь отсутствует внешняя сила f (t), но имеет место сопротивление среды Обыкновенные дифференциальные уравнения, например сопротивление воздуха. В этом случае уравнение (3) принимает вид

Обыкновенные дифференциальные уравнения

Характеристическое уравнение Обыкновенные дифференциальные уравнения имеет корни

Обыкновенные дифференциальные уравнения

Здесь возможны три случая.

1)Тогда корни Обыкновенные дифференциальные уравненияОбыкновенные дифференциальные уравнения—вещественные, различные и отрицательные. Общее решение уравнения (8) имеет вид

Обыкновенные дифференциальные уравнения

Из полученной формулы следует, что груз колебаний не совершает, при неограниченном возрастании t отклонение груза у бесконечно долго приближается к положению равновесия (Обыкновенные дифференциальные уравнения). В этом случае говорят, что груз совершает непериодическое затухающее движение.

2)Обыкновенные дифференциальные уравнения Тогда корни Обыкновенные дифференциальные уравнения — вещественные равные и отрицательные. Общее решение уравнения (8) имеет вид

Обыкновенные дифференциальные уравнения

В этом случае груз совершает движение, аналогичное предыдущему.

3)Обыкновенные дифференциальные уравнения Тогда корни Обыкновенные дифференциальные уравненияОбыкновенные дифференциальные уравнения— комплексные. Общее решение уравнения (8) имеет вид

Обыкновенные дифференциальные уравнения

где Обыкновенные дифференциальные уравнения. Заменяя Обыкновенные дифференциальные уравнения на постоянные А и ф, запишем решение уравнения (8) в виде

Обыкновенные дифференциальные уравнения

Здесь, в отличие от формулы (4), амплитуда Обыкновенные дифференциальные уравнения зависит от времени t. Так как Обыкновенные дифференциальные уравнения<0, то амплитуда стремится к нулю при Обыкновенные дифференциальные уравнения. Поэтому в данном случае груз совершает свободные затухающие колебания около положения равновесия.

Вынужденные колебания. Резонанс. Рассмотрим теперь случай, когда на колебательную систему действует периодическая внешняя сила f (t) = a sin Обыкновенные дифференциальные уравнения предполагая для простоты, что сопротивление среды отсутствует (Обыкновенные дифференциальные уравнения = 0). В этом случае уравнение (8) принимает вид

Обыкновенные дифференциальные уравнения

Это линейное неоднородное уравнение с постоянными коэффициентами. Известно, что общее решение этого уравнения является суммой общего решения У соответствующего однородного уравнения, которое было найдено выше [см. формулу (4)], и частного решения у неоднородного уравнения, которое надо найти.

Рассмотрим отдельно два случая.

а)Обыкновенные дифференциальные уравнения, т. е. частота внешней периодической силы отлична от частоты свободных колебаний груза. Так как число Обыкновенные дифференциальные уравнения, не совпадает с корнем характеристического уравнения Обыкновенные дифференциальные уравнения, то частное решение можно найти в виде

Обыкновенные дифференциальные уравнения

Дифференцируя у дважды и подставляя у и у» в уравнение (9), найдем: Обыкновенные дифференциальные уравнения. Таким образом,

Обыкновенные дифференциальные уравнения

и общее решение уравнения (9) имеет вид

Обыкновенные дифференциальные уравнения

Как следует из формулы (10),частное решение у определяет колебание системы, создаваемое внешней силой, общее решение У=Обыкновенные дифференциальные уравнения свободное колебание груза, а общее решение у — сложное колебательное движение, получающееся в результате сложения двух колебаний с разными частотами Обыкновенные дифференциальные уравнения.

В этом случае амплитуда постоянна, и если Обыкновенные дифференциальные уравнения, близки по величине, то груз совершает колебания около положения равновесия с большой амплитудой.

б)Обыкновенные дифференциальные уравнения, т. е. частота внешней периодической силы совпадает с частотой свободных колебаний груза. Так как Обыкновенные дифференциальные уравнения— корень характеристического уравнения Обыкновенные дифференциальные уравнения, то в этом случае частное решение следует искать в виде

Обыкновенные дифференциальные уравнения

Дифференцируя у дважды и подставляя Обыкновенные дифференциальные уравнения в уравнение (9), найдем: Обыкновенные дифференциальные уравнения. Таким образом,

Обыкновенные дифференциальные уравнения

и общее решение уравнения (9) имеет вид

Обыкновенные дифференциальные уравнения

Как следует из найденной формулы, в данном случае, как и в предыдущем, имеет место сложное колебательное движение, получающееся в результате сложения двух колебаний, но с одинаковыми частотами.

Наличие множителя t во втором члене свидетельствует, что амплитуда колебаний неограниченно возрастает при неограниченном возрастании времени t, т. е. груз будет совершать через некоторый промежуток времени колебания с очень большой амплитудой, даже если амплитуда а внешней силы мала. Это явление называется резонансом. Иногда оно приводит к разрушению колеблющихся систем.

На примере груза, подвешенного на пружине, рассмотрен случай механических колебаний упругих систем (к ним относится колебание на рессорах вагонов, автомобилей и т. п.). Аналогичное исследование проводится и при изучении электрических, звуковых и многих других колебаний. Главную роль в этих исследованиях играют линейные дифференциальные уравнения.

В заключение отметим, что изложенная теория линейных дифференциальных уравнений второго порядка полностью переносится и на линейные дифференциальные уравнения любого порядка.

Дифференциальные уравнения — решение заданий и задач по всем темам с вычислением

При изучении темы «Дифференциальные уравнения» вы познакомитесь с различными типами уравнений первого и второго порядков и освоите методы их решения. Вы изучите линейные уравнения с постоянными коэффициентами, структуру их общего решения и методы его нахождения (метод Эйлера, метод подбора частных решений, метод Лагранжа).

Понятие решения

Постановка задачи. Доказать, что функция у = у(х) удовлетворяет дифференциальному уравнению F(x,y,y) = 0.

План решения.

Для доказательства того, что функция у = у(х) удовлетворяет
уравнению F(x,y,y’) = 0, достаточно вычислить производную y'(x),
подставить у(х) и у'(х) в это уравнение и убедиться в том, что получается тождество, т.е. F(x,y(x),y'(x)) = 0 для всех допустимых х.

Пример:

Доказать, что функция Дифференциальные уравнения решение задач  и примеры удовлетворяет
уравнению

Дифференциальные уравнения решение задач  и примеры

Решение:

Имеем

Дифференциальные уравнения решение задач  и примеры

Подставим у и у’ в левую часть уравнения и проведем необходимые преобразования:

Дифференциальные уравнения решение задач  и примеры

Получаем тождество Дифференциальные уравнения решение задач  и примеры

Ответ. Функция Дифференциальные уравнения решение задач  и примеры удовлетворяет заданному уравнению.

Уравнения с разделяющимися переменными

Постановка задачи. Найти интегральные кривые дифференциального уравнения вида

E(x)F(y) dx = G(x)H(y) dy. (1)

План решения.

1.В области, где Дифференциальные уравнения решение задач  и примеры и Дифференциальные уравнения решение задач  и примеры разделяем переменные, т.е.
представляем уравнение (1) в виде

Дифференциальные уравнения решение задач  и примеры

2.Вычислим интегралы в уравнении

Дифференциальные уравнения решение задач  и примеры

и преобразуем его к виду Дифференциальные уравнения решение задач  и примеры

3.Ответ записываем в таком виде: интегральные кривые определяются уравнением Дифференциальные уравнения решение задач  и примеры при всевозможных значениях С.

Замечание:

Если одно или оба уравнения G(x) = 0 и F(y) = 0
имеют решения Дифференциальные уравнения решение задач  и примеры то равенства Дифференциальные уравнения решение задач  и примеры и
Дифференциальные уравнения решение задач  и примеры нужно присоединить к ответу, так как они являются интегральными кривыми дифференциального уравнения (1).

Пример:

Найти интегральные кривые дифференциального уравнения

Дифференциальные уравнения решение задач  и примеры

Решение:

1.Перепишем исходное уравнение в виде

Дифференциальные уравнения решение задач  и примеры

Поскольку Дифференциальные уравнения решение задач  и примеры разделяем переменные, т.е.
представляем уравнение (2) в виде

Дифференциальные уравнения решение задач  и примеры

2.Вычислим интегралы в уравнении

Дифференциальные уравнения решение задач  и примеры

Имеем

Дифференциальные уравнения решение задач  и примеры

Дифференциальные уравнения решение задач  и примеры

Следовательно,

Дифференциальные уравнения решение задач  и примеры

где Дифференциальные уравнения решение задач  и примеры

3.Упростив это равенство, получим

Дифференциальные уравнения решение задач  и примеры

Ответ. Интегральные кривые определяются уравнением

Дифференциальные уравнения решение задач  и примеры

при всевозможных значениях С.

Однородные уравнения

Постановка задачи. Найти интегральные кривые однородного
дифференциального уравнения первого порядка, т.е. дифференциального уравнения вида

P(x,y)dx + Q(x,y)dy = O, (1)

где Р(х,у) и Q(x,y) — однородные функции одинакового порядка,
т.е.
Дифференциальные уравнения решение задач  и примеры

План решения.

1.Преобразуем уравнение (1) к виду

Дифференциальные уравнения решение задач  и примеры

2.Делаем подстановку у(х) = х z(x), где z(x) — новая неизвестная
функция. Тогда у’ = z + x z’ и уравнение (1′) приводится к виду

Дифференциальные уравнения решение задач  и примеры

т.е. к уравнению с разделяющимися переменными.

Заметим, что подстановку у(х) = xz(x) можно делать сразу в
уравнении (1), не приводя его к виду (1′).

3.Разделяем переменные в области, где Дифференциальные уравнения решение задач  и примеры

Дифференциальные уравнения решение задач  и примеры

4.Интегрируем полученное уравнение с разделенными переменными и делаем замену z(x) = y(x)/x. Записываем ответ.

Замечания:

1.Если Дифференциальные уравнения решение задач  и примеры — корень уравнения f(z) — z = 0, то решением
уравнения (1) будет также Дифференциальные уравнения решение задач  и примеры

2.Интегральные кривые однородного уравнения можно искать и
в полярных координатах.

Пример:

Найти интегральные кривые дифференциального уравнения

Дифференциальные уравнения решение задач  и примеры

Решение:

1.Преобразуем заданное уравнение к виду

Дифференциальные уравнения решение задач  и примеры

(мы разделили числитель и знаменатель правой части заданного уравнения на Дифференциальные уравнения решение задач  и примеры

2.Делаем подстановку у = xz(x), где z(x) — новая неизвестная
функция. Тогда у’ = z + xz’ и уравнение приводится к виду

Дифференциальные уравнения решение задач  и примеры

т.е. к уравнению с разделяющимися переменными.

В результате простых преобразований получаем

Дифференциальные уравнения решение задач  и примеры

3.Разделяем переменные Дифференциальные уравнения решение задач  и примеры

Дифференциальные уравнения решение задач  и примеры

4.Интегрируя, получаем

Дифференциальные уравнения решение задач  и примеры

Заменяя z на у/х, получаем

Дифференциальные уравнения решение задач  и примеры

Ответ. Интегральные кривые определяются уравнением

Дифференциальные уравнения решение задач  и примеры

при всевозможных значениях С.

Линейные уравнения 1-го порядка

Постановка задачи. Решить задачу Коши для уравнения

Дифференциальные уравнения решение задач  и примеры

с начальным условием

Дифференциальные уравнения решение задач  и примеры

План решения.

1-й способ.

1.Запишем соответствующее однородное линейное уравнение:

Дифференциальные уравнения решение задач  и примеры

Это уравнение с разделяющимися переменными.

2.Разделяя переменные и интегрируя, получим общее решение
однородного уравнения (2)

Дифференциальные уравнения решение задач  и примеры

3.Применяем метод вариации произвольной постоянной:

а) ищем решение неоднородного уравнения (1) в виде (3), считая
С неизвестной функцией ж, т.е. полагая С = С(х);

б) подставляем в уравнение (1) у и у’, определяемые из соотношения (3), где С = С(х). Из полученного дифференциального уравнения
определяем функцию С(х).

4.Таким образом, общее решение неоднородного уравнения (1)
получаем в виде

Дифференциальные уравнения решение задач  и примеры

Здесь С(х) содержит произвольную постоянную Дифференциальные уравнения решение задач  и примеры.

5.Используя начальные условия (1′), находим значение Дифференциальные уравнения решение задач  и примеры и получаем решение поставленной задачи Коши.

Записываем ответ в виде Дифференциальные уравнения решение задач  и примеры

2-й способ.

1.Ищем решение уравнения (1) в виде

y = u(x)v(x), (4)

где u и v — неизвестные функции х.

2.Уравнение (1) принимает вид

u’v + uv’ + p(x)uv = q(x). (5)

3.Поскольку теперь мы имеем две неизвестные функции, а уравнение, которому они должны удовлетворять, только одно, то еще одно
уравнение мы можем принять произвольно, лишь бы оно не сужало
множество решений у.

Пусть одна из функций (например, u(х)) удовлетворяет уравнению

u’+р(х)u = 0. (6)

Тогда уравнение (5) примет вид

v’u = q(x). (7)

Решая уравнение (6) (с разделяющимися переменными), находим и, не равное тождественно нулю, чтобы не сужать множество решений у.

4.Подставляем u(х) в уравнение (7) и решаем его относительно v.

5.Записываем общее решение уравнения в виде у(х) = u(x)v(x).

6.Используя начальные условия (1′), получаем решение поставленной задачи Коши.

Записываем ответ в виде Дифференциальные уравнения решение задач  и примеры

Пример:

Найти решение задачи Коши для уравнения с начальным условием

Дифференциальные уравнения решение задач  и примеры

Решение:

1-й способ.

1.Записываем соответствующее однородное линейное уравнение:

Дифференциальные уравнения решение задач  и примеры

Это уравнение с разделяющимися переменными.

2.Разделяя переменные и интегрируя, получаем общее решение
однородного уравнения

у = Сх.

3.Применяем метод вариации произвольной постоянной:

а) ищем решение неоднородного уравнения (8) в виде

у = С(х)х,

где С(х) — неизвестная функция х;

б) подставляя в уравнение (8)
у = С(х)х и у’ = С'(х)х + С(х),

получаем дифференциальное уравнение относительно С(х)

Дифференциальные уравнения решение задач  и примеры

Это уравнение с разделяющимися переменными.

Разделяя переменные

Дифференциальные уравнения решение задач  и примеры

и интегрируя, получаем

Дифференциальные уравнения решение задач  и примеры

где Дифференциальные уравнения решение задач  и примеры — произвольная постоянная.

4.Таким образом, общее решение неоднородного уравнения (8)
имеет вид

Дифференциальные уравнения решение задач  и примеры

5.Используя начальное условие у(1) = 1, получаем

Дифференциальные уравнения решение задач  и примеры

находим Дифференциальные уравнения решение задач  и примеры и подставляем в общее решение (9).

Ответ. у = 1/х.

2-й способ.

1.Ищем решение уравнения (8) в виде

у = u(x)v(x),

где u и v — неизвестные функции х.

2.Уравнение (8) принимает вид

Дифференциальные уравнения решение задач  и примеры

3.Поскольку теперь мы имеем две неизвестные функции, а уравнение, которому они должны удовлетворять, только одно, то еще одно
уравнение мы можем принять произвольно, лишь бы оно не сужало
множество решений у.

Пусть одна из функций (например, u(х)) удовлетворяет уравнению

Дифференциальные уравнения решение задач  и примеры

Тогда уравнение (10) принимает вид

Дифференциальные уравнения решение задач  и примеры

Решая уравнение (11) (с разделяющимися переменными), находим

u(х) = Ах,

где А — произвольная постоянная (Дифференциальные уравнения решение задач  и примеры чтобы не сужать множество решений).

4.Подставляем u(х) в уравнение (12) и решаем его относительно v:

Дифференциальные уравнения решение задач  и примеры

где В — произвольная постоянная.

5.Записываем общее решение уравнения (8) в виде

Дифференциальные уравнения решение задач  и примеры

где С = АВ — произвольная постоянная.

6.Используя начальное условие у(1) = 1, находим C = 0.

Ответ. у = 1/х.

Уравнение Бернулли

Постановка задачи. Найти решение задачи Коши для уравнения Бернулли

Дифференциальные уравнения решение задач  и примеры

с начальным условием

Дифференциальные уравнения решение задач  и примеры

План решения.

1.С помощью подстановки

Дифференциальные уравнения решение задач  и примеры

уравнение приводится к линейному

Дифференциальные уравнения решение задач  и примеры

где Дифференциальные уравнения решение задач  и примеры и Дифференциальные уравнения решение задач  и примеры

2.Решаем линейное уравнение (2) и делаем замену Дифференциальные уравнения решение задач  и примеры

3.Используя начальное условие Дифференциальные уравнения решение задач  и примеры находим решение поставленной задачи Коши.

Записываем ответ в виде Дифференциальные уравнения решение задач  и примеры

Замечание:

При решения уравнения Бернулли можно не приводить его к линейному, а искать решение в виде у = u(x)v(x) или
применять метод вариации произвольной постоянной.

Пример:

Найти решение задачи Коши

Дифференциальные уравнения решение задач  и примеры

с начальным условием

у(1)=1.

Решение:

Преобразовав уравнение к виду

Дифференциальные уравнения решение задач  и примеры

убеждаемся, что это уравнение Бернулли с Дифференциальные уравнения решение задач  и примеры

1.С помощью подстановки

Дифференциальные уравнения решение задач  и примеры

уравнение (3) приводится к линейному

Дифференциальные уравнения решение задач  и примеры

2.Решаем уравнение (4) методом вариации произвольной постоянной:

а) решаем однородное уравнение получаем

Дифференциальные уравнения решение задач  и примеры

получаем z=Cx;

б) ищем решение неоднородного уравнения в виде

z = С(х)х;

в) подставляя в уравнение (4)

z = С(х)х, z’= С'(х)х + С(х),

получаем уравнение с разделяющимися переменными

С'(х)х = -1.

Разделяя переменные и интегрируя, получаем

Дифференциальные уравнения решение задач  и примеры

Таким образом, общее решение неоднородного уравнения (4) имеет
вид

Дифференциальные уравнения решение задач  и примеры

или, после замены Дифференциальные уравнения решение задач  и примеры

Дифференциальные уравнения решение задач  и примеры

3.Используя начальное условие y(1) = 1:

Дифференциальные уравнения решение задач  и примеры

получаем Дифференциальные уравнения решение задач  и примеры

Ответ. Дифференциальные уравнения решение задач  и примеры

Уравнения в полных дифференциалах

Постановка задачи. Найти интегральные кривые дифференциального уравнения

P(x,y)dx + Q(x,y)dy = 0. (1)

План решения.

1.Если в некоторой области D Р(х, у) и Q(x, у) имеют непрерывные частные производные и выполнено условие

Дифференциальные уравнения решение задач  и примеры

то P(x,y)dx + Q(x,y)dy — дифференциал некоторой функции
U(x,y). Тогда уравнение (1) называется уравнением в полных дифференциалах и может быть записано в виде

Дифференциальные уравнения решение задач  и примеры

где U(x, у) — дважды непрерывно дифференцируемая неизвестная
функция.

Из Дифференциальные уравнения решение задач  и примеры следует, что интегральные кривые определяются уравнением U(x,y) = С при всевозможных значениях С.

Для отыскания U заметим, что

Дифференциальные уравнения решение задач  и примеры

2.Интегрируя первое равенство в (2) по x, получим

Дифференциальные уравнения решение задач  и примеры

где Дифференциальные уравнения решение задач  и примеры — неизвестная функция, которую еще предстоит найти.

3.Дифференцируя U по у, имеем

Дифференциальные уравнения решение задач  и примеры

Используя второе равенство в (2), получим уравнение

Дифференциальные уравнения решение задач  и примеры

4.Находим Дифференциальные уравнения решение задач  и примеры и затем U(x,y).

Ответ. Интегральные кривые определяются уравнением U(x,y) = C
при всевозможных значениях С.

Пример:

Найти интегральные кривые дифференциального урав-
уравнения

Дифференциальные уравнения решение задач  и примеры

Решение:

1.Преобразуем уравнение (3):

Дифференциальные уравнения решение задач  и примеры

В данном случае

Дифференциальные уравнения решение задач  и примеры

Эти функции непрерывно дифференцируемы в области Дифференциальные уравнения решение задач  и примеры Кроме того,

Дифференциальные уравнения решение задач  и примеры

Поэтому Р(х, у) dx + Q(x, у) dy — дифференциал некоторой функции
U(x, у) в любой односвязной области D, не содержащей точку х = 0,
у = 0. Следовательно, уравнение (3) является уравнением в полных
дифференциалах и может быть записано в виде

dU(x,y) = 0 при Дифференциальные уравнения решение задач  и примеры

При этом

Дифференциальные уравнения решение задач  и примеры

2.Интегрируя первое равенство в (4) по x, получим, что при Дифференциальные уравнения решение задач  и примеры

Дифференциальные уравнения решение задач  и примеры

где Дифференциальные уравнения решение задач  и примеры — неизвестная функция, которую еще предстоит найти.

3.Дифференцируя U по y, имеем

Дифференциальные уравнения решение задач  и примеры

Используя второе равенство в (2), получим

Дифференциальные уравнения решение задач  и примеры

После преобразований имеем

Дифференциальные уравнения решение задач  и примеры

4.Отсюда

Дифференциальные уравнения решение задач  и примеры

и, следовательно,

Дифференциальные уравнения решение задач  и примеры

Ответ. Интегральные кривые в области у > 0 или в области у < 0
определяются уравнением

Дифференциальные уравнения решение задач  и примеры

при всевозможных значениях С.

Уравнения вида Дифференциальные уравнения решение задач  и примеры

Постановка задачи. Найти общее решение дифференциального
уравнения

Дифференциальные уравнения решение задач  и примеры

План решения.

1.Полагал Дифференциальные уравнения решение задач  и примеры получим дифференциальное уравнение
первого порядка

Дифференциальные уравнения решение задач  и примеры

2.Определяя тип этого уравнения и применяя соответствующий
метод решения, находим Дифференциальные уравнения решение задач  и примеры где Дифференциальные уравнения решение задач  и примеры — произвольная постоянная.

3.Так как Дифференциальные уравнения решение задач  и примеры имеем

Дифференциальные уравнения решение задач  и примеры

Последовательно интегрируя k раз (при каждом интегрировании не
забывая о произвольной постоянной), получим ответ

Дифференциальные уравнения решение задач  и примеры

Дифференциальные уравнения решение задач  и примеры — произвольные постоянные.

Пример:

Найти общее решение дифференциального уравнения

Дифференциальные уравнения решение задач  и примеры

Решение:

1.Поскольку дифференциальное уравнение не содержит у, то полагая у’ = р(х), имеем у» = р'(х). Получаем дифференциальное уравнение первого порядка

Дифференциальные уравнения решение задач  и примеры

2.Уравнение

Дифференциальные уравнения решение задач  и примеры

линейное относительно р и p’. Решая его, например, методом вариации произвольной постоянной, находим

Дифференциальные уравнения решение задач  и примеры

3.Так как р = у'(х), имеем

Дифференциальные уравнения решение задач  и примеры

Интегрируя, получим общее решение Дифференциальные уравнения решение задач  и примеры

Ответ. Дифференциальные уравнения решение задач  и примеры

Уравнения вида F(y,y’,y»)=0

Постановка задачи. Найти решение задачи Коши для дифференциального уравнения

F(y,y’,y»)=0

с начальными условиями

Дифференциальные уравнения решение задач  и примеры

План решения.

1.Поскольку дифференциальное уравнение не содержит явно независимой переменной х, полагаем

у=р(у),

где р(у) — новая неизвестная функция. Тогда по формуле для производной сложной функции имеем

Дифференциальные уравнения решение задач  и примеры

Получим уравнение первого порядка относительно р(у)

Дифференциальные уравнения решение задач  и примеры

2.Определяя тип этого уравнения и применяя соответствующий
метод решения, находим р = f(y,C), где С — произвольная постоянная.

3.Используя начальные условия (оба), находим Дифференциальные уравнения решение задач  и примеры

4.Подставляя Дифференциальные уравнения решение задач  и примеры получаем дифференциальное уравнение с разделяющимися переменными

Дифференциальные уравнения решение задач  и примеры

Разделяя переменные в области, где Дифференциальные уравнения решение задач  и примеры получаем

Дифференциальные уравнения решение задач  и примеры

и, интегрируя, находим Дифференциальные уравнения решение задач  и примеры

Проверяем, не является ли решение Дифференциальные уравнения решение задач  и примеры особым решением
исходного уравнения, удовлетворяющим начальным условиям.

5.Используем начальные условия для нахождения второй постояннойДифференциальные уравнения решение задач  и примеры (значение Дифференциальные уравнения решение задач  и примеры уже было найдено в п. 3) и получаем решение
задачи Коши.

Ответ записываем в виде у = у(х) или х = х(у).

Пример:

Найти решение задачи Коши для дифференциального
уравнения

Дифференциальные уравнения решение задач  и примеры

с начальными условиями

у(1) = -1, у(1) = -1

Решение:

1.Поскольку дифференциальное уравнение не содержит явно независимой переменной х, полагаем

у’ =р(у),

где р(у) — новая неизвестная функция. Тогда по формуле для производной сложной функции имеем

Дифференциальные уравнения решение задач  и примеры

Получим уравнение первого порядка относительно

Дифференциальные уравнения решение задач  и примеры

2.Разделяя переменные и интегрируя, находим

Дифференциальные уравнения решение задач  и примеры

т.е.

Дифференциальные уравнения решение задач  и примеры

(знак минус мы выбрали из начального условия у'(1) = —1 < 0.)

3.Из начальных условий (обоих) имеем у’ = — 1 при у = — 1.
Отсюда, Дифференциальные уравнения решение задач  и примеры Учитывая, что в силу первого начального условия
у < 0 и, следовательно, Дифференциальные уравнения решение задач  и примеры получаем

Дифференциальные уравнения решение задач  и примеры

4.Разделяя переменные и интегрируя, находим

Дифференциальные уравнения решение задач  и примеры

5.Из начального условия у(1) = —1 получим Дифференциальные уравнения решение задач  и примеры
Следовательно,

Дифференциальные уравнения решение задач  и примеры

(Знак минус мы выбрали из начального условия у(1) = — 1 < 0.)

Ответ. Дифференциальные уравнения решение задач  и примеры

Линейные уравнения с постоянными коэффициентами

Постановка задачи. Найти общее решение линейного дифференциального уравнения второго порядка с постоянными коэффициентами

Дифференциальные уравнения решение задач  и примеры

где Дифференциальные уравнения решение задач  и примеры — многочлен степени п, Дифференциальные уравнения решение задач  и примеры— многочлен степени т
и p,q,a,b — действительные числа.

План решения.

Общее решение неоднородного линейного уравнения n-го порядка
имеет следующую структуру:

Дифференциальные уравнения решение задач  и примеры

где Дифференциальные уравнения решение задач  и примеры — фундаментальная система решений и Дифференциальные уравнения решение задач  и примеры — общее решение соответствующего однородного уравнения, Дифференциальные уравнения решение задач  и примеры — какое-нибудь частное решение неоднородного уравнения.

1.Записываем соответствующее однородное уравнение

Дифференциальные уравнения решение задач  и примеры

и ищем его решение в виде Дифференциальные уравнения решение задач  и примеры где Дифференциальные уравнения решение задач  и примеры — неизвестное число.

Подставляя Дифференциальные уравнения решение задач  и примеры в уравнение (2) и
сокращая Дифференциальные уравнения решение задач  и примеры получаем так называемое характеристическое уравнение

Дифференциальные уравнения решение задач  и примеры

2.Решаем характеристическое уравнение. Обозначим корни характеристического уравнения Дифференциальные уравнения решение задач  и примеры и Дифференциальные уравнения решение задач  и примеры. Тогда фундаментальная система решений и общее решение уравнения (2) записываются в одном из следующих трех видов:

а) если Дифференциальные уравнения решение задач  и примеры и Дифференциальные уравнения решение задач  и примеры вещественны и Дифференциальные уравнения решение задач  и примеры то фундаментальная система решений — это Дифференциальные уравнения решение задач  и примеры и общее решение имеет
вид

Дифференциальные уравнения решение задач  и примеры

б) если Дифференциальные уравнения решение задач  и примеры и Дифференциальные уравнения решение задач  и примеры вещественны и Дифференциальные уравнения решение задач  и примеры то фундаментальная система решений — это Дифференциальные уравнения решение задач  и примеры и общее решение имеет
вид

Дифференциальные уравнения решение задач  и примеры

в) если Дифференциальные уравнения решение задач  и примеры и Дифференциальные уравнения решение задач  и примеры комплексные, т.е. Дифференциальные уравнения решение задач  и примеры то фундаментальная система решений — это Дифференциальные уравнения решение задач  и примеры и общее решение имеет вид

Дифференциальные уравнения решение задач  и примеры

3.Ищем какое-либо частное решение неоднородного уравнения.
Поскольку правая часть уравнения имеет вид

Дифференциальные уравнения решение задач  и примеры

можно применить метод подбора частных решений:

если а ± ib не является корнем характеристического уравнения (3), то

Дифференциальные уравнения решение задач  и примеры

где Дифференциальные уравнения решение задач  и примеры и Дифференциальные уравнения решение задач  и примеры — многочлены степени k = max{n,m} с неопределенными коэффициентами;

если а ± ib есть корень характеристического уравнения (3) кратности s, то

Дифференциальные уравнения решение задач  и примеры

где Дифференциальные уравнения решение задач  и примеры и Дифференциальные уравнения решение задач  и примеры — многочлены степени k = max{n,m} с неопределенными коэффициентами.

4.Находим неопределенные коэффициенты, подставляя Дифференциальные уравнения решение задач  и примеры в исходное уравнение.

Записываем ответ по формуле (1).

Замечание:

Аналогично решаются линейные дифференциальные
уравнения с постоянными коэффициентами любого порядка.

Пример:

Найти общее решение линейного дифференциального
уравнения

у» + у = х sin х. (5)

Решение:

1.Записываем соответствующее однородное уравнение

у» + у = 0 (6)

и ищем его решение в виде Дифференциальные уравнения решение задач  и примеры где Дифференциальные уравнения решение задач  и примеры — неизвестное число.
Подставляя Дифференциальные уравнения решение задач  и примеры в уравнение (6) и
сокращая Дифференциальные уравнения решение задач  и примеры получаем характеристическое уравнение

Дифференциальные уравнения решение задач  и примеры

2.Характеристическое уравнение имеет два комплексно сопряженных корня Дифференциальные уравнения решение задач  и примеры

Имеем фундаментальную систему решений

Дифференциальные уравнения решение задач  и примеры

и общее решение однородного уравнения (6)

Дифференциальные уравнения решение задач  и примеры

3.Ищем какое-либо частное решение неоднородного уравнения
(5). В нашем случае правая часть неоднородного уравнения имеет
вид (4) с а = 0, b = 1, n = 0, m = 1.

Так как характеристическое уравнение имеет комплексные корни
а ± ib = ±i кратности s = 1 и max {n, m} = 1, то частное решение
ищем в виде

Дифференциальные уравнения решение задач  и примеры

где Дифференциальные уравнения решение задач  и примеры — неизвестные числа (неопределенные коэффициенты) .

4.Находим неопределенные коэффициенты, дифференцируя Дифференциальные уравнения решение задач  и примеры
два раза и подставляя в уравнение (5).

Приравнивая коэффициенты в обеих частях равенства при cos x,
х cos x, sin x, х sin x, получаем четыре уравнения

Дифференциальные уравнения решение задач  и примеры

из которых определяем Дифференциальные уравнения решение задач  и примеры Таким
образом,

Дифференциальные уравнения решение задач  и примеры

По формуле (1) находим общее решение неоднородного уравнения

Дифференциальные уравнения решение задач  и примеры

Ответ.Дифференциальные уравнения решение задач  и примеры

Принцип суперпозиции

Постановка задачи. Найти общее решение линейного дифференциального уравнения п — го порядка с постоянными коэффициентами

Дифференциальные уравнения решение задач  и примеры

где Дифференциальные уравнения решение задач  и примеры

План решения.

Принцип суперпозиции. Если правая часть уравнения (1)
есть сумма нескольких функций

Дифференциальные уравнения решение задач  и примеры

и Дифференциальные уравнения решение задач  и примеры — какое-нибудь частное решение каждого уравнения

Дифференциальные уравнения решение задач  и примеры

то в силу линейности уравнения (1) его общее решение имеет вид

Дифференциальные уравнения решение задач  и примеры

где Дифференциальные уравнения решение задач  и примеры — общее решение однородного уравнения

Дифференциальные уравнения решение задач  и примеры

1.Находим фундаментальную систему решений и общее решение
Дифференциальные уравнения решение задач  и примеры однородного уравнения.

2.Для каждого неоднородного уравнения (2) (i = 1, 2,…, k) находим частное решение Дифференциальные уравнения решение задач  и примеры (используя, например, метод подбора или
метод вариации произвольных постоянных).

Записываем ответ в виде (3).

Пример:

Найти общее решение линейного дифференциального
уравнения

Дифференциальные уравнения решение задач  и примеры

Решение:

1.Записываем соответствующее однородное уравнение

Дифференциальные уравнения решение задач  и примеры

и ищем его решение в виде Дифференциальные уравнения решение задач  и примеры где Дифференциальные уравнения решение задач  и примеры — неизвестное число.
Подставляя Дифференциальные уравнения решение задач  и примеры в уравнение (4) и
сокращая Дифференциальные уравнения решение задач  и примеры получаем характеристическое уравнение

Дифференциальные уравнения решение задач  и примеры

Характеристическое уравнение имеет три корня Дифференциальные уравнения решение задач  и примеры и
Дифференциальные уравнения решение задач  и примеры

Таким образом, имеем фундаментальную систему решений Дифференциальные уравнения решение задач  и примеры
и общее решение однородного уравнения

Дифференциальные уравнения решение задач  и примеры

2.Решаем неоднородное уравнение, используя принцип суперпозиции:

а) ищем частное решение Дифференциальные уравнения решение задач  и примеры неоднородного уравнения

Дифференциальные уравнения решение задач  и примеры

в виде Дифференциальные уравнения решение задач  и примеры где А — неопределенный коэффициент (так
как а + ib = 10 — корень характеристического уравнения кратности
s = 1).

Дифференцируя Дифференциальные уравнения решение задач  и примеры три раза и подставляя в уравнение (5), находим А = 1/10. Таким образом,

Дифференциальные уравнения решение задач  и примеры

б) ищем частное решение Дифференциальные уравнения решение задач  и примеры неоднородного уравнения

у'» -100у’ = 100cos10x (6)

в виде Дифференциальные уравнения решение задач  и примеры где Дифференциальные уравнения решение задач  и примеры и Дифференциальные уравнения решение задач  и примеры — неопределенные
коэффициенты (так как а±ib = ±10i не являются корнями характеристического уравнения, то множитель х отсутствует).

Дифференцируя Дифференциальные уравнения решение задач  и примеры три раза и подставляя в уравнение (6), находим Дифференциальные уравнения решение задач  и примеры и Дифференциальные уравнения решение задач  и примеры Таким образом,

Дифференциальные уравнения решение задач  и примеры

Используя принцип суперпозиции (3), получаем

Дифференциальные уравнения решение задач  и примеры

Ответ. Дифференциальные уравнения решение задач  и примеры

Метод Лагранжа

Постановка задачи. Найти решение задачи Коши для линейного неоднородного уравнения с постоянными коэффициентами

Дифференциальные уравнения решение задач  и примеры

с начальными условиями

Дифференциальные уравнения решение задач  и примеры

План решения.

1.Записываем соответствующее однородное уравнение с постоянными коэффициентами

Дифференциальные уравнения решение задач  и примеры

Находим фундаментальную систему решений Дифференциальные уравнения решение задач  и примеры и Дифференциальные уравнения решение задач  и примеры и общее
решение однородного уравнения

Дифференциальные уравнения решение задач  и примеры

2.Применяем метод Лагранжа (метод вариации произвольных
постоянных).

Если известна фундаментальная система решений Дифференциальные уравнения решение задач  и примеры
однородного уравнения (2), то общее решение соответствующего неоднородного уравнения (1) может быть найдено по формуле

Дифференциальные уравнения решение задач  и примеры

где функции Дифференциальные уравнения решение задач  и примеры и Дифференциальные уравнения решение задач  и примеры определяются из системы линейных алгебраических уравнений

Дифференциальные уравнения решение задач  и примеры

3.Интегрируя, находим функции Дифференциальные уравнения решение задач  и примеры и Дифференциальные уравнения решение задач  и примеры и записываем общее решение неоднородного уравнения.

3.Используя начальные условия Дифференциальные уравнения решение задач  и примеры находим решение задачи
Коши.

Записываем ответ в виде у = у(х).

Пример:

Найти решение задачи Коши

Дифференциальные уравнения решение задач  и примеры

с начальными условиями Дифференциальные уравнения решение задач  и примеры

Решение:

1.Записываем соответствующее однородное уравнение:

у» + у = 0.

Находим фундаментальную систему решений Дифференциальные уравнения решение задач  и примеры и Дифференциальные уравнения решение задач  и примеры и общее решение однородного уравнения

Дифференциальные уравнения решение задач  и примеры

2.Применяем метод Лагранжа (метод вариации произвольных
постоянных):

а) ищем решение данного неоднородного уравнения в виде

Дифференциальные уравнения решение задач  и примеры

б) записываем систему уравнений для определения функций Дифференциальные уравнения решение задач  и примеры
и Дифференциальные уравнения решение задач  и примеры

Дифференциальные уравнения решение задач  и примеры

Решая ее (так как решение ищем в окрестности точки х = 0, то
cos х > 0), получим

Дифференциальные уравнения решение задач  и примеры

Интегрируя, находим

Дифференциальные уравнения решение задач  и примеры

в) записываем полученное общее решение данного неоднородного
уравнения

Дифференциальные уравнения решение задач  и примеры

3.Используя начальные условия, определяем константы Дифференциальные уравнения решение задач  и примеры и Дифференциальные уравнения решение задач  и примеры
Так как

Дифференциальные уравнения решение задач  и примеры

то Дифференциальные уравнения решение задач  и примеры Так как

Дифференциальные уравнения решение задач  и примеры

то Дифференциальные уравнения решение задач  и примеры

Ответ. у = cosx(ln cos x + 1) + х sin х.

Как решать дифференциальные уравнения — подробная инструкция с примерами

Дифференциальные уравнения первого порядка

1°. Дифференциальное уравнение (ДУ) первого порядка имеет общий вид

Дифференциальные уравнения

а разрешенное относительно первой производной:

Дифференциальные уравнения

Если функцию f (x, у) можно представить в виде отношения двух функций Дифференциальные уравнения то дифференциальное уравнение представимо в дифференциальной форме

Дифференциальные уравнения

Решением дифференциального уравнения называется каждая функция у = у (х), которая после подстановки в уравнение вместе с у'(х) превращает его в тождество.

Теорема:

Существования и единственности решения. Если в уравнении у’ = f(x,y) функция f(x,y) и ее частная производная f(x, y) непрерывны в некоторой области D плоскости Оху, Дифференциальные уравнения— точка этой области, то существует, и притом единственное, решение у = у (х), удовлетворяющее условию Дифференциальные уравнения Дифференциальные уравнения

Условие Дифференциальные уравнениязаписываемое также в виде Дифференциальные уравненияили Дифференциальные уравнения при Дифференциальные уравненияназывается начальным условием, или условием Коши, для данного дифференциального уравнения. Геометрически начальное условие изображает точку в области D, через которую должна проходить интегральная кривая (решение уравнения) у = у(х).

2°. Общим решением дифференциального уравнения первого порядка называется функция

Дифференциальные уравнения

содержащая произвольную постоянную С и удовлетворяющая условиям

1) Дифференциальные уравнения удовлетворяет ДУ при любом значении С

2) каково бы ни было начальное условие Дифференциальные уравненияможно найти значение Дифференциальные уравнения такое, что Дифференциальные уравнения удовлетворяет этому условию, т е

Дифференциальные уравнения

3°. Частным решением дифференциального уравнения называется любая функция Дифференциальные уравнениясоответствующая фиксированному значению постоянной С.

Решить, или проинтегрировать, данное дифференциальное уравнение означает найти общее решение или, иногда говорят, общий интеграл и если начальные условия заданы, то найти также то частное решение, которое удовлетворяет начальным условиям.

Под общим интегралом дифференциального уравнения понимается соотношение вида

Ф (x, y, C) = 0,

получаемое при решении уравнения. Оно определяет решение
у = у(х, С) как неявную функцию двух переменных.

4°. Геометрически общее решение (общий интеграл) изображается семейством (множеством) кривых в области D, зависящих от постоянной С.

Каждая кривая этого семейства соответствует частному решению уравнения, а выделить какое-либо частное решение из общего можно заданием точки, через которую оно проходит. Именно это выражает условие Коши.

5°. Дифференциальное уравнение y’ = f(x,y), разрешенное относительно производной, определяется функцией f(x,y). В каждой точке Дифференциальные уравнениязначение Дифференциальные уравненияесть Дифференциальные уравненияа это — угловой коэффициент (направление) касательной к кривой у = у(х) в точке с абсциссой Дифференциальные уравнения.

Тем самым уравнение у’ = f (х, у) геометрически изображается множеством или, как говорят, полем направлений в области D. Другими словами, каждая точка области снабжена направлением, вдоль которого должна через эту точку пройти интегральная кривая.

Геометрическое место точек, в которых у’ = К, где К — постоянная, или, что то же самое, f(x,y) = К (линия уровня функции f(х,у)), называется изоклиной данного уравнения. В точках изоклины направления поля параллельны между собой.

Метод изоклин представляет собой метод приближенного построения интегральной кривой, исходя из построенного поля направлений данного дифференциального уравнения.

Примеры с решениями

Пример:

Дано дифференциальное уравнение у’ = х. Построить поле направлений и указать приближенно общее решение этого уравнения.

Решение:

Имеем f(х,у) = х. Эта функция определена при всех значениях х и у (она не зависит от у). Это означает, что Дифференциальные уравнения т.е. областью D является вся плоскость Оху. Во всех точках плоскости, в которых х = 0 (это ось Оу), имеем у’ = 0. Это значит, что в точках оси Оу поле горизонтально (на рис. 4.1 это изображено короткими горизонтальными штрихами). В точках плоскости, в которых х = 1 (это прямая х = 1), имеем у’ = 1, т.е. касательные ко всем кривым, пересекающим прямую х = 1, составляют с осью Ох угол в 45°. В точках прямой х = — 1 поле (множество касательных) составляет с положительным направлением оси Ох угол 135°, и т.д. Прямые х = К являются изоклинами поля направлений уравнения, и эти направления образуют с положительным направлением оси Ох углы, тангенс которых равен К. Тем самым поле направлений построено. Если через какую либо точку, скажем, через начало координат (0, 0), провести интегральную кривую, то в точке (0,0) эта кривая имеет горизонтальную касательную, а в окрестности точки х = 0, справа от нее, угловой коэффициент интегральной кривой положителен; прямую х = 1 она пересекает под углом в 45°, прямую Дифференциальные уравненияпод углом 60°, прямую х = 2 под большим углом, и т. д. Если эту интегральную кривую продолжать влево, то угловые коэффициенты касательных

Дифференциальные уравнения

меняются так, что прямую х = — 1 кривая пересекает под углом в 135°, прямую Дифференциальные уравнения под углом 120° и т. п. Поскольку поле симметрично относительно оси Оу, то такими же будут и интегральные кривые. Аналогичным образом можно строить интегральную кривую, проходящую через другую точку плоскости, например, через точку (0, 1). Поле указывает на «параллельность» семейства интегральных кривых. Густая сеть изоклин и тщательное построение интегральных кривых наводят на мысль о том, что интегральные кривые напоминают параболы.

И это оправдано, так как общее решение исходного уравнения имеет вид Дифференциальные уравнения

Пример:

Дано дифференциальное уравнение

Дифференциальные уравнения

Проверить, является ли функция Дифференциальные уравнения решением данного уравнения. Будет ли она его общим решением?

Решение:

Находим

Дифференциальные уравнения

Выражения для у и у’ подставим в левую часть исходного уравнения. После преобразований получим:

Дифференциальные уравнения

Получили тождество при Дифференциальные уравнения т.е. данная функция является решением уравнения; оно не является общим решением, поскольку она не содержит произвольную постоянную.

Пример:

Проверить, что функция, заданная параметрически:

Дифференциальные уравнения

является решением дифференциального уравнения х = ln у’ + sin у’.

Решение:

Находим у’ по правилу дифференцирования параметрически заданной функции:

Дифференциальные уравнения

Дифференцируем:

Дифференциальные уравнения

Подставляем соответствующие выражения для у и у’ в исходное уравнение: ln t + sin t = ln t + sin t.

Получили тождество при t > 0. Данная функция есть решение исходного уравнения. Заметим, что она не является общим решением (не содержит постоянную).

Пример:

Проверить, что функция Дифференциальные уравнениязависящая от произвольной постоянной С, является общим решением дифференциального уравнения Дифференциальные уравнения

Решение:

Имеем

Дифференциальные уравнения

После подстановки в уравнение получаем тождество (проверить самостоятельно) при всех Дифференциальные уравнения

При любых значениях С данная функция есть решение исходного уравнения.

Примечание:

Пусть Дифференциальные уравнения и Дифференциальные уравнения— произвольные числа. Найдем значение Дифференциальные уравнения, такое, что Дифференциальные уравнения Для этого решим относительно Дифференциальные уравнения уравнение Дифференциальные уравнения При Дифференциальные уравнениянаходим Дифференциальные уравнения

Подставим значение Дифференциальные уравнения в выражение для

Дифференциальные уравнения

Это — частное решение, полученное из общего и удовлетворяющее условию Коши: Дифференциальные уравнения при Дифференциальные уравнения

Таким образом, данная функция представляет собой общее решение исходного уравнения.

Пример:

Построить (в приближенном виде) поле направлений и интегральные кривые для дифференциального уравнения Дифференциальные уравнения

Решение:

Правая часть уравнения определена во всей плоскости Оху, т.е. область D, о которой речь идет в теореме существования и единственности, есть вся плоскость Оху. В начале координат имеем
х = у = 0 и у’ = 0. Ни в каких других точках не выполняется у’ = 0, т.е. начало координат — это единственная точка с горизонтальным направлением поля. В точке (1, 0) имеем у’ = 1, что соответствует направлению касательной под углом в 45°. Такое же направление касательных будет во всех точках окружности Дифференциальные уравнения (в них Дифференциальные уравнения).

Окружность Дифференциальные уравнения является изоклиной поля с направлением в 45°. В точках окружности Дифференциальные уравненияДифференциальные уравнения поле направлено под углом 60°. В точках окружности Дифференциальные уравненияДифференциальные уравненияполе направлено под углом v3 в 30°, и т.п. Чем меньше радиус окружности, тем меньше угол направления поля, и чем больше радиус окружности, тем круче наклон поля.

Пройдя через все точки поля, получим (условно говоря) все интегральные кривые. Общее представление об интегральных кривых можно получить из построенного чертежа (рис. 4.2). Одна кривая проходит через точку (0, 0) и имеет в этой точке горизонтальную касательную. Других кривых с горизонтальной касательной нет.

Пример:

Построить дифференциальное уравнение, если известно его общее решение Дифференциальные уравнения

Решение:

Воспользуемся определением: нужно составить уравнение, содержащее аргумент х, неизвестную функцию у и ее производную y’. Для этого данную функцию продифференцируем, а затем из двух уравнений, объединенных в систему, исключим постоянную С. Итак,

Дифференциальные уравнения

или

Дифференциальные уравнения

Таким образом, в качестве искомого дифференциального уравнения можно взять одно из последних равенств.

Уравнения с разделенными и с разделяющимися переменными

1°. Уравнением с разделенными переменными называется дифференциальное уравнение вида

Дифференциальные уравнения

Считая, что искомая функция у = у(х), уравнение можно записать в виде равенства двух дифференциалов

Дифференциальные уравнения

Это равносильно тому, что первообразные Ф(х) и G(y(x)) левой и правой частей отличаются на произвольную постоянную

Дифференциальные уравнения

Это равенство является общим интегралом, или общим решением данного уравнения.

2°. Уравнение вида

Дифференциальные уравнения

называется уравнением с разделяющимися переменными. Оно приводится к уравнению с разделенными переменными после почленного деления на Дифференциальные уравнения

3°. Уравнение вида у’ = f (ах + by) можно привести к уравнению с разделенными переменными при помощи подстановки ах + by = v, где v — новая неизвестная функция.

Примеры с решениями

Пример:

Решить дифференциальное уравнение Дифференциальные уравненияДифференциальные уравнения Найти также частное решение, удовлетворяющее начальному условию у = 0 при х = 0.

Решение:

Имеем уравнение с разделенными переменными: Дифференциальные уравненияКаждую часть интегрируем по своей переменной: Дифференциальные уравнения т.е. Дифференциальные уравнения

Получили общий интеграл, или общее решение в неявном виде. Отсюда получаем решение в явном виде: Дифференциальные уравнения (мы не учитываем периодичность тангенса). Подставляем сюда (или в общий интеграл) х = 0, у = 0 и определяем значение постоянной С: 0 = arctg С (или 0 = 0 + С ). Получаем С = 0 и подставляем это значение в общее решение. Искомое частное решение имеет вид Дифференциальные уравнения

О т в е т Дифференциальные уравнения, Дифференциальные уравнения

Пример:

Решить уравнение

Дифференциальные уравнения

Решение:

Вынесением за скобки приводим уравнение к уравнению с разделяющимися переменными:

Дифференциальные уравнения

Почленно разделим на Дифференциальные уравнения

Получаем уравнение с разделенными переменными:

Дифференциальные уравнения

Проинтегрируем (второе слагаемое по частям):

Дифференциальные уравнения

Получили общий интеграл, или общее решение в неявном виде.

О т в е т

Дифференциальные уравнения

Пример:

Решить уравнение Дифференциальные уравнения

Решение:

Очевидно, что правая часть данного уравнения является функцией от 2х + 3у. Обозначим v = 2х + 3у. Это равенство дифференцируем по переменной х (х — аргумент и для функции у и для функции v): 2 + 3у’ = v’.

Отсюда находим Дифференциальные уравнения и подставляем это выражение в исходное уравнение:

Дифференциальные уравнения

Интегрируем

Дифференциальные уравнения
Дифференциальные уравнения

Получили общее решение вспомогательного уравнения. Возвращаясь к х и у, записываем общее решение исходного уравнения в неявном виде.

О т в е т

Дифференциальные уравнения

Пример:

Решить уравнение

Дифференциальные уравнения

Решение:

Переписав данное уравнение в виде Дифференциальные уравненияДифференциальные уравнениязамечаем целесообразность подстановки (замены переменной) 2х + у = v. Дифференцируя это равенство (по переменной х), получаем 2 + у’ = v’, или у’ = v’ — 2. Исходное уравнение принимает вид (у = v — 2х): Дифференциальные уравнения или Дифференциальные уравнения

Это уравнение с разделяющимися переменными относительно неизвестной функции v: Дифференциальные уравненияИнтегрируем:

Дифференциальные уравнения

— это общее решение вспомогательного уравнения. Возвращаемся к исходной неизвестной: v = 2х + у.

О т в е т у = 2tg(2х + С) — 2х.

Пример:

Проинтегрировать дифференциальное уравнение

Дифференциальные уравнения

и найти также то частное решение, которое удовлетворяет условию Коши: Дифференциальные уравнения

Решение:

Данное уравнение преобразуем с целью разделения переменных (заменив Дифференциальные уравнения ) :

Дифференциальные уравнения

Интегрируем полученное равенство:

Дифференциальные уравнения
Дифференциальные уравнения

Получили общее решение в неявном виде. Для получения частного решения подставим в общее решение значения х = 2, у = 1 и найдем постоянную

Дифференциальные уравнения
Дифференциальные уравнения

Подставив это значение С в формулу общего решения, получаем искомое частное решение в неявном виде:

Дифференциальные уравнения

Иногда привести уравнение к виду, удобному для интегрирования,можно нетривиальной подстановкой (заменой).

Пример:

Решить дифференциальное уравнение

Дифференциальные уравнения

Решение:

Очевидно, что разделить переменные нельзя. Выполним замену ху = z, т.е. Дифференциальные уравнениягде z— новая неизвестная функция аргумента х. Дифференцируя первое равенство, получаем у + ху’ — z’. Отсюда ху’ = z’ — у. Подстановки в исходное уравнение дают:

Дифференциальные уравнения

Заменим Дифференциальные уравнения и разделим переменные: Дифференциальные уравнения

После интегрирования получаем

Дифференциальные уравнения

Остается вернуться к переменным х и у (z = xy):

Дифференциальные уравнения

О т в е т

Дифференциальные уравнения

Однородные уравнения первого порядка

1°. Однородным уравнением первого порядка называется дифференциальное уравнение вида

Дифференциальные уравнения

При помощи подстановки — Дифференциальные уравнения т.е. у = t х, у’ = t х + t однородное уравнение приводится к уравнению с разделяющимися переменными относительно новой функции t = t(x).

2°. Уравнение вида Дифференциальные уравнения где Дифференциальные уравненияприводится к однородному при помощи подстановок Дифференциальные уравненияДифференциальные уравнения где Дифференциальные уравнения и Дифференциальные уравнения— числа, удовлетворяющие системе уравнений

Дифференциальные уравнения

Имея в виду равенство

Дифференциальные уравнения

данное уравнение приводим к однородному относительно новой функции v = v (u)

Дифференциальные уравнения

Если приведенная линейная система неразрешима, т.е. ее определитель равен нулю, то данное уравнение приводится к уравнению с разделяющимися переменными подстановкой Дифференциальные уравнения.

Примеры с решениями

Пример:

Найти общий интеграл уравнения

Дифференциальные уравнения

Решение:

Формально не очевидно, что правая часть этого уравнения есть функция, зависящая от отношения Дифференциальные уравненияДля того, чтобы это увидеть, вынесем за скобки в числителе и знаменателе Дифференциальные уравненияи сократим на этот множитель. Получим

Дифференциальные уравнения

Получили однородное уравнение, вид которого обозначен в п. 1°. Положим у = tx, у’ = t’x + t (здесь t — новая искомая функция от х). Получаем уравнение Дифференциальные уравнения

Преобразуем, перенося t в первую часть

Дифференциальные уравнения

или

Дифференциальные уравнения

разделим переменные: Дифференциальные уравнения

Проинтегрируем: Дифференциальные уравненияДифференциальные уравненияДифференциальные уравнения или

Дифференциальные уравнения

Подставляя Дифференциальные уравненияполучим искомый общий интеграл.

Ответ.

Дифференциальные уравнения

Пример:

Решить уравнение Дифференциальные уравнения

Решение:

Это уравнение было бы однородным, если в числителе и знаменателе правой части отсутствовали бы свободные члены. Оно имеет вид, указанный в п. 2°, и может быть приведено к однородному после дополнительного поиска.

Из линейной системы Дифференциальные уравнения находим Дифференциальные уравнения

Эти числа используем в последующих линейных заменах аргумента х и функции у. Положим х = u — 1, у = v + 2, у’ = v’. Получаем (убедитесь в этом) уравнение Дифференциальные уравнения Это новое уравнение является однородным (в нем имеем новую неизвестную функцию v и новый аргумент u).

Подстановкой v = tu, v’ = t’u + t (здесь u — независимая переменная,
t = t (u) — неизвестная функция от u) приходим к уравнению с разделяющимися переменными. Имеем Дифференциальные уравнения или Дифференциальные уравнения. Заменяем Дифференциальные уравненияразделяем переменные и интегрируем: Дифференциальные уравненияДифференциальные уравненият.е.

Дифференциальные уравнения

Возвращаемся к х и Дифференциальные уравнения

Под знаком логарифма приведем к общему знаменателю, логарифм частного заменим разностью логарифмов, равные слагаемые в правой и левой частях сократим. Общее решение получаем в неявном виде.

Ответ

Дифференциальные уравнения

Пример:

Решить уравнение Дифференциальные уравнения

Решение:

Система Дифференциальные уравнения, неразрешима, поэтому уравнение к однородному не приводится. Заметим, что правая часть есть функция от х + у. Уравнение можно привести к уравнению с разделяющимися переменными. Положим x + y = z. Тогда
2х +2у + 5 = 2z + 5. После дифференцирования предыдущего равенства получаем: 1 + у’ = z’, у’ = z’ — 1. Приходим к уравнениюДифференциальные уравнения откуда Дифференциальные уравненияили

Дифференциальные уравнения

Интегрируем и возвращаемся к переменным х и у. Получаем

Дифференциальные уравнения

Ответ

Дифференциальные уравнения

Линейные дифференциальные уравнения первого порядка и уравнения Бернулли

1°. Линейным дифференциальным уравнением первого порядка называется уравнение, линейное относительно неизвестной функции и ее производной. Оно имеет вид

Дифференциальные уравнения

Уравнение вида

Дифференциальные уравнения

называется уравнением Бернулли с показателем Дифференциальные уравнения При Дифференциальные уравненияуравнение Бернулли становится линейным.

Линейное уравнение и уравнение Бернулли можно решить единым методом, а именно подстановкой

Дифференциальные уравнения

Здесь u и v — две неизвестные функции, относительно которых имеет место равенство

Дифференциальные уравнения

На новые неизвестные функции u и v накладывается дополнительное условие (помимо того, что они должны удовлетворять предыдущему уравнению):

Дифференциальные уравнения

Если имеет место это равенство, то автоматическим следствием уравнения является равенство:

Дифференциальные уравнения

Предыдущее уравнение — с разделяющимися переменными. Решив его и подставив полученное v в последнее уравнение, определим функцию u. Тем самым, поскольку у = uv, общее решение первоначального уравнения найдено (в явном виде).

Примечание:

Из сказанного следует, что подстановкой у = uv линейное уравнение или уравнение Бернулли сводится к равносильной системе уравнений:

Дифференциальные уравнения

2°. Линейное уравнение можно решить и другим методом.

1) Если Дифференциальные уравнениято уравнение Дифференциальные уравнения называется линейным однородным. Оно является уравнением с разделяющимися переменными и имеет общее решение

Дифференциальные уравнения

2) Общее решение неоднородного уравнения (т.е. при Дифференциальные уравнения) можно найти методом вариации произвольной постоянной, который заключается в том, что оно ищется в виде

Дифференциальные уравнения

Это выражение вместе с его производной подставляется в исходное уравнение, в результате чего получается новое дифференциальное уравнение относительно неизвестной функции С(х). Найдя эту функцию, подставим ее в последнее равенство.

Ниже мы ограничимся только первым способом решения линейных уравнений и уравнений Бернулли.

Примеры с решениями

Пример:

Решить уравнение

Дифференциальные уравнения

Решение:

Положим Дифференциальные уравненияу’ = u’v + uv’. Получаем Дифференциальные уравненияДифференциальные уравненияДифференциальные уравнения

1) Из уравнения uv’ + xuv = 0, т.е. Дифференциальные уравнения или Дифференциальные уравнения

находим Дифференциальные уравнения или Дифференциальные уравнения(постоянную интегрирования принимаем равной нулю).

2) Оставшееся уравнение Дифференциальные уравненияпосле замены Дифференциальные уравненияпринимает вид Дифференциальные уравненияили Дифференциальные уравненияОтсюда Дифференциальные уравненияили Дифференциальные уравнения

Из полученных решений u и v составим общее решение данного уравнения.

О т в е т Дифференциальные уравнения

Пример:

Решить уравнение Дифференциальные уравнения

Решение:

Данное уравнение не является линейным (или уравнением Бернулли), поэтому подстановка вида у = uv неприемлема. Необходимо вспомнить о взаимно обратных функциях и связи между их производными: Дифференциальные уравненияДифференциальные уравнениячтобы заменить данное уравнение уравнением х’ = x cos у + sin 2у, линейным относительно функции х = х (у) аргумента у.

Выполним замену Дифференциальные уравненияx’ = u’v + u v’

Подставляем в новое уравнение:

Дифференциальные уравнения

после чего можно воспользоваться схемой, примененной в предыдущем примере.

1) Приравниваем средние члены уравнения: Дифференциальные уравнения
v’=v cos y, ln |v| = sin y, Дифференциальные уравнения(постоянную интегрирования принимаем равной нулю).

2) Осталось равенство крайних слагаемых: u’v = sin 2у,

Дифференциальные уравнения
Дифференциальные уравнения

(интегрировали по частям). Умножив на v, получаем выражение для х = х (у).

Ответ

Дифференциальные уравнения

Пример:

Проинтегрировать дифференциальное уравнение

Дифференциальные уравнения

Решение:

Меняем ролями функцию у и аргумент х:

Дифференциальные уравнения

Получили линейное относительно неизвестной функции х = х (у) уравнение. Полагая Дифференциальные уравнения, х’ = u’v + uv‘, получим систему

Дифференциальные уравнения

Ответ Дифференциальные уравнения

Уравнения в полных дифференциалах

1°. Дифференциальное уравнение вида

Дифференциальные уравнения

называется уравнением в полных дифференциалах, если функции Р(х,у) и Q(x,y) удовлетворяют условию Дифференциальные уравнения

В таком случае левая часть уравнения представляет собой полный дифференциал некоторой функции двух переменных u (х, у), и если эту функцию найти, то общее решение уравнения имеет вид
u (х,у) = С.

2 . Уравнения, для которых условие Дифференциальные уравнения не выполняется

(они не являются уравнениями в полных дифференциалах), приводятся к уравнениям в полных дифференциалах почленным умножением на надлежащий множитель р, называемый интегрирующим множителем:

Дифференциальные уравнения

зависит только от x;

Дифференциальные уравнения

зависит только от y;

Примеры с решениями

Пример:

Решить уравнение (х + у) dx + (х + 2у) dy = 0.

Решение:

Здесь Р = х + у, Q = х + 2у, Дифференциальные уравненияИмеем уравнение в полных дифференциалах. Найдем функцию u = u (х, у) из условия du = (х + у) dx + (x + 2у) dy. Так как Дифференциальные уравнения то Дифференциальные уравненияДифференциальные уравнения

Из первого условия находим

Дифференциальные уравнения

где Дифференциальные уравнения— некоторая функция от у, определяемая из второго условия:

Дифференциальные уравнения

Отсюда следует, что Дифференциальные уравнения т.е. Дифференциальные уравнения а тогда u (х,у) =Дифференциальные уравнения

Ответ. Дифференциальные уравнения

Пример:

Решить уравнение

Дифференциальные уравнения

Решение:

Имеем Дифференциальные уравненияQ = — 2ху, Дифференциальные уравненияДифференциальные уравнения

Уравнение не является уравнением в полных дифференциалах. Выражение Дифференциальные уравнения— зависит только от переменной х, поэтому согласно п. 2° (случай 1) уравнение допускает интегрирующий множитель Дифференциальные уравнения зависящий от х:

Дифференциальные уравнения

Умножим почленно исходное уравнение на множитель Дифференциальные уравненияПолучаем новое уравнение

Дифференциальные уравнения

Теперь имеем

Дифференциальные уравнения

т.е. уравнение в полных дифференциалах.

Находим функцию u (х,у), такую, что Дифференциальные уравнения Дифференциальные уравненияДифференциальные уравнения

Дифференцируем эту функцию по у и приравниваем к Дифференциальные уравнения

Дифференциальные уравнения

Отсюда Дифференциальные уравненият.е. Дифференциальные уравненияне зависит от у.

Можно принять Дифференциальные уравнения Тогда Дифференциальные уравнения

О т в е т. Дифференциальные уравнения

Пример:

Решить уравнение

Дифференциальные уравнения

Решение:

Имеем

Дифференциальные уравнения

Находим интегрирующий множитель:

Дифференциальные уравнения

Отсюда Дифференциальные уравненияПосле почленного умножения уравнения на Дифференциальные уравнения получаем уравнение в полных дифференциалах

Дифференциальные уравнения

Его общее решение имеет вид

Дифференциальные уравнения

Ответ

Дифференциальные уравнения

Примечание:

Некоторые уравнения допускают интегрирующие Множители, зависящие от обеих переменных х и у. Для получения таких множителей необходимо доумножать уравнение почленно на Дифференциальные уравненияи анализировать возможности, при которых новое уравнение может быть уравнением в полных дифференциалах. Такие уравнения мы не рассматриваем.

Дифференциальные уравнения первого порядка, не разрешенные относительно производной

Уравнения, не разрешенные относительно производной:

Дано дифференциальное уравнение первого порядка степени n, не разрешенное относительно первой производной у, вида

Дифференциальные уравнения

— известные непрерывные функции своих аргументов.

Предположим, что данное уравнение разрешимо относительно у’ и имеет Дифференциальные уравнениядействительных решений

Дифференциальные уравнения

Такие уравнения были рассмотрены нами в предыдущих параграфах, а значит, общий интеграл исходного уравнения представляет собой совокупность решений всех отдельных уравнений.

Рассмотрим некоторые уравнения, которые не разрешимы относительно производной у’.

Уравнения вида Дифференциальные уравнения

Предположим, что уравнение Дифференциальные уравнения разрешимо только относительно переменной у, и пусть Дифференциальные уравнения— какое-либо его действительное решение. В таком случае решение уравнения Дифференциальные уравнения, а тогда и уравнения Дифференциальные уравнениябудем искать в параметрической форме.

Положим у’ = t (t — параметр). Равенство у = g(у’) дифференцируем по переменной х с последующей заменой у’ на t. Учитывая, что t = t(x) есть функция от х, получаем

Дифференциальные уравнения

Равенство крайних членов Дифференциальные уравненияпредставляет собой дифференциальное уравнение, которое перепишем в виде (разделяем переменные) Дифференциальные уравнения Отсюда Дифференциальные уравненияи к этому равенству присоединяем первоначальное уравнение у = g(у’) т. е. y=g(t) Получаем параметрическое решение исходного уравнения
y = g (y)

Дифференциальные уравнения

Это решение составляет часть решения уравнения Дифференциальные уравнения

Уравнения вида Дифференциальные уравнения

Предположим, что уравнение Дифференциальные уравненияразрешимо только относительно переменной x и х = g(у’) — какое-то его действительное решение. Это — уравнение, не разрешенное относительно y’. Его решение будем искать также в параметрической форме.

Положим у’ = t, т.е. Дифференциальные уравненияОтсюда Дифференциальные уравненияРавенство х = g (у’), т.е.
х = g(t), дифференцируем по х:

Дифференциальные уравнения

Отсюда dx = g'(t) dt. Сравнивая это равенство с равенством Дифференциальные уравненияДифференциальные уравненияполучаем Дифференциальные уравненияили Дифференциальные уравнения

Это равенство можно проинтегрировать и присоединить к нему равенство x = g (t).

Получаем: Дифференциальные уравнения параметрическое решение уравнения
х = g(у’) составляющее также часть решений исходного уравнения Дифференциальные уравнения

Уравнение Клеро

Уравнением Клеро называется уравнение вида Дифференциальные уравненияне разрешенное относительно у’, где Дифференциальные уравнения— некоторая известная функция.

1) Подстановка у’ = С (С — произвольная постоянная) показывает, что Дифференциальные уравнения есть общее решение уравнения Клеро (из этого равенства следует, что у’ = С). Таким образом, общее решение уравнения Клеро геометрически представляет собой семейство прямых линий.

2) Уравнение Клеро допускает еще одно решение, которое не получается из общего, а потому не является частным и называется особым решением. Это решение найдем в параметрической форме по схеме, описанной выше.

Положим у’ = t (t — переменная, параметр) и подставим в уравнение Клеро. Получим Дифференциальные уравненияи продифференцируем это равенство по х с последующей заменой у’ на t:

Дифференциальные уравнения

Отсюда Дифференциальные уравненияа тогда Дифференциальные уравнения( t’ = 0 приводит к
t = С, у’ = С, а это уже было рассмотрено). К этому равенству присоединяем уравнение Клеро и тем самым получаем параметрическое выражение особого решения:

Дифференциальные уравнения

Уравнение Лагранжа

Уравнением Лагранжа называется уравнение вида

Дифференциальные уравнения

не разрешенное относительно производной у’, где Дифференциальные уравненияи Дифференциальные уравнения— некоторые известные непрерывные функции.

Общий интеграл этого уравнения найдем в параметрической форме. Положим у’ = t. Тогда Дифференциальные уравнения Дифференцируем это равенство по х. Получаем последовательно

Дифференциальные уравнения

Заменим здесь у’ на t, a t’ на Дифференциальные уравнения и перепишем полученное равенство в виде

Дифференциальные уравнения

или

Дифференциальные уравнения

Это уравнение, очевидно, является линейным дифференциальным уравнением относительно неизвестной функции x = x(t) при условии Дифференциальные уравнения

К его общему решению присоединим равенство (уравнение) Лагран-жа и в результате получим общее решение уравнения Лагранжа в параметрической форме:

Дифференциальные уравнения

Случай обращения в нуль множителя Дифференциальные уравнения при некотором значении Дифференциальные уравнения параметра t приводит к Дифференциальные уравнения и
Дифференциальные уравненияДифференциальные уравнения

Это — решение уравнения Лагранжа, не получаемое из общего решения, а потому оно является особым.

Примеры с решениями

Проинтегрировать дифференциальные уравнения, не разрешенные относительно производной.

Пример:

Дифференциальные уравнения

Решение:

Анализ левой части уравнения позволяет разложить ее на множители:

Дифференциальные уравнения

Это уравнение равносильно совокупности двух уравнений с разделяющимися переменными:

Дифференциальные уравнения

Общее решение исходного уравнения состоит из совокупности двух функций, каждая из которых зависит от своей произвольной постоянной (правая функция определена явно, вторая неявно, но для единообразия решение запишем в неявной форме).

О т в е т Дифференциальные уравнения или Дифференциальные уравнения

Примечание. В общем решении мы используем две константы Дифференциальные уравнения и Дифференциальные уравнения, чтобы подчеркнуть независимость решений отдельных уравнений. Однако использование одной постоянной не является ошибкой.

Пример:

Дифференциальные уравнения

Решение:

Данное уравнение необходимо разрешить как квадратное уравнение относительно производной у’.

Находим:

Дифференциальные уравнения

Пришли к совокупности уравнений, каждое из которых решается одним из известных способов. Первое решим заменой:

Дифференциальные уравнения

Второе интегрируем непосредственно:

Дифференциальные уравнения

Ответ Дифференциальные уравненияДифференциальные уравнения

Примечание:

К исходным уравнениям примеров 1 и 2 (не разрешенным относительно производной у’) теорема существования и единственности неприменима. В качестве иллюстрации заметим, что в примере 2 через точку Дифференциальные уравнения проходят две различные интегральные кривые: Дифференциальные уравнения и Дифференциальные уравнения

Пример:

Дифференциальные уравнения

Решение:

Для разложения левой части на множители преобразуем первую дробь:

Дифференциальные уравнения

Теперь нетрудно заметить, что исходное уравнение можно представить в виде

Дифференциальные уравнения

Решая два простых уравнения (предлагаем сделать это самостоятельно), приходим к общему решению.

Ответ

Дифференциальные уравнения

Для следующих уравнений приемы, примененные выше, неэффективны. Общие их решения можно представить только параметрически.

Пример:

Дифференциальные уравнения

Решение:

Данное уравнение можно отнести к уравнениям общего вида Дифференциальные уравнения и применить к нему рассуждения п. 6.2.

Положим у’ = t (t — параметр). Получаем равенство Дифференциальные уравнениякоторое дифференцируем по х (подчеркнем, что t = у’ — это функция от х, t = t(x), а тогда Дифференциальные уравнения Получаем Дифференциальные уравненияДифференциальные уравнения или (заменяя y’ = t ) Дифференциальные уравнения Анализируем это равенство.

1) Если t = 0, т.е. у’ = 0, то функция у = С является решением полученного уравнения, но, вообще говоря, не удовлетворяет исходному уравнению: Дифференциальные уравненияимеет место только при
С = 0, что дает одно тривиальное решение исходного уравнения: у = 0.

2) Если Дифференциальные уравнения то анализируемое уравнение можно представить в виде Дифференциальные уравнения или dx = 3(2-5t)dt. Его решение:

Дифференциальные уравнения

Это равенство не может определять общее решение исходного уравнения, ибо t не является независимой переменной (поскольку
t = у’) и, кроме этого, в полученном равенстве нет переменной у, а потому нет и аналитического выражения интегральной кривой, т.е. зависимости между х и у.

Присоединяем к полученному равенству исходное уравнение (равенство) с t = у’ в качестве параметра. Получаемая система представляет собой общее решение исходного дифференциального уравнения в параметрической форме.

Ответ

Дифференциальные уравнения

Пример:

Дифференциальные уравнения

Решение:

относительно производной Дифференциальные уравнения а отсюда получится общее решение исходного уравнения. Однако наша цель — проиллюстрировать рассуждения п. 6.3. Положим у’ = t, Дифференциальные уравненият. е. Дифференциальные уравнения

Исходное уравнение можно переписать в виде Дифференциальные уравнения

Продифференцируем это равенство пo Дифференциальные уравненияПоскольку Дифференциальные уравнения получаем уравнение Дифференциальные уравненияотносительно неизвестной функции y = у(х) — у (х (t)).

Интегрирование дает:

Дифференциальные уравнения

Присоединяем к этому равенству первоначальное равенство Дифференциальные уравненияи тем самым получаем общее решение исходного уравнения в параметрической форме.

Ответ

Дифференциальные уравнения

Пример:

Решить уравнение Клеро Дифференциальные уравнения

Решение:

Уравнение Клеро Дифференциальные уравнения(здесь Дифференциальные уравнения)

решается в два этапа.

1) Заметим, что подстановка у’ = С (Дифференциальные уравнения— произвольная постоянная) удовлетворяет данному уравнению: из него следует, что Дифференциальные уравненияположив здесь Дифференциальные уравнения приходим к тождествуДифференциальные уравнения (левая часть получена интегрированием равенства у’ = С, а правая часть взята из уравнения с заменой у’ = С). Значит, равенство Дифференциальные уравнения дает общее решение исходного уравнения.

2) Поиск общего решения. Положим в исходном уравнении, как обычно, у’ = t, а затем продифференцируем полученное равенство по х с последующей заменой у’ на t:

Дифференциальные уравнения

Анализируем это равенство:

а) t’ = 0 Дифференциальные уравненияt = С Дифференциальные уравненияу’ = С; получили соотношение, уже рассмотренное в п. 1); оно приводит к общему решению;

б) Дифференциальные уравнения Дифференциальные уравнения — (зависимость х от параметра t).

Присоединив к этому выражению зависимость у от t, получим параметрическое решение: Дифференциальные уравнения, Дифференциальные уравнения

Кстати, данный вид функции Дифференциальные уравненияпозволяет явно выразить параметр t через х и тем самым получить явную функцию у = у(х) — решение исходного уравнения. Сделаем это:

Дифференциальные уравнения

подставив в равенство Дифференциальные уравнения получаем

Дифференциальные уравнения
Дифференциальные уравнения

Отсюда Дифференциальные уравнения (наиболее простая неявная форма записи). Это так называемое особое (не частное) решение исходного уравнения, которое не получается из общего решения ни при каком значении постоянной С.

Ответ

Дифференциальные уравнения

Примечание:

Между общим и особым решениями уравнений Клеро имеется связь, которая допускает геометрическую иллюстрацию.

График Г особого решения представляет собой полукубическую
параболу Дифференциальные уравнениясимметричную относительно оси у (рис. 4.3).

Дифференциальные уравнения

Оказывается, что общее решение есть семейство прямых Дифференциальные уравнения ( С — произвольная постоянная, Дифференциальные уравнения), причем каждая прямая семейства является касательной к Г, а точка (x, у) графика Г принадлежит одновременно и какой-либо прямой Дифференциальные уравнения Говорят, что особое решение составляет огибающую семейства прямых (общих интегральных кривых).

Вообще, предположим, что некоторое дифференциальное уравнение обладает общим решением со следующим свойством: существует интегральная кривая, не принадлежащая этому семейству, но имеющая общую точку с каждой интегральной кривой общего решения. Тогда эта новая линия является огибающей первого семейства и особым решением исходного уравнения. Именно такова линия Дифференциальные уравнениядля уравнения примера 6.

Пример:

Проинтегрировать дифференциальное уравнение Лагранжа Дифференциальные уравнения

Решение:

Заменим у’ = t. Уравнение принимает вид Дифференциальные уравненияЭто равенство дифференцируем по х, затем вместо у’ подставим

Дифференциальные уравнения

Поскольку Дифференциальные уравнениято последнее уравнение можно представить в виде линейного дифференциального уравнения относительно неизвестной функции Дифференциальные уравнения

Решим это уравнение подстановкой Дифференциальные уравненияx’=u’v + u v’ ПолучаемДифференциальные уравнения

Дифференциальные уравнения

Из этой системы получаем общее решение линейного уравнения:

Дифференциальные уравнения

К этому равенству присоединяем выражение для у и получаем общее решение исходного уравнения Лагранжа в параметрической форме.

Ответ

Дифференциальные уравнения

Примечание:

В последнем равенстве можно вместо х вписать его выражение из первого равенства.

Дифференциальные уравнения порядка выше первого. Уравнения, допускающие понижение порядка

1°. Дифференциальное уравнение порядка n в общем виде записывается так:

Дифференциальные уравнения

Для уравнения порядка n, разрешенного относительно старшей производной:

Дифференциальные уравнения

справедлива теорема существования и единственности решения.

Теорема:

Если функция Дифференциальные уравненияДифференциальные уравнения ее частные производные по переменным Дифференциальные уравнениянепрерывны в некоторой области D из Дифференциальные уравнениясодержащей точку с координатами

Дифференциальные уравнения

существует, и притом единственное, решение у = у (х) уравнения (1), удовлетворяющее условиям

Дифференциальные уравнения

Эти условия называются начальными условиями, или условиями Коши.

2°. Общим решением дифференциального уравнения порядка n

Дифференциальные уравнения

называется функция зависящая от n произвольных постоянных Дифференциальные уравнения такая, что:

Дифференциальные уравнения

1) она удовлетворяет уравнению при любых значениях постоянных;

2) при заданных начальных условиях

Дифференциальные уравнения

где точка Дифференциальные уравненияпринадлежит области D, можно подобрать такие значения постоянных Дифференциальные уравнения что

Дифференциальные уравнения

будет удовлетворять этим условиям.

Иногда общее решение определяется более общим соотношением

Дифференциальные уравнения

называемым общим интегралом, или неявным общим решением дифференциального уравнения.

3°. Для уравнения второго порядка у» = f(x,y, y’) начальные условия Дифференциальные уравненияДифференциальные уравнениявыражают условия определения интегральной кривой у = у(х), проходящей через данную точку Дифференциальные уравнения и имеющей в этой точке угловой коэффициент касательной (или, другими словами, направление), равный Дифференциальные уравнения

Согласно теореме 2, через данную точку Дифференциальные уравнения может проходить бесконечное множество интегральных кривых, но только одна — в направлении, определенном значением Дифференциальные уравнения

4°. Уравнения второго и более высоких порядков допускают решение или интегрирование в отдельных случаях; некоторые типы таких уравнений перечислим здесь.

1) Простейшее уравнение порядка n имеет вид

Дифференциальные уравнения

Общее решение этого уравнения получается в результате последовательного n-кратного интегрирования.

2) Уравнение вида

Дифференциальные уравнения

допускает понижение порядка на к единиц при помощи подстановок

Дифференциальные уравнения

не содержащее явно переменную у, сводится к двум уравнениям первого порядка:

Дифференциальные уравнения

где Дифференциальные уравнения — общее решение уравнения F(x,p,p’) = 0.

3) Уравнение второго порядка, не содержащее явно независимую переменную х:

Дифференциальные уравнения

подстановками

Дифференциальные уравнения

(переменная у принимается в качестве независимой) сводится к двум уравнениям первого порядка:

Дифференциальные уравнения

где Дифференциальные уравнения— общее решение уравнения F(y,p,p’) =0.

4) Подстановкой Дифференциальные уравненияможно понизить на k единиц порядок n уравнения вида

Дифференциальные уравнения

Примеры с решениями

Пример:

Решить уравнение Дифференциальные уравнения

Решение:

Уравнение относится к типу 1). Интегрируем:

Дифференциальные уравнения

Интегрируем еще раз:

Дифференциальные уравнения

Далее, по аналогии:

Дифференциальные уравнения

Последнее интегрирование приводит к ответу (найдите его самостоятельно).

Ответ

Дифференциальные уравнения

Примечание:

Произведения (частные от деления) постоянных на числа являются также постоянными, поэтому предыдущий ответ можно записать в более удобном виде: Дифференциальные уравненияДифференциальные уравнения Дифференциальные уравненияДифференциальные уравненияДифференциальные уравненияДифференциальные уравнения

Пример:

Найти общее решение уравнения Дифференциальные уравненияДифференциальные уравнения и частное решение, удовлетворяющее начальным условиям х = 0, у = 1, у’ = 1.

Решение:

Данное уравнение не содержит явно переменную у, поэтому, согласно 2), воспользуемся подстановкой у’ = р, у» = р’. Приходим к уравнению с разделяющимися переменными относительно р:

Дифференциальные уравнения

Отсюда arctg р = — arctg х + arctg C, или

Дифференциальные уравнения

Остается проинтегрировать простейшее уравнение вида 1) (или с разделенными переменными) Дифференциальные уравнения Получаем

Дифференциальные уравнения

это общее решение. Для получения искомого частного решения подставим сюда и в выражение для у’ значения х = 0, у = 1, у’ = 1 и определим Дифференциальные уравнения

Эти значения подставим в выражение для у, откуда и получим искомое частное решение.

Дифференциальные уравнения

Пример:

Найти частное решение уравнения Дифференциальные уравненияудовлетворяющее начальным условиям х = 1, Дифференциальные уравнения у’ = 1.

Решение:

Визуально определяем, что это уравнение типа 3) — в нем явно отсутствует независимая переменная х. Принимаем у в качестве нового аргумента и выполняем подстановку у’ = р = р(у(х)). По правилу дифференцирования сложной функции находим

Дифференциальные уравнения

а исходное уравнение приводим к уравнению Дифференциальные уравненияили
р’ = cos y (при этом мы опускаем одно решение р = 0, или у = С, которое не является общим: оно не содержит двух произвольных постоянных; если же рассматривать это решение как частное, то оно не удовлетворяет начальному у’ = 1). Находим p = sin y + C, или
y’ = sin y + C. Вычислим сначала значение С , исходя из начальных условий Дифференциальные уравненияу’ = 1. Получим С = 0 и решим уравнение у’ = sin у, т.е.Дифференциальные уравнения

Находим Дифференциальные уравненияИз начальных условий х = 1, Дифференциальные уравненияопределяем Дифференциальные уравнения

Тогда Дифференциальные уравнения т.е. Дифференциальные уравненияДифференциальные уравнения

Ответ Дифференциальные уравненияДифференциальные уравнения

Примечание:

В этом примере требовалось найти у (частное решение), поэтому не стали интегрировать уравнение y’ = siny + C, илиДифференциальные уравненияв общем виде, хотя это можно было делать. Это следует учитывать в тех случаях, когда промежуточное уравнение в общем виде трудно интегрировать.

Пример:

Найти частное решение дифференциального уравнения

Дифференциальные уравнения

удовлетворяющее начальным условиям Дифференциальные уравненияДифференциальные уравнения Дифференциальные уравненияДифференциальные уравненияДифференциальные уравнения

Решение:

Данное уравнение относится к типу 2) с n = 3 и k = 2. Обозначим у» = р(х). Тогда у‘» = р’. После почленного деления уравнения на Дифференциальные уравненияи соответствующих замен приходим к уравнению

Дифференциальные уравнения

Это линейное уравнение первого порядка решим подстановкой
р = uv, р’ = u’v + vu’. Опуская промежуточные выкладки (которые надо выполнить самостоятельно), укажем, что общее решение этого линейного уравнения имеет вид

Дифференциальные уравнения

После двух интегрирований находим общее решение исходного уравнения:

Дифференциальные уравнения

Значения трех постоянных определим исходя из начальных условий:

Дифференциальные уравнения

Начнем с равенства Дифференциальные уравнения, в которое подставим x = 1,

Дифференциальные уравнения

Находим Дифференциальные уравнения А теперь в равенство Дифференциальные уравнения

подставляем x = 1, Дифференциальные уравнения Получаем Дифференциальные уравненияНаконец, в равенство Дифференциальные уравненияДифференциальные уравнения подставляем

Дифференциальные уравнения

Ответ Дифференциальные уравнения

Пример:

Решить уравнение

Дифференциальные уравнения

Решение:

Прежде чем решить то или иное уравнение, имеет смысл проанализировать его конструкцию с целью обнаружения интегрируемых выражений или комбинаций.

В данном случае замечаем равенства (производные берутся по х):

Дифференциальные уравнения

а значит, исходное уравнение можно переписать в виде уравнения с разделенными переменными (вместо переменных имеем целые выражения):

Дифференциальные уравнения

Отсюда следует неравенство

Дифференциальные уравнения

В левой его части имеем (lnу’)’, а значит, после интегрирования получаем

Дифференциальные уравнения

Интегрируя по частям, имеем:

Дифференциальные уравнения

Тем самым общее решение получено.
Ответ

Дифференциальные уравнения

Пример:

Решить дифференциальное уравнение у»( 1 +2 ln у’) = 1.
Решение:

Уравнение можно отнести как к типу 2), так и к типу 3).

а) Принимаем, как для типа 2) р = у’ (х), р’ = у» (х). Приходим к уравнению с разделенными переменными (1 +2lnp)dp = dx. После интегрирования получаем Дифференциальные уравнения, или 2у’lnу’ — у’ = х + С1.

Это уравнение не разрешимо относительно производной у’ и не встречается среди типов уравнений, которые можно интегрировать. Принимаем его как неразрешимое уравнение.

б) Принимаем, как для типа 3), р = р(у) = у’, Дифференциальные уравнения

Приходим к уравнению первого порядка pp'( 1+ 2ln p) = 1, или

р(1 + 2lnp) dp = dy. После интегрирования получаем Дифференциальные уравненияДифференциальные уравнения или Дифференциальные уравнения

Это уравнение первого порядка также не встречается среди интегрируемых типов. Принимаем его как неразрешимое.

в) Переписываем результаты предыдущих пунктов а) и б) в виде системы

Дифференциальные уравнения

и принимаем эту систему в качестве общего решения исходного уравнения в параметрической форме с параметром t = у’.

Ответ

Дифференциальные уравнения

Линейные однородные дифференциальные уравнения второго порядка с постоянными коэффициентами

1°. Линейным дифференциальным уравнением второго порядка (кратко ЛДУ) называется уравнение вида

ay» + by’ + су = f(x)

(а, b, с — известные числа, f(х) — непрерывная функция) .

Если Дифференциальные уравнения в некотором промежутке, то уравнение называется однородным (кратко ЛОДУ), в противном случае уравнение называется неоднородным (кратко ЛНДУ).

Обозначим А (у) = ay» + by’ + су. Тогда ЛОДУ можно записать так:
A(y) = 0, а ЛНДУ так: A(у) = f(x). В таком случае говорят об операторной записи дифференциального уравнения.

Запись Дифференциальные уравнения (или Дифференциальные уравнения) означает, что функцияДифференциальные уравнения является решением ЛОДУ (ЛНДУ).

2°. Две дважды дифференцируемые функции Дифференциальные уравненияиДифференциальные уравненияназываются линейно независимыми на некотором отрезке [а; b], если их отношение не является постоянной величиной на этом отрезке, и линейно зависимыми в противном случае. Другими словами, если Дифференциальные уравнениято Дифференциальные уравнения и Дифференциальные уравнениялинейно независимы, а если Дифференциальные уравненияпостоянно, то Дифференциальные уравненияи Дифференциальные уравнения линейно зависимы.

О линейной зависимости или независимости функций Дифференциальные уравнения и Дифференциальные уравненияможно судить по определителю Вронского

Дифференциальные уравнения

Если функции Дифференциальные уравнения и Дифференциальные уравнения линейно зависимы на отрезке [a; b], то Дифференциальные уравнения для всех х из [а; b].

Теорема:

Вронского. Если Дифференциальные уравнения и Дифференциальные уравнения — линейно независимые на отрезке [а; b] решения ЛОДУ, то определитель Вронского этих функций отличен от нуля в каждой точке этого отрезка.

3°. Общее решение ЛОДУ (и ЛНДУ) зависит от корней характеристического уравнения (кратко: хар. ур.) Дифференциальные уравнениясоответствующего ЛОДУ, получаемого заменой порядка производной неизвестной функции степенью обычной неизвестной переменной, которую обозначим буквой k. Общее решение можно найти, пользуясь только алгебраическими действиями. Вывод о структуре общего решения Дифференциальные уравнениятаких уравнений можно делать из следующих утверждений, которые легко проверить непосредственно (рекомендуем это проделать самостоятельно в качестве упражнений по решению квадратных уравнений и дифференцированию функций):

Дифференциальные уравнения

4) следующие условия равносильны:

Дифференциальные уравнения

5) если уравнение Дифференциальные уравнения имеет два равных корня Дифференциальные уравнения

Дифференциальные уравнения

6) если уравнение Дифференциальные уравнения имеет два комплексно сопряженных корня Дифференциальные уравнения то Дифференциальные уравнения и Дифференциальные уравнения

Теорема:

Если Дифференциальные уравнения и Дифференциальные уравнения — два линейно независимых решения ЛОДУ, то общее решение ЛОДУ имеет вид

Дифференциальные уравнения

Примечание:

Теорема 3 обеспечивает возможность получения частного решения данного дифференциального уравнения при любых начальных условиях.

4°. Алгоритм решения ЛОДУ с постоянными коэффициентами состоит в составлении и решении характеристического уравнения Дифференциальные уравнения Если при этом оно имеет:

1) два различных действительных корня Дифференциальные уравнения то

Дифференциальные уравнения

2) два равных действительных корня Дифференциальные уравнения то

Дифференциальные уравнения

3) два комплексно сопряженных корня Дифференциальные уравнениято Дифференциальные уравнения

Дифференциальные уравнения

Пример:

Проверить на линейную зависимость или линейную независимость следующие пары функций:

Дифференциальные уравнения

Решение:

Все функции заданы на всей числовой прямой. Пары функций а) и б) линейно независимы, так как их отношения не являются постоянными величинами: Дифференциальные уравнения

Пары функций в) и д) линейно зависимы, так как их отношения являются постоянными: Дифференциальные уравнения

Пример:

Проверить, что данные функции являются решениями данных дифференциальных уравнений, и составить общие решения этих уравнений:

Дифференциальные уравнения

Решение:

1) Находим первую и вторую производные первой функции:

Дифференциальные уравнения

Следовательно, Дифференциальные уравненияесть частное решение уравнения Дифференциальные уравненияДифференциальные уравненияАналогично можно проверить, что функция Дифференциальные уравнениятакже удовлетворяет этому уравнению (предлагаем проделать это самостоятельно). Данные функции линейно независимы, т.к. отношение Дифференциальные уравнения

Согласно теореме 4 общее решение данного уравнения имеет вид

Дифференциальные уравнения

2) Аналогично решается этот пункт примера. Имеем (ограничимся решением Дифференциальные уравнения данного уравнения);

Дифференциальные уравнения

3) Ограничимся проверкой функции Дифференциальные уравнения(завершить самостоятельно):

Дифференциальные уравнения

Пример:

Найти общие решения данных линейных однородных дифференциальных уравнений с постоянными коэффициентами. В тех случаях, где даны начальные условия, найти соответствующие частные решения:

Дифференциальные уравнения

Решение:

Все дифференциальные уравнения решаются по единой схеме, поэтому незначительные детали иногда опускаются. Составляем характеристическое уравнение, находим его корни и общее решение ЛОДУ строим согласно алгоритму п. 4°.

Имеем Дифференциальные уравненияДифференциальные уравнения

Согласно 1) п. 4° получаем

ОтветДифференциальные уравнения

Дифференциальные уравнения

Согласно 2) п. 4° получаем общее решение

Дифференциальные уравнения

Для получения частного решения необходимо это равенство продифференцировать:

Дифференциальные уравнения

Подставляя в последние равенства начальные условия х = 1, у = 2,
у’ = 1, приходим к системе

Дифференциальные уравнения

Отсюда Дифференциальные уравнения

Дифференциальные уравнения

Согласно 3) п. 4° получаем
Ответ

Дифференциальные уравнения

4) Дифференциальные уравнения Дифференциальные уравненияОбщее решение однородного уравнения имеет вид

Дифференциальные уравнения

Это равенство дифференцируем:

Дифференциальные уравнения

Подставляя в выражения для у и у’ начальные условия Дифференциальные уравнения у = 2,
у’ + 1, получаем систему из двух линейных уравнений с двумя неизвестными:

Дифференциальные уравнения

Разрешимость системы обеспечивается теоремой 2. Находим Дифференциальные уравнения

Дифференциальные уравнения

Линейные неоднородные дифференциальные уравнения второго порядка с постоянными коэффициентами

1°. Уравнение вида

Дифференциальные уравнения

является линейным неоднородным дифференциальным уравнением (ЛНДУ) второго порядка с постоянными коэффициентами, или уравнением с правой частью, а уравнение А(у) = 0 однородным уравнением, соответствующим неоднородному уравнению А(у) = f(x).

Теорема:

Если Дифференциальные уравнения— некоторое частное решение ЛНДУ, то общее решение этого уравнения имеет вид Дифференциальные уравнения— общее решение ЛОДУ).

Теорема:

Пусть Дифференциальные уравнения — частное решение уравнения А(у) = /|(х); У2 — частное решение уравнения Дифференциальные уравненияТогда Дифференциальные уравнения есть частное решение уравнения Дифференциальные уравнения

2°. Частное решение уч ЛНДУ можно найти методом неопределенных коэффициентов (алгебраически), если правая часть /(ж) уравнения имеет специальный вид.

Ниже Pn,Qm. … — известные многочлены соответственно степеней п и то, Рn, Qm — многочлены с неизвестными коэффициентами (индекс многочлена обозначает его степень).

Дифференциальные уравнения

3°. Неизвестные коэффициенты многочленов, фигурирующих в теоремах 7 и 8, вычисляются следующим образом.

Выражения для Дифференциальные уравненияподставляют в левую часть решаемого уравнения, а затем приравнивают коэффициенты при подобных членах в правой и левой частях уравнения. Получаемая система уравнений и определяет искомые коэффициенты.

Примечание:

Теоремы 5-8 распространяются на уравнения порядка n:

Дифференциальные уравнения

Общее решение Дифференциальные уравнения уравнения

Дифференциальные уравнения

представляет собой линейную комбинацию линейно независимых частных решений этого уравнения, которые получаются аналогичным образом, по корням характеристического уравнения

Дифференциальные уравнения

4°. Если правая часть неоднородного уравнения второго порядка имеет вид. отличный от тех, что указаны в теоремах 7-8, то частное решение для таких уравнений можно найти методом вариации произвольных постоянных.

Теорема:

Пусть Дифференциальные уравненияи Дифференциальные уравнения — два частных линейно независимых решения ЛОДУ. Тогда существует функции Дифференциальные уравнения, такие, что функция Дифференциальные уравнения является частным решением ЛНДУ и Дифференциальные уравнения При этом функции Дифференциальные уравненияявляются решениями системы дифференциальных уравнений первого порядка

Дифференциальные уравнения

Примеры с решениями

Пример:

Решить уравнение Дифференциальные уравнения

Решение:

Имеем: Дифференциальные уравнения Дифференциальные уравненияДифференциальные уравненияДифференциальные уравнения

Правая часть неоднородного уравнения имеет вид Дифференциальные уравненияДифференциальные уравнения т.е. Дифференциальные уравненияне есть корень характеристического уравнения. Согласно п. 1) теоремы 3 частное решёние неоднородного уравнения следует выбирать в виде:

Дифференциальные уравнения

где А, В,С и D — неизвестные числа. Для их определения подставим в исходное уравнение выражения для Дифференциальные уравнения

Вычисления организуем следующим образом. Выражения для Дифференциальные уравнениязапишем в столбик, а перед ними — соответствующие коэффициенты уравнения.

Дифференциальные уравнения

Систему уравнений получили в результате приравнивания коэффициентов при одинаковых степенях переменной х в правой и левой частях уравнения, получаемых при подстановке в него выражений для Дифференциальные уравнения (метод неопределенных коэффициентов):

Дифференциальные уравнения

Общее решение неоднородного уравнения Дифференциальные уравнениясоставим согласно теореме 1

Ответ

Дифференциальные уравнения

Пример:

Найти общее решение уравнения Дифференциальные уравнения

Указать также частное решение, удовлетворяющее начальным условиям x = 0, y = 1, y’= — 3

Решение:

Имеем

Дифференциальные уравнения

Правая часть имеет вид

Дифференциальные уравнения

однократный корень характеристического уравнения. Согласно п. 2) теоремы 3:

Дифференциальные уравнения

Сравнение коэффициентов при одинаковых степенях х в правой и левой частях уравнения приводит к системе уравнений (формально имеем 4 уравнения и 3 неизвестных, но система всегда разрешима, ибо одно из уравнений является тривиальным):

Дифференциальные уравнения

Получили

Дифференциальные уравнения

Дифференцируем: Дифференциальные уравненияДифференциальные уравнения

Начальные условия приводят к новой системе относительно неизвестных постоянных: Дифференциальные уравненияДифференциальные уравненияДифференциальные уравненияДифференциальные уравнения

Ответ

Дифференциальные уравнения

Пример:

Решить уравнение

Дифференциальные уравнения

Решение:

Имеем:

Дифференциальные уравнения
Дифференциальные уравнения

Дано Дифференциальные уравнения — двукратный корень хар ур., n = I. Согласно п. 3) теоремы 3 составим соответствующую систему, предварительно преобразуя и предполагаемое частное решение и дифференцируя его.

Дифференциальные уравнения

Приравниваем коэффициенты при одинаковых степенях:

Дифференциальные уравненияДифференциальные уравнения(триивиальное уравнение);

Дифференциальные уравнения(тривиальное уравнение);

Дифференциальные уравнения

Ответ

Дифференциальные уравнения

Пример:

Решить уравнение Дифференциальные уравненияДифференциальные уравненияДифференциальные уравнения

Решение:

Дифференциальные уравненияПравую часть уравнения сравним f(х) из теоремы 8. Имеем Дифференциальные уравнения n = 0, Дифференциальные уравненияn = 1,
s = mах(0, 1) = 1, числа Дифференциальные уравнения не являются корнями характеристического уравнения. По теореме 4, п. 1 будем иметь (при дифференцировании подобные члены объединяются):

Дифференциальные уравнения

Ниже в начале каждой строки записан признак подобия сравниваемых членов.

Дифференциальные уравнения

Пример:

Решить уравнение Дифференциальные уравненияДифференциальные уравненияДифференциальные уравнения

Решение:

Дифференциальные уравнения

Далее (см. теорему 2) находим два частных решения (решаем как бы два уравнения, что равносильно одному уравнению с двумя независимыми правыми частями).

а) Находим частное решение уравнения соответствующее тригонометрической части, имея в виду, что числа Дифференциальные уравнения— это корни характеристического уравнения.

Дифференциальные уравнения

б) Находим частное решение, соответствующее алгебраической части:

Дифференциальные уравнения

Пример:

Решить уравнение

Дифференциальные уравнения

Решение:

Имеем:

Дифференциальные уравнения
Дифференциальные уравнения

Принимаем Дифференциальные уравненияв качестве двух линейно независимых частных решений ЛОДУ. Тогда Дифференциальные уравненияи Дифференциальные уравнениярешим систему дифференциальных уравнений (из теоремы 9):

Дифференциальные уравнения

Подчеркнем, что Дифференциальные уравненияДифференциальные уравнения — функции переменной х. При помощи действий, указанных справа от системы, получаем:

Дифференциальные уравнения

Линейные дифференциальные уравнения с постоянными коэффициентами порядка выше второго

1°. Линейное дифференциальное уравнение с постоянными коэффициентами порядка выше второго имеет вид

Дифференциальные уравнения

где n > 2.

Если Дифференциальные уравнениято уравнение называется однородным, в противном случае — неоднородным.

Схема решения уравнений второго порядка распространяется на уравнения порядка выше второго, с учетом конкретной специфики.

Например, корни характеристического уравнения

Дифференциальные уравнения

вносят следующую специфику:

1) каждому простому действительному корню k (или паре комплексных корней Дифференциальные уравнения) характеристического уравнения соответствует единственное частное решение вида Дифференциальные уравнения(пара частных решений вида Дифференциальные уравненияи Дифференциальные уравнения) исходного уравнения;

2) каждому действительному корню k кратности r соответствуют r линейно независимых корней исходного уравнения:

Дифференциальные уравнения

3) каждой паре комплексных корней Дифференциальные уравнениякратности r соответствуют г пар линейно независимых решений исходного уравнения:

Дифференциальные уравнения

Теорема:

Общее решение ЛОДУ порядка п является линейной комбинацией всех n его частных линейно независимых решений, соответствующих корням характеристического уравнения, с произвольными коэффициентами. Общее решение ЛОДУ порядка п зависит от n произвольных постоянных.

Общее решение ЛНДУ представляет собой сумму общего решения ЛОДУ и одного частного решения ЛНДУ.

2°. Линейная независимость системы решений Дифференциальные уравненияЛОДУ на данном отрезке определяется ненулевым значением определителя Вронского для этой системы, который имеет вид

Дифференциальные уравнения

3°. Подбор частного решения неоднородного уравнения порядка n осуществляется так же, как в случае уравнения второго порядка, с учетом информации, приведенной в п. 1.

4°. Метод вариации произвольных постоянных для нахождения частного решения неоднородного уравнения состоит в определении соответствующего набора констант, зависящих от аргумента х и удовлетворяющих некоторой системе дифференциальных уравнений. Для случая уравнения третьего порядка

Дифференциальные уравнения

общее решение однородного уравнения имеет вид

Дифференциальные уравнения

где Дифференциальные уравнения — фундаментальная система решений однородного уравнения (система линейно независимых решений); частное решение неоднородного уравнения имеет вид

Дифференциальные уравнения

где соответствующие константы являются решениями системы уравнений

Дифференциальные уравнения

Примеры с решениями

Пример:

Найти общее решение дифференциального уравнения

Решение:

Корни характеристического уравнения Дифференциальные уравненияДифференциальные уравненияможно определить методом разложения на множители:

Дифференциальные уравнения

Находим

Дифференциальные уравнения

Ответ.

Дифференциальные уравнения

Пример:

Найти частное решение дифференциального уравнения Дифференциальные уравнения удовлетворяющее начальным условиям x = 0, y= 1, y’ = 0, у» = 4.

Решение:

1)По характеристическому уравнению Дифференциальные уравнения и его корням Дифференциальные уравненияДифференциальные уравнения составим общее решение однородного уравнения:

Дифференциальные уравнения

2) По правой части уравнения подберем частное решение неоднородного уравнения

Дифференциальные уравнения

Коэффициенты А, В, С определим известным образом из соответствующей системы линейных уравнений. Сначала составим таблицу производных:

Дифференциальные уравнения

А теперь составим систему:

Дифференциальные уравнения

Получаем

Дифференциальные уравнения

3) Составим общее решение исходного уравнения:

Дифференциальные уравнения

4) Остается найти то частное решение неоднородного уравнения, которое удовлетворяет начальным условиям. Для этого необходимо дважды продифференцировать Дифференциальные уравнения

Дифференциальные уравнения

В выражения для Дифференциальные уравненияи подставим начальные условия
х = 0, у = 1, Дифференциальные уравнения После подстановки получаем систему, из которой определим Дифференциальные уравнения

Дифференциальные уравнения
Дифференциальные уравнения

Пример:

Решить дифференциальное уравнение Дифференциальные уравнения

Решение:

Корнями характеристического уравнения Дифференциальные уравненияявляются числа Дифференциальные уравненияДифференциальные уравнения

Поэтому общее решение однородного уравнения имеет вид Дифференциальные уравненияДифференциальные уравнения Частное решение неоднородного уравнения можно определить только методом вариации произвольных постоянных, т.е. Дифференциальные уравненияДифференциальные уравненияДифференциальные уравнения где являются решениями системы дифференциальных уравнений

Дифференциальные уравнения

Из общего решения Дифференциальные уравнения следует, что можно принять Дифференциальные уравнения Дифференциальные уравненияДифференциальные уравнения Дифференциальные уравнения Тогда соответствующая система имеет вид:

Дифференциальные уравнения

При помощи первого действия, указанного справа от системы, получаем:

Дифференциальные уравнения
Дифференциальные уравнения

При помощи второго действия получаем: Дифференциальные уравнения т.е. Дифференциальные уравнения

Из первого уравнения системы, записанного в виде Дифференциальные уравненияДифференциальные уравнениянаходим Дифференциальные уравнения Отсюда Дифференциальные уравненияДифференциальные уравнения т.е. Дифференциальные уравненияТем самым Дифференциальные уравненияДифференциальные уравненияДифференциальные уравненияДифференциальные уравнения

Дифференциальные уравнения

Системы дифференциальных уравнений

1°. Систему дифференциальных уравнений первого порядка

Дифференциальные уравнения

назовем нормальной системой. Решить эту систему — означает найти n функций Дифференциальные уравненияпеременной х, удовлетворяющих системе.

Начальные условия, или условия Коши для системы дифференциальных уравнений представляют собой систему равенств (условий)

Дифференциальные уравнения

и выражают координаты точки (n + 1)-мерного пространства Дифференциальные уравнения.

2°. Нормальную систему можно привести к одному уравнению порядка n (или меньше) относительно одной неизвестной функции, например, Дифференциальные уравнения, при помощи следующего алгоритма.

1) Дифференцируем уравнение (1) системы по переменной х:

Дифференциальные уравнения

Производные Дифференциальные уравненияв правой части этого равенства заменим выражениями в правых частях уравнений (1)—(n) системы. Получим уравнение

Дифференциальные уравнения

2) Это равенство дифференцируем по переменной х:

Дифференциальные уравнения

Производные Дифференциальные уравнения в правой части этого равенства заменим их выражениями, заданными уравнениями (1)—(n).

Получим

Дифференциальные уравнения

Это уравнение дифференцируем по переменной х и т. д., пока не придем к уравнению

Дифференциальные уравнения

3) Полученные новые уравнения и уравнение (1) системы объединим в новую систему

Дифференциальные уравнения

4) Если первые (n — 1) уравнений этой системы разрешить относительно Дифференциальные уравнения, а полученные выражения подставить в последнее уравнение, то получим уравнение порядка n относительно неизвестной функции у:

Дифференциальные уравнения

5) Из общего решения этого уравнения можно получить общее решение системы или требуемое частное решение.

Заметим, что порядок последнего уравнения может быть меньше, чем n, если при его получении были использованы не все уравнения системы.

3°. Системы более общего вида, чем нормальные системы п. 1°, можно решать специфическими способами, которые непросто систематизировать и классифицировать, но решение некоторых таких систем будет продемонстрировано на конкретных примерах.

Примеры с решениями

Пример:

Решить систему

Дифференциальные уравнения

Найти частное решение, если x = 0, Дифференциальные уравненияДифференциальные уравнения

Решение:

Имеем дело с нормальной и к тому же линейной системой. Воспользуемся схемой п. 2°.

Дифференциальные уравнения

Первое уравнение продифференцировали (по переменной х, считающейся аргументом всех трех неизвестных функций системы), правую часть заменили на сумму правых частей второго и третьего уравнений, а затем сумму Дифференциальные уравнения заменили левой частью первого уравнения. Получили уравнение второго порядка относительно неизвестной функции Дифференциальные уравнения (при этом, очевидно, не использовали все операции, предусмотренные в п. 2°):

Дифференциальные уравнения

Его общее решение: Дифференциальные уравненияили просто (индекс оо опускаем) Дифференциальные уравнения

2) Для получения остальных функций постараемся выразить их через уже найденную и ее производные. Из второго уравнения системы вычитаем первое. Получаем

Дифференциальные уравнения

После замены

Дифференциальные уравнения

получаем новое уравнение Дифференциальные уравнения

Общим решением этого неоднородного уравнения, или линейного уравнения первого порядка

Дифференциальные уравнения
Дифференциальные уравнения

является функция Дифференциальные уравнения

3) Из первого уравнения Дифференциальные уравненияДифференциальные уравненияДифференциальные уравнения

Общее решение системы найдено, оно содержит три произвольных постоянных:

Дифференциальные уравнения
Дифференциальные уравнения

4) Начальные условия x = 0, Дифференциальные уравненияДифференциальные уравненияприводят к системе

Дифференциальные уравнения

Отсюда

Дифференциальные уравнения
Дифференциальные уравнения

Одним из специфических методов интегрирования систем дифференциальных уравнений является метод поиска интегрируемых выражений (комбинаций).

Пример:

Решить систему уравнений

Дифференциальные уравнения

Решение:

Сразу заметим, что неизвестные функции обозначены различными буквами, а аргументом будем считать переменную t.

Смысл метода заключается в том, чтобы перейти от данной системы к другой, равносильной, в которой можно было бы выделить дифференциальные группы, или полные дифференциалы. Например, если над уравнениями данной системы выполнить указанные ниже действия (умножения на соответствующие числа и последующее сложение результатов), то последовательно получим:

Дифференциальные уравнения

Именно в получении последнего равенства состояла наша цель. Преобразуем последнее равенство и интегрируем: Дифференциальные уравнения

Дифференциальные уравнения

Получили одно соотношение между неизвестными функциями, которое перепишем так: Дифференциальные уравнения

Составим новую систему, присоединяя к последнему равенству первое уравнение исходной системы:

Дифференциальные уравнения

Складывая (как обозначено на схеме) эти уравнения, получим линейное (неоднородное) уравнение первого порядка Дифференциальные уравнениякоторое можно решить методом подстановки:
х = uv, х’ = u’v + uv’. Получаем последовательно

Дифференциальные уравнения

Дифференциальные уравненияДифференциальные уравнения— это первая неизвестная функция системы.

Вторую неизвестную функцию у = y(t) находим из соотношения Дифференциальные уравненияДифференциальные уравнения

Дифференциальные уравнения

Пример:

Решить систему уравнений

Дифференциальные уравнения

Решение:

Данную систему способом, описанным выше в п. 2°, приведем к одному уравнению третьего порядка. При этом порядок использования того или иного уравнения может быть произвольным.

1) Продифференцируем (по t) первое уравнение системы:
х» = Зх’ — у’ + z’. В этом равенстве заменим разность z’ — у’, найденную из двух других уравнений: z’ — у’ = 2х — 6у + 4z.

Получаем уравнение второго порядка относительно х, но с посторонними неизвестными функциям у и z: х» = Зх’ + 2x — 6у + 4z.

2) Это равенство продифференцируем еще раз по
t: х'» = Зх» + 2х’ — 6 у’ + 4z’ и выразим в нем —6 у’ + 4 z’ через переменные х, у, z. Для этого необходимо произвести некоторые преобразования исходной системы: второе уравнение умножим на
— 6, третье на 4 и результаты сложим (получаем искомое выражение): — 6y’ + 4z’ = 10х — 34у + 18z.

Подставляя это выражение в предыдущее, получаем: х'» = Зх» + 2х’+ 10х — 34у + 18 z.

3) Составим теперь новую систему из трех уравнений: первое уравнение исходной системы; уравнение, полученное в конце действия 1); последнее уравнение, которое получили перед этим абзацем. Уравнения напишем так:

Дифференциальные уравнения

В этой системе также имеем посторонние неизвестные у и z, которые будем исключать следующим образом. Преобразуем первые два уравнения этой системы: сначала первое уравнение, умноженное на —6, сложим со вторым уравнением; а затем это же первое уравнение, умноженное на — 4, сложим со вторым уравнением. Получим два вспомогательных уравнения, которые объединим в систему:

Дифференциальные уравнения

в одном из них отсутствует у, а в другом отсутствует z.

4) Обратимся теперь к этой системе: первое ее уравнение умножим на 9, а второе умножим на -17, и полученные результаты сложим с последним уравнением системы, составленной в начале действия 3). Этим мы исключаем посторонние функции из уравнения третьего порядка относительно х: х'» — 11 х» + 36x’ — 36х = 0. Получаем искомое уравнение третьего порядка с одной неизвестной функцией х, которое решаем известным образом.

Характеристическое уравнение Дифференциальные уравнения имеет три простых корня: Дифференциальные уравнения а общее решение дифференциального уравнения имеет вид

Дифференциальные уравнения

5) Другие неизвестные функции у и z найдем из промежуточных уравнений (соотношений), полученных выше в действии 3). Отсюда, в частности, получаем

Дифференциальные уравнения

Остается подставить в эти равенства выражения для х и их производных (предлагаем это проделать самостоятельно).

Ответ

Дифференциальные уравнения

Примечание:

Предлагаем найти более экономный способ получения дифференциального уравнения из действия 4).

Пример:

Решить систему

Дифференциальные уравнения

Найти также частное решение системы, удовлетворяющее начальным условиям t = 0, x = 1, y = 2.Решение. Из системы получим одно уравнение второго порядка с одной неизвестной функцией. Сначала продифференцируем первое уравнение (по t ) и вместо y’ подставим выражение из второго уравнения: х» = 2х’ — 4у’, где
у’ = х — 3у. Получим х» = 2х’ — 4(х — 3у); вместо у подставим его выражение из первого уравнения: 4у = 2х — х’. Окончательно получим искомое уравнение х» + х’ — 2х = 0. Его решение имеет вид Дифференциальные уравнения

Функцию у получаем из первого же уравнения:

Дифференциальные уравнения
Дифференциальные уравнения

Частное решение получаем из общего, используя начальные условия t = 0, х = 1, у = 2.

Дифференциальные уравнения

Примечание:

Изданной системы можно получить два уравнения второго порядка: х» + х’ — 2х = 0 (его получили выше) и у» + у’ — 2у = 0 (по аналогии). Но пара общих решений Дифференциальные уравнения и Дифференциальные уравнения этих уравнений не составляет общее решение системы, поскольку здесь постоянные А, В, С, D — произвольные, а они должны быть согласованы с самой системой.

Другой эффективный способ решения систем линейных дифференциальных уравнений с постоянными коэффициентами принадлежит Л. Эйлеру и основан на использовании собственных чисел и собственных векторов матрицы данной системы.

Продемонстрируем этот метод на только что решенной системе.

Пример:

Решить систему дифференциальных уравнений

Дифференциальные уравнения

Решение:

Обозначим через Дифференциальные уравнения матрицу, составленную из коэффициентов правой части системы. Выпишем характеристическое уравнение матрицы

Дифференциальные уравнения

Собственными числами являются корни этого уравнения Дифференциальные уравнения и Дифференциальные уравнения Это означает, что выражения вида Дифференциальные уравнения (для х и для у) дают частные решения системы. Найдем какие-либо собственные векторы матрицы А, соответствующие этим числам,

Собственный вектор матрицы А, соответствующий собственному числу Дифференциальные уравнения — это произвольное нетривиальное решение однородной системы Дифференциальные уравнения где Е — единичная матрица, размерность которой равна размерности матрицы A, a U — вектор-столбец неизвестных.

В нашем случае

Дифференциальные уравнения

1) При Дифференциальные уравненияоднородная система Дифференциальные уравнения сводится

к одному уравнению Дифференциальные уравнения т.е. Дифференциальные уравнения

Положим, например, Дифференциальные уравнения; тогда Дифференциальные уравнениясобственный вектор, соответствующий Дифференциальные уравнения. Это означает, что Дифференциальные уравнения— частное решение системы.

2) Собственному числу Дифференциальные уравнения соответствует однородная система

Дифференциальные уравнения

или уравнение Дифференциальные уравненияположив Дифференциальные уравненияполучаем Дифференциальные уравненияи собственный вектор Дифференциальные уравнения Это означает, что Дифференциальные уравнения— другое частное решение системы.

Общее решение системы является линейной комбинацией ее частных решений с произвольными постоянными (здесь обозначим их через

Дифференциальные уравнения

Ответ

Дифференциальные уравнения

Примечание:

Обратим внимание на то, что полученное здесь общее решение внешне отличается от решения, полученного выше. На самом деле они совпадают, если принять Дифференциальные уравнения (или Дифференциальные уравненияДифференциальные уравнения

Пример:

Решить систему дифференциальных уравнений

Дифференциальные уравнения

Решение:

Характеристическое уравнение матрицы Дифференциальные уравнения

Дифференциальные уравнения
Дифференциальные уравнения

Отсюда

Дифференциальные уравнения

Это уравнение имеет три простых корня Дифференциальные уравненияа для матрицы А это три собственных числа.

Найдем теперь собственные векторы как решения однородной системы линейных уравнений Дифференциальные уравнениягде Е — единичная матрица третьего порядка, Дифференциальные уравнения— столбец из трех неизвестных.

1) Дифференциальные уравненияСоответствующая линейная однородная система имеет вид (после системы следуют ее преобразование и собственный вектор, координаты которого берем целыми числами):

Дифференциальные уравнения

принимаем Дифференциальные уравнения. Частное решение, соответствующее собственному числу Дифференциальные уравнения и собственному вектору Дифференциальные уравнения имеет вид:Дифференциальные уравненияДифференциальные уравнения

2) Дифференциальные уравнения Имеем:

Дифференциальные уравнения

принимаем Дифференциальные уравнения. Выписываем тройку функций — частное решение системы:

Дифференциальные уравнения

3) Дифференциальные уравнения Имеем:

Дифференциальные уравнения

Принимаем Дифференциальные уравненияЧастное решение, соответствующее собственному числу Дифференциальные уравнения имеет вид:

Дифференциальные уравнения

Составим общее решение исходной системы.

Ответ

Дифференциальные уравнения

Пример:

Решить систему

Дифференциальные уравнения

Решение:

Приведем краткое схематическое решение для закрепления методики собственных чисел и собственных векторов.

Дифференциальные уравнения
  1. Характеристическое уравнение матрицы
Дифференциальные уравнения

имеет три простых корня: Дифференциальные уравнения

2) Для каждого собственого числа находим какой-либо собственный вектор матрицы А и соответствующее ему частное решение системы.

а) Для Дифференциальные уравнения можно выбрать собственный вектор (-1, 0, 1) и соответствующее ему частное решение

Дифференциальные уравнения

б) Для Дифференциальные уравненияможно выбрать собственный вектор (1, 1, 1) и соответствующее ему частное решение

Дифференциальные уравнения

в) Для Дифференциальные уравнения можно выбрать собственный вектор (—1, 2, —1) и соответствующее ему частное решение

Дифференциальные уравнения

в) Составим общее решение системы как линейную комбинацию полученных частных решений.

Ответ

Дифференциальные уравнения

Для решения систем дифференциальных уравнений более общего вида необходимо использовать более тонкие приемы их анализа и комбинирования.

Пример:

Проинтегрировать систему дифференциальных уравнений

Дифференциальные уравнения

Решение:

В данной системе неизвестные функции х и у зависят от аргумента t.

1) Обе части второго уравнения умножим на Дифференциальные уравнения

Дифференциальные уравнения

Это равенство сложим с первым уравнением системы и с тождеством Дифференциальные уравнения Получаем:

Дифференциальные уравнения

Заметим, что правая часть этого равенства равна нулю. Левая же ее часть есть сумма производных: Дифференциальные уравненияТаким образом, получили нетривиальную комбинациюДифференциальные уравненияоткуда Дифференциальные уравнениячто представляет собой часть общего решения системы (имеем только одну постоянную).

2) Другую, аналогичную комбинацию получим не менее «творческим» способом. Обе части первого уравнения умножим на Дифференциальные уравнения обе части второго уравнения умножим на Дифференциальные уравненияполученные результаты сложим, и к ним еще прибавим равенство t’ = 1 (производная аргумента равна единице). Имеем:

Дифференциальные уравнения

Правая часть равна нулю, а в левой

Дифференциальные уравнения

3) Общий интеграл (общее решение в неявном виде) получаем из системы полученных таким образом равенств.

Ответ Дифференциальные уравнения

Пример:

Проинтегрировать систему уравнений

Дифференциальные уравнения

Решение:

Перепишем систему в другой форме

Дифференциальные уравнения
Дифференциальные уравнения

Сложение и вычитание уравнений этой системы позволяют получить искомые неизвестные функции.

Ответ

Дифференциальные уравнения

Дифференциальные уравнения с подробным объяснением и теорией

При решении различных задач математики, физики, химии и других наук часто пользуются математическими моделями в виде уравнений, связывающих независимую переменную, искомую функцию и ее производные. Такие уравнения называются дифференциальными (термин принадлежит Г. Лейбницу, 1676 г.). Решением дифференциального уравнения называется функция, которая при подстановке в уравнение обращает его в тождество.

Так, решением уравнения Дифференциальные уравнения является функция у = F(x) — первообразная для функции f(x).

Рассмотрим некоторые общие сведения о дифференциальных уравнениях (ДУ).

Если искомая (неизвестная) функция зависит от одной переменной, то ДУ называют обыкновенным; в противном случае — ДУ в частных производных. Далее будем рассматривать только обыкновенные ДУ.

Наивысший порядок производной, входящей в ДУ, называется порядком этого уравнения.

Например, уравнение у'» — 3у» + 2у = 0 — обыкновенное ДУ третьего порядка, а уравнение Дифференциальные уравнения — первого порядка; Дифференциальные уравнения— ДУ в частных производных первого порядка.

Процесс отыскания решения ДУ называется его интегрированием, а график решения ДУ — интегральной кривой.

Рассмотрим некоторые задачи, решение которых приводит к дифференциальным уравнениям.

Задачи, приводящие к дифференциальным уравнениям

Задача:

Материальная точка массы m замедляет свое движение под действием силы сопротивления среды, пропорциональной квадрату скорости V. Найти зависимость скорости от времени. Найти скорость точки через 3 с после начала замедления, если V (0) = 100 м/с, a V(l) = 50 м/с.

Решение:

Примем за независимую переменную время t, отсчитываемое от начала замедления движения материальной точки. Тогда скорость точки V будет функцией t, т. е. V = V(t). Для нахождения V(t) воспользуемся вторым законом Ньютона (основным законом механики): Дифференциальные уравнения — есть ускорение движущегося тела, F — результирующая сила, действующая на тело в процессе движения.

В данном случае Дифференциальные уравнения— коэффициент пропорциональности (знак минус указывает на то, что скорость тела уменьшается). Следовательно, функция V = V(t) является решением дифференциального уравнения Дифференциальные уравнения или Дифференциальные уравнения Здесь m — масса тела.

Как будет показано ниже (пример 48.5), Дифференциальные уравнения где с —const. Найдя зависимость скорости от времени, легко найти скорость точки через 3 с после начала замедления.

Найдем сначала параметры Дифференциальные уравнения Согласно условию задачи, имеем: Дифференциальные уравнения Отсюда Дифференциальные уравнения

Следовательно, скорость точки изменяется по закону Дифференциальные уравнения Поэтому V(3) = 25 м/с.

Задача:

Найти кривую, проходящую через точку (4;1), зная, что отрезок любой касательной к ней, заключенный между осями координат, делится в точке касания пополам.

Решение:

Пусть М(х; у) — произвольная точка кривой, уравнение которой у = f(х). Для определенности предположим, что кривая расположена в первой четверти (см. рис. 212).

Дифференциальные уравнения

Для составления дифференциального уравнения воспользуемся геометрическим смыслом первой производной: Дифференциальные уравненияесть угловой коэффициент касательной; в точке М(х; у) он равен у’, т. е. Дифференциальные уравнения Из рисунка видно, что Дифференциальные уравнения Но

Дифференциальные уравнения

Дифференциальные уравненияПо условию задачи AM = MB, следовательно, ОС =С В = x.

Таким образом, получаем Дифференциальные уравненияРешением полученного дифференциального уравнения является функция Дифференциальные уравнения (гипербола). Решение будет приведено в п. 48.2 (пример 48.4).

Другие задачи

Можно показать, что:

• закон изменения массы радия в зависимости от времени («радиоактивный распад») описывается дифференциальным уравнением Дифференциальные уравнения— коэффициент пропорциональности, m(t) — масса радия в момент t.

• «закон охлаждения тел», т. е. закон изменения температуры тела в зависимости от времени, описывается уравнением Дифференциальные уравнениягде T(t) — температура тела в момент времени t, k — коэффициент пропорциональности, Дифференциальные уравнения— температура воздуха (среды охлаждения);

• зависимость массы х вещества, вступившего в химическую реакцию, от времени t во многих случаях описывается уравнением Дифференциальные уравнения, где k— коэффициент пропорциональности;

• «закон размножения бактерий» (зависимость массы т бактерий от времени t) описывается уравнением Дифференциальные уравнения

• закон изменения давления воздуха в зависимости от высоты над уровнем моря описывается уравнением Дифференциальные уравнения — атмосферное давление воздуха на высоте h, k > 0.

Уже приведенные примеры указывают на исключительно важную роль дифференциальных уравнений при решении самых разнообразных задач.

Дифференциальные уравнения первого порядка

Дифференциальное уравнение первого порядка в общем случае можно записать в виде

Дифференциальные уравнения

Уравнение связывает независимую переменную х, искомую функцию у и ее производную у’. Если уравнение (48.1) можно разрешить относительно у’, то его записывают в виде

Дифференциальные уравнения

и называют ДУ первого порядка, разрешенным относительно производной. Мы в основном будем рассматривать эту форму записи ДУ.

Уравнение (48.2) устанавливает связь (зависимость) между координатами точки (х; у) и угловым коэффициентом у’ касательной к интегральной кривой, проходящей через эту точку. Следовательно, ДУ у’ = f(x;y) дает совокупность направлений (поле направлений) на плоскости Оху. Таково геометрическое истолкование ДУ первого порядка.

Кривая, во всех точках которой направление поля одинаково, называется изоклиной. Изоклинами можно пользоваться для приближенного построения интегральных кривых. Уравнение изоклины можно получить, если положить у’ = с, т. е. f(х; у) = с.

Пример:

С помощью изоклин начертить вид интегральных кривых уравнения у’ = 2х.

Решение:

Уравнение изоклин этого ДУ будет 2х = с, т. е. изоклинами здесь будут прямые, параллельные оси Дифференциальные уравненияВ точках прямых проведем отрезки, образующие с осью Ох один и тот же угол а, тангенс которого равен с.

Так, при с = 0 имеем х = 0,Дифференциальные уравненияпоэтому а = 0;

при с = 1 уравнение изоклины Дифференциальные уравнения поэтому Дифференциальные уравнения
при с = -1: Дифференциальные уравнения

при с = 2: Дифференциальные уравнения

Построив четыре изоклины и отметив на каждой из них ряд стрелочек, наклоненных к оси Ох под определенным углом (см. рис. 213), по их направлениям строим линии. Они, как видно, представляют собой семейство парабол.

Дифференциальные уравнения

Дифференциальное уравнение первого порядка, разрешенное относительно производной, можно записать в дифференциальной форме:

Дифференциальные уравнения

где P(x;y) и Q(x;y) — известные функции. Уравнение (48.3) удобно тем, что переменные х и у в нем равноправны, т. е. любую из них можно рассматривать как функцию другой. Отметим, что от одного вида записи ДУ можно перейти к другому.

Интегрирование ДУ в общем случае приводит к бесконечному множеству решений (отличающихся друг от друга постоянными величинами). Легко догадаться, что решением уравнения у’ = 2х является функция Дифференциальные уравнения, а также Дифференциальные уравнения и вообще Дифференциальные уравнения где с — const.

Чтобы решение ДУ приобрело конкретный смысл, его надо подчинить некоторым дополнительным условиям.

Условие, что при Дифференциальные уравнения функция у должна быть равна заданному числу Дифференциальные уравнения, т. е. Дифференциальные уравнения называется начальным условием. Начальное условие записывается в виде

Дифференциальные уравнения

Общим решением ДУ первого порядка называется функция Дифференциальные уравнения, содержащая одну произвольную постоянную и удовлетворяющая условиям:

  1. Функция Дифференциальные уравнения является решением ДУ при каждом фиксированном значении с.
  2. Каково бы ни было начальное условие (48.4), можно найти такое значение постоянной Дифференциальные уравнения, что функция Дифференциальные уравненияудовлетворяет данному начальному условию.

Частным решением ДУ первого порядка называется любая функция Дифференциальные уравнения, полученная из общего решения Дифференциальные уравненияпри конкретном значении постоянной Дифференциальные уравнения.

Если общее решение ДУ найдено в неявном виде, т. е. в виде уравнения Дифференциальные уравнения, то такое решение называется общим интегралом ДУ. Уравнение Дифференциальные уравнения в этом случае называется частным интегралом уравнения.

С геометрической точки зрения Дифференциальные уравненияесть семейство интегральных кривых на плоскости Оху; частное решениеДифференциальные уравнения — одна кривая из этого семейства, проходящая через точкуДифференциальные уравнения. Задача отыскания решения ДУ первого порядка (48.3), удовлетворяющего заданному начальному условию (48.4), называется задачей Коши.

Теорема:

Существования и единственности решения задачи Коши. Если в уравнении (48.2) функция f(х; у) и ее частная производная Дифференциальные уравнения непрерывны в некоторой области D, содержащей точку Дифференциальные уравнения, то существует единственное решение Дифференциальные уравнения этого уравнения, удовлетворяющее начальному условию (48.4).

(Без доказательства).

Геометрический смысл теоремы состоит в том, что при выполнении ее условий существует единственная интегральная кривая ДУ, проходящая через точку Дифференциальные уравнения.

Рассмотрим теперь методы интегрирования ДУ первого порядка определенного типа.

Уравнения с разделяющимися переменными

Наиболее простым ДУ первого порядка является, уравнение вида

Дифференциальные уравнения

В нем одно слагаемое зависит только от х, а другое — от у. Иногда такие ДУ называют уравнениями с разделенными переменными. Проинтегрировав почленно это уравнение, получаем:

Дифференциальные уравнения

— его общий интеграл.

Пример:

Найти общий интеграл уравнения Дифференциальные уравненияРешение: Данное уравнение есть ДУ с разделенными переменными. Поэтому

Дифференциальные уравнения

Обозначим Дифференциальные уравненияТогда Дифференциальные уравнения— общий интеграл ДУ.

Более общий случай описывают уравнения с разделяющимися переменными, которые имеют вид

Дифференциальные уравнения

Особенность уравнения (48.6) в том, что коэффициенты при dx и dy представляют собой произведения двух функций (чисел), одна из которых зависит только от х, другая — только от у.

Уравнение (48.6) легко сводится к уравнению (48.5) путем почленного деления его на Дифференциальные уравнения Получаем:

Дифференциальные уравнения

— общий интеграл.

Замечания. 1. При проведении почленного деления ДУ на Дифференциальные уравнения могут быть потеряны некоторые решения. Поэтому следует отдельно решить уравнение Дифференциальные уравнения и установить те решения ДУ, которые не могут быть получены из общего решения, — особые решения.

2.Уравнение Дифференциальные уравнения также сводится к уравнению с разделенными переменными. Для этого достаточно положить Дифференциальные уравненияи разделить переменные.

Уравнение Дифференциальные уравнения где а, b, с — числа, путем замены ах + by + с = и сводится к ДУ с разделяющимися переменными. Дифференцируя по х, получаем:

Дифференциальные уравнения

откуда следует

Дифференциальные уравнения

Интегрируя это уравнение и заменяя и на ах + by + с, получим общий интеграл исходного уравнения.

Пример:

Решить уравнение

Дифференциальные уравнения

Решение:

Преобразуем левую часть уравнения:

Дифференциальные уравнения

Оно имеет вид (48.6). Делим обе части уравнения на Дифференциальные уравнения

Дифференциальные уравнения

Решением его является общий интеграл

Дифференциальные уравнения

Здесь уравнение Дифференциальные уравнения имеет вид ху = 0. Его решения х = 0, у = 0 являются решениями данного ДУ, но не входят в общий интеграл. Значит, решения х = 0, у = 0 являются особыми.

Пример:

Решить уравнение Дифференциальные уравнения удовлетворяющее условию у(4) = 1.

Решение:

Этот пример представляет собой решение задачи 2 из п. 47.2.

Имеем: Дифференциальные уравненияПроинтегрировав, получим:

Дифференциальные уравнения

т.е. Дифференциальные уравненияобщее решение ДУ.

Оно представляет собой, геометрически, семейство равносторонних гипербол. Выделим среди них одну, проходящую через точку (4; 1). Подставим х = 4 и у = 1 в общее решение уравнения: Дифференциальные уравнения

Получаем: Дифференциальные уравнения частное решение уравнения Дифференциальные уравнения

Пример:

Найти общее решение ДУ Дифференциальные уравнения

Решение:

Этот пример демонстрирует решение задачи 1 из п. 47.2. Приведем данное уравнение к виду (48.5):

Дифференциальные уравнения

Интегрируем:

Дифференциальные уравнения

Отсюда Дифференциальные уравнения-общее решение уравнения.

Однородные дифференциальные уравнения

К уравнению с разделяющимися переменными приводятся однородные ДУ первого порядка.

Функция f(x,y) называется однородной функцией п-го порядка (измерения), если при умножении каждого ее аргумента на произвольный множитель Дифференциальные уравнения вся функция умножится на Дифференциальные уравнения, т. е.

Дифференциальные уравнения

Например, функция Дифференциальные уравнения есть однородная функция второго порядка, поскольку

Дифференциальные уравнения

Дифференциальное уравнение

Дифференциальные уравнения

называется однородным, если функция f(x; у) есть однородная функция нулевого порядка.

Покажем, что однородное ДУ (48.7) можно записать в виде

Дифференциальные уравнения

Если f(x; у) — однородная функция нулевого порядка, то, по определению, Дифференциальные уравненияПоложив Дифференциальные уравнения, получаем:

Дифференциальные уравнения

Однородное уравнение (48.8) преобразуется в уравнение с разделяющимися переменными при помощи замены переменной (подстановки)

Дифференциальные уравнения

Действительно, подставив у = их и у’ = и’х + и в уравнение (48.8), получаем Дифференциальные уравнения т. е. уравнение с разделяющимися переменными. Найдя его общее решение (или общий интеграл), следует заменить в нем и на Дифференциальные уравнения. Получим общее решение (интеграл) исходного уравнения.

Однородное уравнение часто задается в дифференциальной форме:

Дифференциальные уравнения

ДУ (48.10) будет однородным, если Р(х; у) и Q (х; у) — однородные функции одинакового порядка.

Переписав уравнение (48.10) в виде Дифференциальные уравнения и применив в правой части рассмотренное выше преобразование, получим уравнение Дифференциальные уравнения

При интегрировании уравнений вида (48.10) нет необходимости предварительно приводить их (но можно) к виду (48.8): подстановка (48.9) сразу преобразует уравнение (48.10) в уравнение с разделяющимися переменными.

Пример:

Найти общий интеграл уравнения

Дифференциальные уравнения

Решение:

Данное уравнение однородное, т. к. функции Дифференциальные уравнения— однородные функции второго порядка.

Положим Дифференциальные уравненияТогда Дифференциальные уравненияПодставляем в исходное уравнение:

Дифференциальные уравнения

последнее — уравнение с разделяющимися переменными. Делим переменные

Дифференциальные уравнения

и интегрируем

Дифференциальные уравнения

Обозначим Дифференциальные уравнения Тогда

Дифференциальные уравнения

Заменяя и на Дифференциальные уравнения получаем: Дифференциальные уравнения— общий интеграл исходного уравнения.

Отметим, что данное уравнение можно было сначала привести к виду (48.8):

Дифференциальные уравнения

Затем положить Дифференциальные уравнения тогда Дифференциальные уравнения

Замечание. Уравнение вида Дифференциальные уравнения Где Дифференциальные уравнения— числа, приводится к однородному или с разделяющимися переменными. Для этого вводят новые переменные и и v, положив Дифференциальные уравнения — числа. Их подбирают так, чтобы уравнение стало однородным.

Пример:

Найти общий интеграл уравнения

Дифференциальные уравнения

т.е. Дифференциальные уравнения

Решение:

Положив Дифференциальные уравнения получаем:

Дифференциальные уравнения

Подберем Дифференциальные уравнениятак, чтобы

Дифференциальные уравнения

Находим, что Дифференциальные уравнения Заданное уравнение примет вид

Дифференциальные уравнения

и будет являться однородным. Его решение получается, как это было показано выше, при помощи подстановки v = tu. Заметим, что, решив его, следует заменить u и v соответственно на x— 1 и у + 1. В итоге получим Дифференциальные уравнения— общий интеграл данного уравнения.

Линейные уравнения. Уравнение Я. Бернулли

Дифференциальное уравнение первого порядка называется линейным, если его можно записать в виде

Дифференциальные уравнения

где р(х) и g(х) — заданные функции, в частности — постоянные.

Особенность ДУ (48.11): искомая функция у и ее производная у’ входят в уравнение в первой степени, не перемножаясь между собой. Рассмотрим два метода интегрирования ДУ (48.11) — метод И. Бернулли и метод Лагранжа.

Метод И. Бернулли

Решение уравнения (48.11) ищется в виде произведения двух других функций, т. е. с помощью подстановкиДифференциальные уравнения— неизвестные функции от х, причем одна из них произвольна (но не равна нулю — действительно любую функцию у(х) можно записать как

Дифференциальные уравнения

где Дифференциальные уравнения). Тогда Подставляя Дифференциальные уравнениявыражения у и у’ в уравнение (48.11), получаем: Дифференциальные уравнения или

Дифференциальные уравнения

Подберем функцию v = v(x) так, чтобы выражение в скобках было равно нулю, т. е. решим ДУ

Дифференциальные уравнения

Интегрируя, получаем:

Дифференциальные уравнения

Ввиду свободы выбора функции v(x), можно принять с = 1. Отсюда

Дифференциальные уравнения

Подставляя найденную функцию v в уравнение (48.12), получаем

Дифференциальные уравнения

Получено уравнение с разделяющимися переменными. Решаем его:

Дифференциальные уравнения

Возвращаясь к переменной у, получаем решение

Дифференциальные уравнения

исходного ДУ (48.11).

Пример:

Проинтегрировать уравнение Дифференциальные уравнения

Решение:

Полагаем Дифференциальные уравненияТогда Дифференциальные уравнения Дифференциальные уравнения Сначала решаем уравнение

Дифференциальные уравнения

Теперь решаем уравнение Дифференциальные уравнения

Дифференциальные уравнения

Итак, общее решение данного уравнения есть

Дифференциальные уравнения

Метод Лагранжа (метод вариации произвольной постоянной)

Уравнение (48.11) интегрируется следующим образом. Рассмотрим соответствующее уравнение без правой части, т. е. уравнение Дифференциальные уравнения Оно называется линейным однородным ДУ первого порядка. В этом уравнении переменные делятся:

Дифференциальные уравнения

Таким образом, Дифференциальные уравнения

Дифференциальные уравнения

Метод вариации произвольной постоянной состоит в том, что постоянную с в полученном решении заменяем функцией с(х), т. е. полагаем с = с(х). Решение уравнения (48.11) ищем в виде

Дифференциальные уравнения

Находим производную:

Дифференциальные уравнения

Подставляем значения у и у’ в уравнение (48.11):

Дифференциальные уравнения

Второе и третье слагаемые взаимно уничтожаются, и уравнение примет вид

Дифференциальные уравнения

Следовательно,

Дифференциальные уравнения

Интегрируя, находим:

Дифференциальные уравнения

Подставляя выражение c(x) в равенство (48.14), получим общее решение ДУ (48.11):

Дифференциальные уравнения

Естественно, та же формула была получена методом Бернулли (ср. с (48.13)).

Пример:

Решить пример 48.8 методом Лагранжа.

Решение:

Решаем уравнение у’ + 2ху = 0. ИмеемДифференциальные уравнения или Дифференциальные уравнения

Заменяем с на c(х), т. е. решение ДУ у’ + 2ху = 2х ищем в виде Дифференциальные уравнения Имеем

Дифференциальные уравнения

Тогда

Дифференциальные уравнения

или

Дифференциальные уравнения

Поэтому Дифференциальные уравненияили Дифференциальные уравнения — общее решение данного уравнения.

Замечание:

Уравнение вида

Дифференциальные уравнения

— заданные функции, можно свести к линейному, если х считать функцией, а у — аргументом: х = х(у). Тогда, пользуясь равенством Дифференциальные уравнения получаем

Дифференциальные уравнения

— линейное относительно х уравнение. Его решение ищем в виде Дифференциальные уравнения — две неизвестные функции.

Пример:

Найти общее решение уравнения Дифференциальные уравнения

Решение:

Учитывая, что Дифференциальные уравнения от исходного уравнения переходим к линейному уравнению Дифференциальные уравнения

Применим подстановку Дифференциальные уравнения Тогда Дифференциальные уравненияПолучаем:

Дифференциальные уравнения

Находим функцию Дифференциальные уравнения

Находим функцию

Дифференциальные уравнения

Интегрируя по частям, находим: Дифференциальные уравнения

Значит, общее решение данного уравнения:

Дифференциальные уравнения

или Дифференциальные уравнения

Уравнение Я. Бернулли

Уравнение вида

Дифференциальные уравнения

называется уравнением Бернулли. Покажем, что его можно привести к линейному.

Если п = 0, то ДУ (48.15) — линейное, а при п =1 — с разделяющимися переменными.

В общем случае, разделив уравнение (48.15) на Дифференциальные уравнения получим:

Дифференциальные уравнения

Обозначим Дифференциальные уравнения Тогда Дифференциальные уравненияОтсюда находим Дифференциальные уравнения Уравнение (48.16) принимает вид

Дифференциальные уравнения

Последнее уравнение является линейным относительно z. Решение его известно. Таким образом, подстановкаДифференциальные уравнения сводит уравнение (48.15) к линейному. На практике ДУ (48.15) удобнее искать методом И. Бернулли в виде Дифференциальные уравнения (не сводя его к линейному).

Уравнение в полных дифференциалах. Интегрирующий множитель

Уравнение

Дифференциальные уравнения

называется уравнением в полных дифференциалах, если его левая часть есть полный дифференциал некоторой функции и(х; у), т. е.

Дифференциальные уравнения

В этом случае ДУ (48.17) можно записать в виде du(x; у) = 0, а его общий интеграл будет:

Дифференциальные уравнения

Приведем условие, по которому можно судить, что выражение

Дифференциальные уравнения

есть полный дифференциал.

Теорема 48.2. Для того чтобы выражение Дифференциальные уравнения

где функции Р(х; у) и Q(x; y) и их частные производные Дифференциальные уравнениянепрерывны в некоторой области D плоскости Оху, было полным дифференциалом, необходимо и достаточно выполнение условия

Дифференциальные уравнения

Необходимость

Пусть Дифференциальные уравнения есть полный дифференциал, т. е.

Дифференциальные уравнения

Учитывая, что Дифференциальные уравнения (см. п. 44.3), имеем:

Дифференциальные уравнения

Дифференцируя эти равенства по у и по х соответственно, получаем

Дифференциальные уравнения

А так как смешанные частные производные Дифференциальные уравнения Равны между собой (см. п. 44.2), получаем (48.19).

Достаточность

Пусть в области D выполняется условие (48.19). Покажем, что существует функция и(х; у) в области D такая, что

Дифференциальные уравнения

Найдем эту функцию. Искомая функция должна удовлетворять требованиям:

Дифференциальные уравнения

Если в первом уравнении (48.20) зафиксировать у и проинтегрировать его по х, то получим:

Дифференциальные уравнения

Здесь произвольная постоянная Дифференциальные уравнения зависит от у (либо является числом). В решении (48.21) не известна лишь Дифференциальные уравнения. Для ее нахождения продифференцируем функцию (48.21) по у:

Дифференциальные уравнения

Используя второе равенство (48.20), можно записать:

Дифференциальные уравнения

Отсюда

Дифференциальные уравнения

В равенстве (48.22) левая часть зависит от у. Покажем, что и правая часть равенства зависит только от у.

Для этого продифференцируем правую часть по и убедимся, что производная равна нулю. Действительно,

Дифференциальные уравнения

Из равенства (48.22) находим Дифференциальные уравнения:

Дифференциальные уравнения

Подставляя найденное значение для Дифференциальные уравнения в равенство (48.21), находим функцию и(х; у) такую, что du ( x ; у) = Р(х; у) dx + <3(х; у) dy.

Таким образом, при решении ДУ вида (48.17) сначала проверяем выполнение условия (48.19). Затем, используя равенства (48.20), находим функцию и(х;у). Решение записываем в виде (48.18).

Пример:

Решить уравнение Дифференциальные уравнения

Решение:

Запишем уравнение в дифференциальной форме:

Дифференциальные уравнения

Здесь Дифференциальные уравнения Проверяем выполнение условия (48.19):

Дифференциальные уравнения

Следовательно, данное уравнение есть уравнение в полных дифференциалах. Условия (48.20) будут здесь выглядеть как

Дифференциальные уравнения

Отсюда имеем

Дифференциальные уравнения

Далее

Дифференциальные уравнения

Общим интегралом является

Дифференциальные уравнения

Если условие (48.19) не выполняется, то ДУ (48.17) не является уравнением в полных дифференциалах.

Однако это уравнение иногда можно привести к уравнению в полных дифференциалах умножением его на некоторую функцию t(х; у), называемую интегрирующим множителем.

Чтобы уравнение Дифференциальные уравнения было уравнением в полных дифференциалах, должно выполняться условие

Дифференциальные уравнения

Выполнив дифференцирование Дифференциальные уравненияи приведя подобные слагаемые, получим

Дифференциальные уравнения

Для нахождения t(x;y) надо проинтегрировать полученное ДУ в частных производных. Решение этой задачи не простое. Нахождение интегрирующего множителя может быть упрощено, если допустить существование t как функции только одного аргумента х либо только у. Пусть, например, t = t(x). Тогда уравнение (48.23) принимает вид

Дифференциальные уравнения

Отсюда

Дифференциальные уравнения

При этом выражение Дифференциальные уравнениядолжно зависеть только от х. Аналогично получаем, что если t = t(y) (t не зависит от х), то

Дифференциальные уравнения

а подынтегральное выражение должно зависеть только от у.

Пример:

Решить уравнение Дифференциальные уравнения

Решение: Здесь

Дифференциальные уравнения

Однако

Дифференциальные уравнения

зависит только от х.

Следовательно, уравнение имеет интегрирующий множитель, зависящий только от х, выражение которого может быть получено при помощи формулы (48.24). В нашем случае получим, что

Дифференциальные уравнения

Умножая исходное уравнение на Дифференциальные уравнения получаем:

Дифференциальные уравнения

т.е. уравнение в полных дифференциалах! Решив его, найдем, что общий интеграл заданного уравнения имеет вид

Дифференциальные уравнения

Уравнения Лагранжа и Клеро

Рассмотрим дифференциальные уравнения, неразрешенные относительно производной. К ним, в частности, относятся уравнения Лагранжа и Клеро.

Уравнение Лагранжа

Уравнение вида

Дифференциальные уравнения

где Дифференциальные уравнения — известные функции от Дифференциальные уравненияназывается уравнением Лагранжа.

Введем вспомогательный параметр, положив у’ = р. Тогда уравнение (48.25) примет вид

Дифференциальные уравнения

Дифференцируя по х, получим:

Дифференциальные уравнения

Уравнение (48.27) есть линейное уравнение относительно неизвестной функции х = х(р). Решив его, найдем:

Дифференциальные уравнения

Исключая параметр р из уравнений (48.26) и (48.28), получаем общий интеграл уравнения (48.25) в виде Дифференциальные уравнения

Отметим, что, переходя к уравнению (48.27), мы делили на Дифференциальные уравненияПри этом могли быть потеряны решения, для которых Дифференциальные уравнения т. е. Дифференциальные уравнения Это значение Дифференциальные уравнения является корнем уравненияДифференциальные уравнения (см. (48.27)).

Решение Дифференциальные уравнения является особым для уравнения (48.25) (см. понятие особого решения в п. 48.2).

Уравнение Клеро:

Рассмотрим частный случай уравнения Лагранжа при Дифференциальные уравнения

Уравнение (48.25) принимает вид

Дифференциальные уравнения

и называется уравнением Клеро. Положив у’ = р, получаем:

Дифференциальные уравнения

Дифференцируя по х, имеем:

Дифференциальные уравнения

Если Дифференциальные уравнения Поэтому, с учетом (48.30), ДУ (48.29) имеет общее решение

Дифференциальные уравнения

Если Дифференциальные уравнения то получаем частное решение уравнения в параметрической форме:

Дифференциальные уравнения

Это решение — особое решение уравнения Клеро: оно не содержится в формуле общего решения уравнения.

Пример:

Решить уравнение Клеро Дифференциальные уравнения

Решение:

Общее решение, согласно формуле (48.31), имеет вид Дифференциальные уравненияОсобое решение уравнения получаем согласно формулам (48.32) в виде Дифференциальные уравнения Отсюда следует:

Дифференциальные уравнения

Дифференциальные уравнения высших порядков

Дифференциальные уравнения порядка выше первого называются ДУ высших порядков. ДУ второго порядка в общем случае записывается в виде

Дифференциальные уравнения

или, если это возможно, в виде, разрешенном относительно старшей производной:

Дифференциальные уравнения

Будем в основном рассматривать уравнение вида (49.2): от него всегда можно перейти к (49.1).

Решением ДУ (49.2) называется всякая функция Дифференциальные уравнения, которая при подстановке в уравнение обращает его в тождество.

Общим решением ДУ (49.2) называется функция Дифференциальные уравнениягде Дифференциальные уравнения— не зависящие от х произвольные постоянные, удовлетворяющая условиям:

  1. Дифференциальные уравнения является решением ДУ для каждого фиксированного значения Дифференциальные уравнения.
  2. Каковы бы ни были начальные условия
Дифференциальные уравнения

существуют единственные значения постоянных Дифференциальные уравнениятакие, что функция Дифференциальные уравнения является решением уравнения (49.2) и удовлетворяет начальным условиям (49.3).

Всякое решение Дифференциальные уравнения уравнения (49.2), получающееся из общего решения Дифференциальные уравнения при конкретных значениях постоянных Дифференциальные уравнения, называется частным решением. Решения ДУ (49.2), записанные в виде

Дифференциальные уравнения

называются общим и частным интегралом соответственно.

График всякого решения ДУ второго порядка называется интегральной кривой. Общее решение ДУ (49.2) представляет собой множество интегральных кривых; частное решение — одна интегральная кривая этого множества, проходящая через точкуДифференциальные уравнения и имеющая в ней касательную с заданным угловым коэффициентом Дифференциальные уравнения Переписав ДУ (49.1) в виде

Дифференциальные уравнения

видим, что ДУ второго порядка устанавливает связь между координатами точки (х; у) интегральной кривой, угловым коэффициентом k = у’ касательной к ней и кривизной Дифференциальные уравнения в точке (х; у). В этом состоит геометрическое истолкование ДУ второго порядка.

Как и в случае уравнения первого порядка, задача нахождения решения ДУ (49.2), удовлетворяющего заданным начальным условиям (49.3), называется задачей Коши.

Теорема:

Существования и единственности задачи Коши. Если в уравнении (49.2) функция f(x;y;y’) и ее частные производные Дифференциальные уравнения непрерывны в некоторой области D изменения переменных х, у и у’, то для всякой точки Дифференциальные уравнения существует единственное решение Дифференциальные уравнения уравнения (49.2), удовлетворяющее начальным условиям (49.3).

Примем теорему без доказательства.

Аналогичные понятия и определения имеют место для ДУ п-го порядка, которое в общем виде записывается ка

Дифференциальные уравнения

или

Дифференциальные уравнения

если его можно разрешить относительно старшей производной. Начальные условия для ДУ (49.4) имеют вид

Дифференциальные уравнения

Общее решение ДУ n-го порядка является функцией вида

Дифференциальные уравнения

содержащей п произвольных, не зависящих от х постоянных.

Решение ДУ (49.4), получающееся из общего решения при конкретных значениях постоянных Дифференциальные уравнения называется частным решением.

Задача Коши для ДУ n-го порядка: найти решение ДУ (49.4), удовлетворяющее начальным условиям (49.5).

Проинтегрировать (решить) ДУ n-го порядка означает следующее: найти его общее или частное решение (интеграл) в зависимости от того, заданы начальные условия или нет.

Задача нахождения решения ДУ n-го порядка сложнее, чем первого. Поэтому рассмотрим лишь отдельные виды ДУ высших порядков.

Уравнения, допускающие понижение порядка

Одним из методов интегрирования ДУ высших порядков является метод понижения порядка. Суть метода состоит в том, что с помощью замены переменной (подстановки) данное ДУ сводится к уравнению, порядок которого ниже.

Рассмотрим три типа уравнений, допускающих понижение порядка.

I. Пусть дано уравнение

Дифференциальные уравнения

Порядок можно понизить, введя новую функцию р(х), положив у’ = р(х). Тогда у» = р'(х) и получаем ДУ первого порядка: р’ = f(x). Решив его, т. е. найдя функцию р = р(х), решим уравнение у’ = р(х). Получим общее решение заданного уравнения (49.6).

На практике поступают иначе: порядок понижается непосредственно путем последовательного интегрирования уравнения.

Так как Дифференциальные уравнения уравнение (49.6) можно записать в виде dy’ = f(x)dx. Тогда, интегрируя уравнение у» = f(x), получаем:

Дифференциальные уравнения

Далее, интегрируя полученное уравнение по х, находим: Дифференциальные уравнения — общее решение данного уравнения. Если дано уравнение

Дифференциальные уравнения

то, проинтегрировав его последовательно п раз, найдем общее решение уравнения:

Дифференциальные уравнения

Пример:

Решить уравнение Дифференциальные уравнения

Решение:

Последовательно интегрируя четыре раза данное уравнение, получим

Дифференциальные уравнения

II. Пусть дано уравнение

Дифференциальные уравнения

не содержащее явно искомой функции у.

Обозначим у’ = р, где р = р(х) — новая неизвестная функция. Тогда у» = р’ и уравнение (49.7) принимает вид р’ = f(x;p). Пусть Дифференциальные уравнения — общее решение полученного ДУ первого порядка. Заменяя функцию р на у’, получаем ДУ:Дифференциальные уравнения. Оно имеет вид (49.6). Для отыскания у достаточно проинтегрировать последнее уравнение. Общее решение уравнения (49.7) будет иметь вид Дифференциальные уравненияЧастным случаем уравнения (49.7) является уравнение

Дифференциальные уравнения

не содержащее также и независимую переменную х. Оно интегрируется тем же способом: Дифференциальные уравнения Получаем уравнение р’ = f(р) с разделяющимися переменными. Если задано уравнение вида

Дифференциальные уравнения

которое также не содержит явно искомой функции, то его порядок можно понизить на k единиц, положив Дифференциальные уравненияТогда Дифференциальные уравнения и уравнение (49.9) примет вид Дифференциальные уравнения Частным случаем уравнения (49.9) является уравнение

Дифференциальные уравнения

или

Дифференциальные уравнения

С помощью замены Дифференциальные уравненияэто уравнение сводится к ДУ первого порядка.

Пример:

Решить уравнение Дифференциальные уравнения

Решение:

Полагаем у’ = р, где р = р(х), у» = р’.

Тогда Дифференциальные уравнения Это уравнение с разделяющимися переменными: Дифференциальные уравненияИнтегрируя, получим Дифференциальные уравнения Возвращаясь к исходной переменной, получим Дифференциальные уравнения — общее решение уравнения.

III. Рассмотрим уравнение

Дифференциальные уравнения

которое не содержит явно независимой переменной х.

Для понижения порядка уравнения введем новую функцию р = р(у), зависящую от переменной у, полагая у’ = р. Дифференцируем это равенство по х, учитывая, что р = р(у(х)):

Дифференциальные уравнения

т. e. Дифференциальные уравненияТеперь уравнение (49.10) запишется в виде Дифференциальные уравнения

Пусть Дифференциальные уравнения является общим решением этого ДУ первого порядка. Заменяя функцию р(у) на у’, получаем Дифференциальные уравнения — ДУ с разделяющимися переменными. Интегрируя его, находим общий интеграл уравнения (49.10):

Дифференциальные уравнения

Частным случаем уравнения (49.10) является ДУ

Дифференциальные уравнения

Такое уравнение решается при помощи аналогичной подстановки:

Дифференциальные уравнения

Так же поступаем при решении уравнения Дифференциальные уравнения Его порядок можно понизить на единицу, положив Дифференциальные уравнения

По правилу дифференцирования сложной функции находим Дифференциальные уравнения

Затем найдем

Дифференциальные уравнения

и т. д.

Замечание. Уравнение (49.8) также можно решать, применяя подстановку у’ = р, где р = р(у).

Пример:

Найти частное решение уравнения

Дифференциальные уравнения

удовлетворяющее начальным условиям: у(0) = 2, у'(0) = 2.

Решение:

Уравнение имеет вид (49.10). Положив Дифференциальные уравнения получаем:

Дифференциальные уравнения

Так как Дифференциальные уравнения (иначе у’ = 0, что противоречит начальному условию Дифференциальные уравнения — получили линейное ДУ первого порядка.

Проведем решение полученного линейного ДУ методом Бернулли (п. 48.4). Полагаем Дифференциальные уравнения Имеем: u’v + uv’ — uv + у -1 =0, или u’v + u(v’ — v) — 1 — у.

Подберем функцию v так, чтобы v’ — v = 0. Тогда Дифференциальные уравнения

Получаем:

Дифференциальные уравнения

Интегрируя это равенство, находим, что

Дифференциальные уравнения

Следовательно,

Дифференциальные уравнения

Заменяя р на у‘, получаем: Дифференциальные уравнения Подставляя у’ = 2 и у = 2 в это равенство, находим Дифференциальные уравнения:

Дифференциальные уравнения

Имеем у’ = у. Отсюда Дифференциальные уравненияНаходим Дифференциальные уравнения из начальных условий: Дифференциальные уравненияТаким образом, Дифференциальные уравнения— частное решение данного ДУ.

Линейные дифференциальные уравнения высших порядков

Многие задачи математики, механики, электротехники и других технических наук приводят к линейным дифференциальным уравнениям.

Уравнение вида

Дифференциальные уравнения

где Дифференциальные уравнения — заданные функции (от х), называется линейным. ДУ п-го порядка.

Оно содержит искомую функцию у и все ее производные лишь в первой степени. Функции Дифференциальные уравненияназываются коэффициентами уравнения (49.11), а функция g(х) — его свободным членом. Если свободный член Дифференциальные уравнения то уравнение (49.11) называется линейным однородным уравнением; если Дифференциальные уравнения то уравнение (49.11) называется неоднородным.

Разделив уравнение (49.11) на Дифференциальные уравнения и обозначив

Дифференциальные уравнения

запишем уравнение (49.11) в виде приведенного:

Дифференциальные уравнения

Далее будем рассматривать линейные ДУ вида (49.12) и считать, что коэффициенты и свободный член уравнения (49.12) являются непрерывными функциями (на некотором интервале (а; b)). При этих условиях справедлива теорема существования и единственности решения ДУ (49.12) (см. теорему 49.1).

Линейные однородные ДУ второго порядка

Рассмотрим линейное однородное дифференциальное уравнение (ЛОДУ) второго порядка:

Дифференциальные уравнения

и установим некоторые свойства его решений.

Теорема:

Если функции Дифференциальные уравнения являются частными решениями уравнения (49.13), то решением этого уравнения является также функция

Дифференциальные уравнения

где Дифференциальные уравнения— произвольные постоянные.
Подставим функцию Дифференциальные уравненияи ее производные в левую часть ЛОДУ (49.13). Получаем:

Дифференциальные уравнения

так как функции Дифференциальные уравнения— решения уравнения (49.13) и, значит, выражения в скобках тождественно равны нулю.

Таким образом, функция Дифференциальные уравнения также является решением уравнения (49.13).

Из теоремы 49.2, как следствие, вытекает, что если Дифференциальные уравнения — решения уравнения (49.13), то решениями его будут также функции Дифференциальные уравнения

Функция (49.14) содержит две произвольные постоянные и является решением уравнения (49.13). Может ли она являться общим решением уравнения (49.13)?

Для ответа на вопрос введем понятие линейной зависимости и линейной независимости функций.

[ф] Функции Дифференциальные уравнения называются линейно независимыми на интервале (а; b), если равенство

Дифференциальные уравнения

где Дифференциальные уравнения выполняется тогда и только тогда, когда Дифференциальные уравнения Если хотя бы одно из чисел Дифференциальные уравнения отлично от нуля и выполняется равенство (49.15), то функции Дифференциальные уравненияназываются линейно зависимыми на (a;b).

Очевидно, что функции Дифференциальные уравнения линейно зависимы тогда и только тогда, когда они пропорциональны, т. е. для всех Дифференциальные уравнения выполняется равенство Дифференциальные уравнения

Например, функции Дифференциальные уравнения линейно зависимы: Дифференциальные уравнения функции Дифференциальные уравнения — линейно независимы: Дифференциальные уравненияфункции Дифференциальные уравнения являются линейно независимыми: равенство Дифференциальные уравнениявыполняется для всех

Дифференциальные уравнения

Средством изучения линейной зависимости системы функций является так называемый определитель Вронского или вронскиан (Ю. Вронский — польский математик).

Для двух дифференцируемых функций Дифференциальные уравнения вронскиан имеет вид

Дифференциальные уравнения

Имеют место следующие теоремы.

Теорема:

Если дифференцируемые функции Дифференциальные уравнения линейно зависимы на (а; b), то определитель Вронского на этом интервале тождественно равен нулю.

Так как функции Дифференциальные уравнения линейно зависимы, то в равенстве (49.15) значениеДифференциальные уравнения отлично от нуля. Пусть Дифференциальные уравненияпоэтому для любого Дифференциальные уравнения

Дифференциальные уравнения

Теорема:

Если функции Дифференциальные уравнения— линейно независимые решения уравнения (49.13) на (а; b), то определитель Вронского на этом интервале нигде не обращается в нуль.

Доказательство теоремы опустим.

Из теорем 49.3 и 49.4 следует, что вронскиан не равен нулю ни в одной точке интервала (а; b) тогда и только тогда, когда частные решения линейно независимы.

Совокупность любых двух линейно независимых на интервале (а; b) частных решений Дифференциальные уравненияЛОДУ второго порядка определяет фундаментальную систему решений этого уравнения: любое произвольное решение может быть получено как комбинация Дифференциальные уравнения

Пример:

Частные решения

Дифференциальные уравнения

(их бесчисленное множество!) уравнения у» + у = 0 образуют фундаментальную систему решений; решения же Дифференциальные уравнения— не образуют.

Теперь можно сказать, при каких условиях функция (49.14) будет общим решением уравнения (49.13).

Теорема:

Структура общего решения ЛОДУ второго порядка. Если два частных решения Дифференциальные уравнения ЛОДУ (49.13) образуют на интервале (а; b) фундаментальную систему, то общим решением этого уравнения является функция

Дифференциальные уравнения

где Дифференциальные уравнения — произвольные постоянные.

Согласно теореме 49.2, функция (49.16) является решением уравнения (49.13). Остается доказать, что это решение общее, т. е. что из него можно выделить единственное частное решение, удовлетворяющее заданным начальным условиям

Дифференциальные уравнения

где Дифференциальные уравнения

Подставив начальные условия (49.17) в решение (49.14), получим систему уравнений

Дифференциальные уравнения

где Дифференциальные уравнения с неизвестными Дифференциальные уравнения. Определитель этой системы

Дифференциальные уравнения

равен значению вронскиана Дифференциальные уравнения

Так как решения Дифференциальные уравненияобразуют фундаментальную систему решений на Дифференциальные уравнения то, согласно теореме 49.4, Дифференциальные уравненияПоэтому система уравнений имеет единственное решение:

Дифференциальные уравнения

Решение Дифференциальные уравнения является частным решением (единственным, в силу теоремы единственности) уравнения (49.13), удовлетворяющим начальным условиям (49.17). Теорема доказана.

Пример:

На основании теоремы 49.5 общим решением уравнения у»+ у=0 (см. пример 49.4) является функция Дифференциальные уравнения

Линейные однородные ДУ n-го порядка

Полученные результаты можно распространить на линейные однородные дифференциальные уравнения п-го порядка, имеющие вид

Дифференциальные уравнения

1.Если функции Дифференциальные уравнения являются частными решениями уравнения (49.18), то его решением является и функция Дифференциальные уравнения

2.Функции Дифференциальные уравнения называются линейно независимыми на (а; b), если равенство Дифференциальные уравнения выполняется лишь в случае, когда все числа Дифференциальные уравнения в противном случае (если хотя бы одно из чисел Дифференциальные уравнения не равно нулю) функции Дифференциальные уравнениялинейно зависимы.

3.Определитель Вронского имеет вид

Дифференциальные уравнения

4. Частные решения Дифференциальные уравнения уравнения (49.18) образуют фундаментальную систему решений на (а; b), если ни в одной точке этого интервала вронскиан не обращается в нуль, т. е. Дифференциальные уравнения для всех Дифференциальные уравнения.

5. Общее решение ЛОДУ (49.18) имеет вид Дифференциальные уравнения— произвольные постоянные, Дифференциальные уравнения — частные решения уравнения (49.18), образующие фундаментальную систему.

Пример:

Показать, что функции Дифференциальные уравненияобразуют фундаментальную систему решений некоторого ЛОДУ третьего порядка (дополнительно: составить это уравнение).

Решение:

Найдем W(x):

Дифференциальные уравнения

Ясно, что Дифференциальные уравнениядля всех Дифференциальные уравнения Следовательно, данные функции образуют фундаментальную систему решений ЛОДУ третьего порядка. В общем виде ЛОДУ третьего порядка выглядит так:

Дифференциальные уравнения

Подставив функции Дифференциальные уравнения в это уравнение, получим систему из трех уравнений относительно функций Дифференциальные уравнения Решая ее, получим ЛОДУ Дифференциальные уравнения его общее решение:

Интегрирование ДУ второго порядка с постоянными коэффициентами

Интегрирование ЛОДУ второго порядка с постоянными коэффициентами:

Частным случаем рассмотренных выше линейных однородных дифференциальных уравнений являются ЛОДУ с постоянными коэффициентами.

Пусть дано ЛОДУ второго порядка

Дифференциальные уравнения

где р и q постоянны.

Для нахождения общего решения уравнения (50.1) достаточно найти два его частных решения, образующих фундаментальную систему (см. теорему 49.5).

Будем искать частные решения уравнения (50.1) в виде

Дифференциальные уравнения

где k — некоторое число (предложено Л. Эйлером). Дифференцируя эту функцию два раза и подставляя выражения для у, у’ и у» в уравнение (50.1), получим:

Дифференциальные уравнения
Дифференциальные уравнения

Уравнение (50.2) называется характеристическим уравнением ДУ (50.1) (для его составления достаточно в уравнении (50.1) заменить у», у’ к у соответственно на Дифференциальные уравнения).

При решении характеристического уравнения (50.2) возможны следующие три случая.

Случай 1. Корни Дифференциальные уравнения уравнения (50.2) действительные и различные:

Дифференциальные уравнения

В этом случае частными решениями уравнения (50.1) являются функции Дифференциальные уравнения Они образуют фундаментальную систему решений (линейно независимы), т. к. их вронскиан

Дифференциальные уравнения

Следовательно, общее решение уравнения (50.1), согласно формуле (49.16), имеет вид

Дифференциальные уравнения

Пример:

Решить уравнение Дифференциальные уравнения

Решение:

Составим характеристическое уравнение: Дифференциальные уравнения Решаем его:Дифференциальные уравнения Записываем общее решение данного уравнения: Дифференциальные уравнения — произвольные постоянные (формула (50.3)).

Случай 2. Корни Дифференциальные уравнения характеристического уравнения (50.2) действительные и равные:

Дифференциальные уравнения

В этом случае имеем лишь одно частное решение Дифференциальные уравнения Покажем, что наряду Дифференциальные уравнения решением уравнения (50.1) будет и Дифференциальные уравнения

Действительно, подставим функцию Дифференциальные уравнения в уравнение (50.1). Имеем:

Дифференциальные уравнения

Ho Дифференциальные уравнения есть корень уравнения (50.2); Дифференциальные уравненият. к. по условию Дифференциальные уравнения

Поэтому Дифференциальные уравнения т. е. функция Дифференциальные уравнения является решением уравнения (50.1).

Частные решения Дифференциальные уравненияобразуют фундаментальную систему решений: Дифференциальные уравненияСледовательно, в этом случае общее решение ЛОДУ (50.1) имеет вид

Дифференциальные уравнения

Случай 3. Корни Дифференциальные уравнения уравнения (50.2) комплексные:

Дифференциальные уравнения

В этом случае частными решениями уравнения (50.1) являются функции Дифференциальные уравнения По формулам Эйлера (см. п. 27.3)

Дифференциальные уравнения

имеем

Дифференциальные уравнения

Найдем два действительных частных решения уравнения (50.1). Для этого составим две линейные комбинации решений Дифференциальные уравнения:

Дифференциальные уравнения

Функции Дифференциальные уравнения являются решениями уравнения (50.1), что следует из свойств решений ЛОДУ второго порядка (см. теорему 49.2).Эти решения Дифференциальные уравнения образуют фундаментальную систему решений, так как Дифференциальные уравнения(убедитесь самостоятельно!). Поэтому общее решение уравнения (50.1) запишется в виде Дифференциальные уравнения или

Дифференциальные уравнения

Пример:

Решить уравнение Дифференциальные уравнения

Решение:

Имеем:

Дифференциальные уравнения

По формуле (50.5) получаем общее решение уравнения:

Дифференциальные уравнения

Таким образом, нахождение общего решения ЛОДУ второго порядка с постоянными коэффициентами (50.1) сводится к нахождению корней характеристического уравнения (50.2) и использованию формул (50.3)-(50.5) общего решения уравнения (не прибегая к вычислению интегралов).

Интегрирование ЛОДУ n-го порядка с постоянными коэффициентами

Задача нахождения общего решения ЛОДУ п-го порядка (п > 2) с постоянными коэффициентами

Дифференциальные уравнения

где Дифференциальные уравнения— числа, решается аналогично случаю уравнения второго порядка с постоянными коэффициентами.

Дифференциальные уравнения

Сформулируем необходимые утверждения и рассмотрим примеры.

Частные решения уравнения (50.6) также ищем в виде Дифференциальные уравнения где k — постоянное число.

Характеристическим для уравнения (50.6) является алгебраическое уравнение n-го порядка вида

Дифференциальные уравнения

Уравнение (50.7) имеет, как известно, п корней (в их числе могут быть и комплексные). Обозначим их через Дифференциальные уравнения

Замечание:

Не все из корней уравнения (50.7) обязаны быть различными. Так, в частности, уравнение Дифференциальные уравнения имеет два равных корня: Дифференциальные уравнения В этом случае говорят, что корень один (k = 3) и имеет кратность Дифференциальные уравнения Если кратность корня равна единице: Дифференциальные уравнения его называют простым.

Случай 1. Все корни уравнения (50.7) действительны и просты (различны). Тогда функции Дифференциальные уравнения являются частными решениями уравнения (50.6) и образуют фундаментальную систему решений (линейно независимы). Поэтому общее решение уравнения (50.6) записывается в виде

Дифференциальные уравнения

Пример:

Найти общее решение уравнения

Дифференциальные уравнения

Решение:

Характеристическое уравнение Дифференциальные уравненияимеет корни Дифференциальные уравненияСледовательно,Дифференциальные уравнения — общее решение данного уравнения.

Случай 2. Все корни характеристического уравнения действительные, но не все простые (есть корни, имеющие кратность m > 1). Тогда каждому простому корню к соответствует одно частное решение вида Дифференциальные уравнения, а каждому корню k кратности m > 1 соответствует m частных решений: Дифференциальные уравнения

Пример:

Решить уравнение

Дифференциальные уравнения

Решение:

Характеристическое уравнение

Дифференциальные уравнения

имеет корни Дифференциальные уравнения Следовательно,

Дифференциальные уравнения

— общее решение уравнения.

Случай 3. Среди корней уравнения (50.7) есть комплексно-сопряженные корни. Тогда каждой паре Дифференциальные уравнения простых комплексно-сопряженных корней соответствует два частных решения Дифференциальные уравненияа каждой паре Дифференциальные уравнения корней кратности m > 1 соответствуют 2m частных решений вида

Дифференциальные уравнения

Эти решения, как можно доказать, образуют фундаментальную систему решений.

Пример:

Решить уравнение

Дифференциальные уравнения

Решение:

Характеристическое уравнение

Дифференциальные уравнения

имеет корни

Дифференциальные уравнения

Следовательно,

Дифференциальные уравнения

— общее решение уравнения.

Линейные неоднородные дифференциальные уравнения (ЛНДУ)

Структура общего решения ЛНДУ второго порядка:

Рассмотрим ЛНДУ второго порядка

Дифференциальные уравнения

Дифференциальные уравнения—заданные, непрерывные на (а; b) функции. Уравнение

Дифференциальные уравнения

левая часть которого совпадает с левой частью ЛНДУ (51.1), называется соответствующим ему однородным уравнением.

Теорема:

Структура общего решения ЛНДУ. Общим решением у уравнения (51.1) является сумма его произвольного частного решения у* и общего решения Дифференциальные уравнения соответствующего однородного уравнения (51.2), т. е.

Дифференциальные уравнения

Убедимся, что функция (51.3) — решение уравнения (51.1). Так как у* есть решение уравнения (51.1), аДифференциальные уравнения — решение уравнения (51.2), то

Дифференциальные уравнения

В таком случае имеем:

Дифференциальные уравнения

Это означает, что функция Дифференциальные уравнения является решением уравнения (51.1). Покажем теперь, что функция

Дифференциальные уравнения

является общим решением уравнения (51.1). Для этого надо доказать, что из решения (51.4) можно выделить единственное частное решение, удовлетворяющее заданным начальным условиям

Дифференциальные уравнения

Продифференцировав функцию (51.4) и подставив начальные условия (51.5) в функцию (51.4) и ее производную, получим систему уравнений:

Дифференциальные уравнения

где Дифференциальные уравнения с неизвестными Дифференциальные уравнения. Определителем этой системы является определитель Вронского Дифференциальные уравнения для функции Дифференциальные уравнения в точке Дифференциальные уравнения Функции Дифференциальные уравнения линейно независимы (образуют фундаментальную систему решений), т. е. Дифференциальные уравнения. Следовательно, система имеет единственное решение: Дифференциальные уравнения

Решение:

Дифференциальные уравнения является частным решением уравнения (51.1), удовлетворяющим заданным начальным условиям (51.5). Теорема доказана.

Метод вариации произвольных постоянных

Рассмотрим ЛНДУ (51.1). Его общим решением является функция (51.3), т. е.

Дифференциальные уравнения

Частное решение у* уравнения (51.1) можно найти, если известно общее решение Дифференциальные уравнения соответствующего однородного уравнения (51.2), методом вариации произвольных постоянных (метод Лагранжа), состоящим в следующем. Пусть Дифференциальные уравнения — общее решение уравнения (51.2). Заменим в общем решении постоянные Дифференциальные уравнения неизвестными функциями Дифференциальные уравнения и подберем их так, чтобы функция

Дифференциальные уравнения

была решением уравнения (51.1). Найдем производную

Дифференциальные уравнения

Подберем функции Дифференциальные уравнения так, чтобы

Дифференциальные уравнения

Тогда

Дифференциальные уравнения

Подставляя выражение для у* , (у*)’ и (у*)» в уравнение (51.1), получим:

Дифференциальные уравнения

или

Дифференциальные уравнения

Поскольку Дифференциальные уравнения — решения уравнения (51.2), то выражения в квадратных скобках равны нулю, а потому

Дифференциальные уравнения

Таким образом, функция (51.6) будет частным решением у* уравнения (51.1), если функцииДифференциальные уравнения удовлетворяют системе уравнений (51.7) и (51.8):

Дифференциальные уравнения

Определитель системы Дифференциальные уравнения так как это определитель Вронского для фундаментальной системы частных решений Дифференциальные уравненияуравнения (51.2). Поэтому система (51.9) имеет единственное решение: Дифференциальные уравнения — некоторые функции от х. Интегрируя эти функции, находимДифференциальные уравнения а затем по формуле (51.6) составляем частное решение уравнения (51.1).

Пример:

Найти общее решение уравнения

Дифференциальные уравнения

Решение:

Найдем общее решение у соответствующего однородного уравнения у» + у = 0. Имеем: Дифференциальные уравнения Следовательно,Дифференциальные уравнения Найдем теперь частное решение у* исходного уравнения. Оно ищется в виде (51.6): Дифференциальные уравнения Для нахождения Дифференциальные уравнения составляем систему уравнений вида (51.9):

Дифференциальные уравнения

Решаем ее:

Дифференциальные уравнения

Запишем частное решение данного уравнения: Дифференциальные уравнения Следовательно, общее решение данного уравнения имеет вид

Дифференциальные уравнения

При нахождении частных решений ЛНДУ может быть полезной следующая теорема.

Теорема:

О наложении решений. Если правая часть уравнения (51.1) представляет собой сумму двух функций: Дифференциальные уравнения — частные решения уравнений

Дифференциальные уравнения

соответственно, то функция Дифференциальные уравнения является решением данного уравнения.

Действительно,

Дифференциальные уравнения

Интегрирование ЛНДУ второго порядка с постоянными коэффициентами и правой частью специального вида

Рассмотрим ЛНДУ второго порядка с постоянными коэффициентами, т. е. уравнение

Дифференциальные уравнения


где р и q — некоторые числа.

Согласно теореме 51.1, общее решение уравнения (51.10) представляет собой сумму общего решения Дифференциальные уравнения соответствующего однородного уравнения и частного решения у* неоднородного уравнения. Частное решение уравнения (51.10) может быть найдено методом вариации произвольных постоянных (п. 51.2).

Для уравнений с постоянными коэффициентами (51.10) существует более простой способ нахождения у*, если правая часть f(х) уравнения (51.10) имеет тале называемый «специальный вид»:

Дифференциальные уравнения

Суть метода, называемого методом неопределенных коэффициентов, состоит в следующем: по виду правой части f(х) уравнения (51.10) записывают ожидаемую форму частного решения с неопределенными коэффициентами, затем подставляют ее в уравнение (51.10) и из полученного тождества находят значения коэффициентов.

Случай 1. Правая часть (51.10) имеет вид Дифференциальные уравнениягде аДифференциальные уравнения — многочлен степени п. Уравнение (51.10) запишется в виде

Дифференциальные уравнения

В этом случае частное решение Дифференциальные уравнения ищем в виде:

Дифференциальные уравнения

где r — число, равное кратности а как корня характеристического уравнения Дифференциальные уравнения(т. е. r — число, показывающее, сколько раз а является корнем уравнения Дифференциальные уравнения), a Qn(x) —Дифференциальные уравнения — многочлен степени п, записанный с неопределенными коэффициентами Дифференциальные уравнения

а) Пусть а не является корнем характеристического уравнения

Дифференциальные уравнения

т. е. Дифференциальные уравнения Следовательно,

Дифференциальные уравнения

После подстановки функции у* и ее производных в уравнение (51.11), сокращения на Дифференциальные уравнения получим:

Дифференциальные уравнения

Слева — многочлен степени n с неопределенными коэффициентами, справа — многочлен степени п, но с известными коэффициентами. Приравнивая коэффициенты при одинаковых степенях х, получим систему (п+ 1) алгебраических уравнений для определения коэффициентов Дифференциальные уравнения

б) Пусть а является однократным (простым) корнем характеристического уравнения

Дифференциальные уравнения

В этом случае искать решение в форме Дифференциальные уравнения нельзя, т. к. Дифференциальные уравнения и уравнение (51.13) принимает вид

Дифференциальные уравнения

В левой части — многочлен степени (п — 1), в правой части — многочлен степени п. Чтобы получить тождество многочленов в решении у, нужно иметь многочлен степени (n + 1). Поэтому частное решение у следует искать в виде Дифференциальные уравнения(в равенстве (51.12) положить r = 1).

в) Пусть а является двукратным корнем характеристического уравнения Дифференциальные уравнения В этом случае Дифференциальные уравнения а поэтому уравнение (51.13) принимает вид Дифференциальные уравнения

Дифференциальные уравнения

Слева стоит многочлен степени п — 2. Понятно, чтобы иметь слева многочлен степени п, частное решение у* следует искать в виде

Дифференциальные уравнения

(в равенстве (51.12) положить r = 2).

Случай 2. Правая часть (51.10) имеет вид

Дифференциальные уравнения

гдеДифференциальные уравнения — многочлены степени пит соответственно, Дифференциальные уравнения — действительные числа. Уравнение (51.10) запишется в виде

Дифференциальные уравнения

Можно показать, что в этом случае частное решение у* уравнения (51.14) следует искать в виде

Дифференциальные уравнения

где r— число, равное кратности Дифференциальные уравнения как корня характеристического уравнения Дифференциальные уравнения— многочлены степени l с неопределенными коэффициентами, l — наивысшая степень многочленов

Дифференциальные уравнения

Замечания:

  1. После подстановки функции (51.15) в (51.14) приравнивают многочлены, стоящие перед одноименными тригонометрическими функциями в левой и правой частях уравнения.
  2. Форма (51.15) сохраняется и в случаях, когда Дифференциальные уравнения
  3. Если правая часть уравнения (51.10) есть сумма функций вида I или II, то для нахождения у* следует использовать теорему 51.2 о наложении решений.

Пример:

Найти общее решение уравнения

Дифференциальные уравнения

Решение:

Найдем общее решение Дифференциальные уравнения ЛОДУ у» — 2у’ + у = 0. Характеристическое уравнение Дифференциальные уравненияимеет корень Дифференциальные уравнениякратности 2. Значит, Дифференциальные уравнения Находим частное решение исходного уравнения. В нем правая часть Дифференциальные уравнения есть формула вида Дифференциальные уравнения причем Дифференциальные уравнения не является корнем характеристического уравнения: Дифференциальные уравненияПоэтому, согласно формуле (51.12), частное решение у* ищем в виде

Дифференциальные уравнения

— неопределенные коэффициенты. Тогда ( у* )’ = А, ( у* )» = 0. Подставив у* , ( у* )’, ( у*в исходное уравнение, получим

Дифференциальные уравнения

Приравнивая коэффициенты при одинаковых степенях х, получаем систему уравнений:

Дифференциальные уравнения

Отсюда А = 1, В = —2. Поэтому частное решение данного уравнения имеет вид у* = х—2. Следовательно, Дифференциальные уравнения — искомое общее решение уравнения.

Пример:

Решить уравнение

Дифференциальные уравнения

Решение:

Общее решение ЛНДУ имеет вид Дифференциальные уравнения Находим решение однородного уравнения Дифференциальные уравненияХарактеристическое уравнениеДифференциальные уравнения имеет корни Дифференциальные уравнения Следовательно,

Дифференциальные уравнения

Находим частное решение Дифференциальные уравнения Правая часть ЛНДУ в нашем случае имеет вид Дифференциальные уравненияТак как Дифференциальные уравнения не совпадает с корнем характеристического уравнения, то r = 0. Согласно формуле (51.15), частное решение ищем в виде Дифференциальные уравнения Подставляем у* в исходное уравнение. Имеем:

Дифференциальные уравнения

Получаем:

Дифференциальные уравнения

или

Дифференциальные уравнения

Отсюда имеем:

Дифференциальные уравнения

Следовательно, А = 1, В = —3. Поэтому у* = cos3x — 3sin3x. И наконец,

Дифференциальные уравнения

— общее решение уравнения.

Интегрирование ЛНДУ n-го порядка (п > 2) с постоянными коэффициентами и правой частью специального вида

Рассмотрим линейное неоднородное ДУ n-го (п > 2) порядка

Дифференциальные уравнения

где Дифференциальные уравнения— заданные непрерывные функции на (a; b).

Соответствующее ему однородное уравнение имеет вид

Дифференциальные уравнения

Теорема:

О структуре общего решения ЛНДУ n-го порядка. Общее решение у ЛНДУ n-го порядка равно сумме частного решения у* неоднородного уравнения и общего решения Дифференциальные уравнения соответствующего ему однородного уравнения, т. е. Дифференциальные уравнения

Частное решение у* ЛНДУ n-го порядка может быть найдено, если известно общее решение Дифференциальные уравнения однородного уравнения, методом вариации произвольных постоянных. Оно ищется в виде

Дифференциальные уравнения

где Дифференциальные уравнения — частные решения, образующие фундаментальную систему, однородного уравнения.

Система уравнений для нахождения неизвестных Дифференциальные уравнения имеет вид

Дифференциальные уравнения

Однако для ЛНДУ n-го порядка с постоянными коэффициентами, правая часть которого имеет специальный вид, частное решение у* может быть найдено методом неопределенных коэффициентов. Метод подбора частного решения у* уравнения

Дифференциальные уравнения

где Дифференциальные уравнения — числа, а правая часть f(х) имеет специальный вид, описанный в п. 51.3 для случая п = 2, переносится без всякого изменения и на случай уравнения, имеющего порядок п > 2.

Пример:

Решить уравнение Дифференциальные уравнения

Решение:

Находим Дифференциальные уравнения:

Дифференциальные уравнения

Находим

Дифференциальные уравнения

отсюда

Дифференциальные уравнения

Тогда —Дифференциальные уравнения Отсюда А = —1, В = 0 и получаем Дифференциальные уравнения

Следовательно, функция

Дифференциальные уравнения

является общим решением уравнения.

Системы дифференциальных уравнении

Для решения многих задач математики, физики, техники (задач динамики криволинейного движения; задач электротехники для нескольких электрических цепей; определения состава системы, в которой протекают несколько последовательных химических реакций I порядка; отыскания векторных линий поля и других) нередко требуется несколько функций. Нахождение этих функций может привести к нескольким ДУ, образующим систему.

Системой ДУ называется совокупность ДУ, каждое из которых содержит независимую переменную, искомые функции и их производные.

Общий вид системы ДУ первого порядка, содержащей п искомых функций Дифференциальные уравнения следующий:

Дифференциальные уравнения

Система ДУ первого порядка, разрешенных относительно производной, т. е. система вида

Дифференциальные уравнения

называется нормальной системой ДУ. При этом предполагается, что число уравнений равно числу искомых функций. Замечание. Во многих случаях системы уравнений и уравнения высших порядков можно привести к нормальной системе вида (52.1).

Так, система трех ДУ второго порядка

Дифференциальные уравнения

описывающая движение точки в пространстве, путем введения новых переменных: Дифференциальные уравненияприводится к нормальной системе

Дифференциальные уравнения

Уравнение третьего порядка Дифференциальные уравненияпутем замены Дифференциальные уравнениясводится к нормальной системе ДУ

Дифференциальные уравнения

Из сказанного выше следует полезность изучения именно нормальных систем.

Решением системы (52.1) называется совокупность из п функций Дифференциальные уравнения удовлетворяющих каждому из уравнений этой системы.

Начальные условия для системы (52.1) имеют вид

Дифференциальные уравнения

Задача Коши для системы (52.1) ставится следующим образом: найти решение системы (52.1), удовлетворяющее начальным условиям (52.2).

Условия существования и единственности решения задачи Коши описывает следующая теорема, приводимая здесь без доказательства.

Теорема:

Если в системе (52.1) все функции

Дифференциальные уравнения

непрерывны вместе со всеми своими частными производными по у, в некоторой области D ((п + 1)-мерного пространства), то в каждой точке Дифференциальные уравнения этой области существует, и притом единственное, решение Дифференциальные уравнения системы, удовлетворяющее начальным условиям (52.2).

Меняя в области D точку Дифференциальные уравнения (т. е. начальные условия), получим бесчисленное множество решений, которое можно записать в виде решения, зависящего от п произвольных постоянных:

Дифференциальные уравнения

Это решение является общим, если по заданным начальным условиям (52.2) можно однозначно определить постоянныеДифференциальные уравнения из системы уравнений

Дифференциальные уравнения

Решение, получающееся из общего при конкретных значениях постоянных Дифференциальные уравнения, называется частным решением системы (52.1).

Интегрирование нормальных систем

Одним из основных методов интегрирования нормальной системы ДУ является метод сведения системы к одному ДУ высшего порядка. (Обратная задача — переход от ДУ к системе — рассмотрена выше на примере.) Техника этого «метода основана на следующих соображениях.

Пусть задана нормальная система (52.1). Продифференцируем по х любое, например первое, уравнение:

Дифференциальные уравнения

Подставив в это равенство значения производных Дифференциальные уравнения из системы (52.1), получим

Дифференциальные уравнения

или, коротко,

Дифференциальные уравнения

Продифференцировав полученное равенство еще раз и заменив значения производных Дифференциальные уравненияиз системы (52.1), получим

Дифференциальные уравнения

Продолжая этот процесс (дифференцируем — подставляем — получаем), находим:

Дифференциальные уравнения

Соберем полученные уравнения в систему:

Дифференциальные уравнения

Из первых (п — 1) уравнений системы (52.3) выразим функции Дифференциальные уравнения через х, функцию Дифференциальные уравнения и ее производные Дифференциальные уравненияПолучим:

Дифференциальные уравнения

Найденные значения Дифференциальные уравнения подставим в последнее уравнение системы (52.3). Получим одно ДУ n-го порядка относительно искомой функции Дифференциальные уравнения Пусть его общее решение есть

Дифференциальные уравнения

Продифференцировав его (n — 1) раз и подставив значения производных Дифференциальные уравнения в уравнения системы (52.4), найдем функции :

Дифференциальные уравнения

Пример:

Решить систему уравнений

Дифференциальные уравнения

Решение:

Продифференцируем первое уравнение: у» = 4у’ — 3z’. Подставляем z’ = 2у — 3z в полученное равенство: у» = 4у’ — 3(2у — 3z), -у» — 4у1 -1-6у *9z. Составляем систему уравнений:

Дифференциальные уравнения

Из первого уравнения системы выражаем z через у и у’ :

Дифференциальные уравнения

Подставляем значение z во второе уравнение последней системы:

Дифференциальные уравнения

т. е. у» — у’ — 6у = 0. Получили одно ЛОДУ второго порядка. Решаем его:

Дифференциальные уравнения

— общее решение уравнения. Находим функцию z. Значения Дифференциальные уравнения подставляем в выражение z через у и у’ (формула (52.5)). Получим:

Дифференциальные уравнения

Таким образом, общее решение данной системы уравнений имеет вид

Дифференциальные уравнения

Замечание:

Систему уравнений (52.1) можно решать методом интегрируемых комбинаций. Суть метода состоит в том, что посредством арифметических операций из уравнений данной системы образуют так называемые интегрируемые комбинации, т. е. легко интегрируемые уравнения относительно новой неизвестной функции.

Проиллюстрируем технику этого метода на следующем примере.

Пример:

Решить систему уравнений:

Дифференциальные уравнения

Решение:

Сложим почленно данные уравнения: х’+у’ = х+у+2, или (х+у)’ = (х+у)+2. Обозначим х+у = z. Тогда имеем z’ = z+2. Решаем
полученное уравнение:

Дифференциальные уравнения
Дифференциальные уравнения

Получили так называемый первый интеграл системы. Из него можно выразить одну из искомых функций через другую, тем самым

уменьшить на единицу число искомых функций. Например, Дифференциальные уравнения Тогда первое уравнение системы примет вид

Дифференциальные уравнения

Найдя из него х (например, с помощью подстановки х = uv), найдем и у.

Замечание:

Данная система «позволяет» образовать еще одну интегрируемую комбинацию: Дифференциальные уравнения Положив х — у = р, имеем: р’ = —р, или

Дифференциальные уравнения

Имея два первых интеграла системы, т. е. Дифференциальные уравнения легко найти (складывая и вычитая первые интегралы),

Дифференциальные уравнения

Системы линейных ДУ с постоянными коэффициентами

Рассмотрим еще один метод интегрирования нормальной системы уравнений (52.1) в случае, когда она представляет собой систему линейных однородных ДУ с постоянными коэффициентами, т. е. систему вида

Дифференциальные уравнения

Для простоты ограничимся рассмотрением системы трех уравнений с тремя неизвестными функциями Дифференциальные уравнения

Дифференциальные уравнения

где все коэффициенты Дифференциальные уравнения— постоянные. Будем искать частное решение системы (52.6) в виде

Дифференциальные уравнения

где Дифференциальные уравнения— постоянные, которые надо подобрать (найти) так, чтобы функции (52.7) удовлетворяли системе (52.6).

Подставив эти функции в систему (52.6) и сократив на множитель Дифференциальные уравнения получим:

Дифференциальные уравнения

или

Дифференциальные уравнения

Систему (52.8) можно рассматривать как однородную систему трех алгебраических уравнений с тремя неизвестными Дифференциальные уравнения Чтобы эта система имела ненулевое решение, необходимо и достаточно, чтобы определитель системы был равен нулю:

Дифференциальные уравнения

Уравнение (52.9) называется характеристическим уравнением системы (52.6). Раскрыв определитель, получим уравнение третьей степени относительно k. Рассмотрим возможные случаи.

Случай 1. Корни характеристического уравнения действительны и различны:Дифференциальные уравнения Для каждого корня Дифференциальные уравнения напишем систему (52.8) и определим коэффициентыДифференциальные уравнения (один из коэффициентов можно считать равным единице). Таким образом, получаем: для корня Дифференциальные уравнениячастное решение системы (52.6):

Дифференциальные уравнения

Можно показать, что эти функции образуют фундаментальную систему, общее решение системы (52.6) записывается в виде

Дифференциальные уравнения

Пример:

Решить систему уравнений:

Дифференциальные уравнения

Решение:

Характеристическое уравнение (52.9) данной системы имеет вид

Дифференциальные уравнения

При Дифференциальные уравнения система (52.8) имеет вид

Дифференциальные уравнения

Эта система имеет бесчисленное множество решений. Положим Дифференциальные уравнениятогда Дифференциальные уравненияПолучаем частные решения

Дифференциальные уравнения

При Дифференциальные уравнения система (52.8) имеет вид

Дифференциальные уравнения

Положим Дифференциальные уравнения тогда Дифференциальные уравнения Значит, корню Дифференциальные уравнения частные решения:

Дифференциальные уравнения

Общее решение исходной системы, согласно формуле (52.10), запишется в виде:

Дифференциальные уравнения

Случай 2. Корни характеристического уравнения различные, но среди них есть комплексные: Дифференциальные уравнения Вид частных решений в этой ситуации определяют так же, как и в случае 1.

Замечание:

Вместо полученных частных решений можно взять их линейные комбинации (п. 50.1, случай 3), применяя формулы Эйлера; в результате получим два действительных решения, содержащих функции вида Дифференциальные уравнения Или, выделяя действительные и мнимые части в найденных комплексных частных решениях, получим два действительных частных решения (можно показать, что они тоже являются решениями уравнения). При этом понятно, что комплексно-сопряженный кореньДифференциальные уравнения не даст новых линейно независимых действительных решений.

Пример:

Найти частное решение системы

Дифференциальные уравнения

удовлетворяющее начальным условиям:

Дифференциальные уравнения

Решение:

Составляем и решаем характеристическое уравнение:

Дифференциальные уравнения
Дифференциальные уравнения

Для Дифференциальные уравнения=1 получаем:

Дифференциальные уравнения

(см. (52.8)). Отсюда находим: Дифференциальные уравнения (положили), Дифференциальные уравнения Частное решение системы: Дифференциальные уравненияДля Дифференциальные уравнения получаем (см. (52.8)):

Дифференциальные уравнения

Отсюда находим: Дифференциальные уравнения (положили), Дифференциальные уравнения Частное комплексное решение системы:

Дифференциальные уравнения

В найденных решениях выделим действительную (Re) и мнимую (Im) части:

Дифференциальные уравнения
Дифференциальные уравнения

Как уже отмечено, корень Дифференциальные уравненияприведет к этим же самым решениям.

Таким образом, общее решение системы имеет вид

Дифференциальные уравнения

Выделим частное решение системы. При заданных начальных условиях получаем систему уравнений для определения постоянных Дифференциальные уравнения

Дифференциальные уравнения

Следовательно, искомое частное решение имеет вид

Дифференциальные уравнения

Случай 3. Характеристическое уравнение имеет корень k кратности m (m = 2,3). Решение системы, соответствующее кратному корню, следует искать в виде:

Дифференциальные уравнения

Это решение зависит от m произвольных постоянных. Постоянные A,B,C,…,N определяются методом неопределенных коэффициентов. Выразив все коэффициенты через m из них, полагаем поочередно один из них равным единице, а остальные равными нулю. Получим m линейно независимых частных решений системы (52.6).

Пример:

Решить систему уравнений:

Дифференциальные уравнения

Решение: Составляем и решаем характеристическое уравнение

Дифференциальные уравнения
Дифференциальные уравнения

КорнюДифференциальные уравнения соответствует система (см. (52.8)):

Дифференциальные уравнения

Полагая Дифференциальные уравнения находим Дифференциальные уравнения Получаем одно частное решение исходной системы:

Дифференциальные уравнения

Двукратному корню Дифференциальные уравнениясоответствует решение вида

Дифференциальные уравнения

Подставляем эти выражения (решения) в уравнения исходной системы:

Дифференциальные уравнения

или, после сокращения на Дифференциальные уравнения и группировки,

Дифференциальные уравнения

Эти равенства тождественно выполняются лишь в случае, когда

Дифференциальные уравнения

Выразим все коэффициенты через два из них (m = 2), например через А и В. Из второго уравнения имеем F = В. Тогда, с учетом первого уравнения, получаем D = В. Из четвертого уравнения находим Е = А — D, т. е. Е = А — В. Из третьего уравнения: С = Е — В, т. е. С = А —В —В, или С = А — 2В. Коэффициенты А и В. — произвольные.

Полагая А = 1, В = 0, находим: С = 1, D = 0, Е = 1, F = 0. Полагая А = 0, В = 1, находим: С = -2, D = 1, Е = -1, F = 1. Получаем два линейно независимых частных решения, соответствующих двукратному корню k = 1:

Дифференциальные уравнения

Записываем общее решение исходной системы:

Дифференциальные уравнения

Дифференциальные уравнения первого порядка

Дифференциальное уравнение первого порядка — это уравнение, в которое входят независимая переменная, неизвестная функция и первая производная этой функции. Общий вид дифференциального уравнения первого порядка

Дифференциальные уравнения

Здесь F — функция трех аргументов. Если уравнение (3.24) разрешить относительно Дифференциальные уравнения, то получим

Дифференциальные уравнения

где f — функция двух переменных х и у, ее называют «правая часть уравнения».

Решение дифференциального уравнения (3.25) — это функция Дифференциальные уравнения которая, будучи подставлена в это уравнение, обращает его в тождество Дифференциальные уравнения

Пример:

Дано уравнение Дифференциальные уравнения

Решение:

Покажем, что функция Дифференциальные уравнения является его решением. Для этого подставим в данное уравнение вместо у и Дифференциальные уравнения функции Дифференциальные уравнения Имеем:

Дифференциальные уравнения

Для дифференциального уравнения (3.25) задача Коши формулируется так: среди всех решений уравнения (3.25) найти решение Дифференциальные уравненияудовлетворяющее условию

Дифференциальные уравнения

где Дифференциальные уравнения заданные числа. Условие (3.26) называется начальным условием, а числа Дифференциальные уравнения — начальными значениями. Решение, удовлетворяющее начальным условиям, называется частным. Функция Дифференциальные уравнения называется общим решением уравнения (3.25), если она является решением уравнения и из нее единственным образом находится частное решение, удовлетворяющее данному начальному условию. В этом случае частное решение уравнения — решение, которое получается из общего при конкретном значении С, причем произвольная постоянная С определяется из уравнения Дифференциальные уравнения

Частное решение находится единственным образом, если правая часть уравнения непрерывна как функция двух переменных и имеет непрерывную производную по аргументу у. Если общее решение задано в неявном виде Дифференциальные уравнения то оно называется общим интегралом.

Уравнения с разделяющимися переменными

Уравнения с разделяющимися переменными — это уравнение, правая часть которого Дифференциальные уравнения есть произведение двух сомножителей Дифференциальные уравнения каждый из которых зависит только от одной переменной

Дифференциальные уравнения

Метод решения: Дифференциальные уравнения заменяется на Дифференциальные уравнения затем умножаются обе части (3.27) на Дифференциальные уравнения

Получим:

Дифференциальные уравнения

Дифференциалы переменных х и у, и соответствующие функции стоят отдельно, т. е. переменные отделены. Если обозначить Дифференциальные уравнения то уравнение (3.28) можно переписать в виде Дифференциальные уравнения

Так как из равенства дифференциалов двух функций следует, что сами функции отличаются на произвольное постоянное слагаемое, то Дифференциальные уравнения или

Дифференциальные уравнения

Выражение (3.29) представляет собой общий интеграл уравнения (3.27).

Пример:

Решить уравнение Дифференциальные уравнения

Решение:

Разрешим уравнение относительно Дифференциальные уравнения

Здесь Дифференциальные уравнения Заменим в этом уравнении Дифференциальные уравнения на Дифференциальные уравнения и умножим обе части уравнения на Дифференциальные уравнения

Дифференциальные уравнения

Получили уравнение с разделенными переменными. Интегрируя, находим Дифференциальные уравнения где Дифференциальные уравнения произвольная постоянная. Отсюда следует: Дифференциальные уравнения В данном случае удобно вместо Дифференциальные уравнения написать Дифференциальные уравнения

Тогда Дифференциальные уравнения или Дифференциальные уравнения Так как Дифференциальные уравненияпринимает любые значения, то обозначая Дифференциальные уравненияокончательно получим Дифференциальные уравнения где С — произвольная постоянная. Это общее решение заданного уравнения. Найдем теперь частное решение, удовлетворяющее начальному условию Дифференциальные уравнения Для этого в равенство Дифференциальные уравнения подставим вместо х и у значения 4 и 1/2. Получим Дифференциальные уравнения Отсюда следует, что Дифференциальные уравнения Таким образом, искомое частное решение имеет вид Дифференциальные уравнения

Если уравнение с разделяющимися переменными задано в виде

Дифференциальные уравнения

то для отделения переменных достаточно разделить обе части на произведение Дифференциальные уравнения

Дифференциальные уравнения

Отсюда получим интеграл:

Дифференциальные уравнения

Пример:

Найти общее решение уравнения

Дифференциальные уравнения

Решение:

Для отделения переменных представим уравнение в виде

Дифференциальные уравнения

Разделим обе части уравнения на Дифференциальные уравнения

Дифференциальные уравнения

Переменные разделились. Интегрируем это уравнение:

Дифференциальные уравнения

Аналогично вычисляется Дифференциальные уравнения Кроме того, вместо С можно взять Дифференциальные уравнения В итоге получаем

Дифференциальные уравнения

Так как Дифференциальные уравнения то находим общий интеграл уравнения в виде

Дифференциальные уравнения

Линейное дифференциальное уравнение первого порядка

Линейным дифференциальным уравнением первого порядка называется уравнение, линейное относительно искомой функции Дифференциальные уравнения и ее производной Дифференциальные уравнения В общем случае оно имеет вид

Дифференциальные уравнения

где коэффициенты А, В, С — заданные непрерывные функции от х. Предполагая, что в некотором интервале изменения х ко-

эффициент Дифференциальные уравнения не обращается в нуль, делим обе части этого уравнения на Дифференциальные уравнения:

Дифференциальные уравнения

Обозначая Дифференциальные уравнения запишем уравнения в виде

Дифференциальные уравнения

Функция Дифференциальные уравнения называется правой частью уравнения. Если Дифференциальные уравнения то уравнение называется линейным однородным уравнением (линейным уравнением без правой части).

Итак, Дифференциальные уравнения линейное однородное уравнение (его можно рассматривать и как уравнение с разделяющимися переменными). Если Дифференциальные уравнения то уравнение (3.30) называется линейным неоднородным уравнением.

Например, уравнение Дифференциальные уравнения является линейным неоднородным уравнением. Однородным по отношению к нему будет уравнение Дифференциальные уравнения Уравнение (3.30) можно решать разными методами. Мы рассмотрим метод Бернулли или метод вариации произвольных постоянных. Он состоит в следующем. Делаем замену:

Дифференциальные уравнения

т.е., вместо одной неизвестной функции ищем две Дифференциальные уравнения но зато одну из них выбираем такую, какую нам надо. Подставим в уравнение (3.30) вместо у и Дифференциальные уравнения их выражения из (3.31), получим

Дифференциальные уравнения

Так как одну из функций в (3.32) можно выбрать произвольно, то функцию v выберем таким образом, чтобы коэффициент при u обратился в нуль, т.е.

Дифференциальные уравнения

Линейное однородное уравнение (3.30) для функции Дифференциальные уравнения является уравнением с разделяющимися переменными. Поэтому из (3.33) имеем:

Дифференциальные уравнения

Интегрируя, находим

Дифференциальные уравнения

Полагаем Дифференциальные уравнения

Получим

Дифференциальные уравнения

Отсюда следует Дифференциальные уравнения

Интегрируя, получим

Дифференциальные уравнения

где С — произвольная постоянная. Для того чтобы найти Дифференциальные уравнения, умножим найденную функцию Дифференциальные уравнения на Дифференциальные уравнения:

Дифференциальные уравнения

Пример:

Найти общее решение уравнения

Дифференциальные уравнения

Решение:

Это линейное неоднородное уравнение, где

Дифференциальные уравнения

Выполним замену Дифференциальные уравнения Заданное дифференциальное уравнение перепишем в виде

Дифференциальные уравнения

Приравняем выражение в скобках нулю:

Дифференциальные уравнения

Получили уравнение для функции Дифференциальные уравнения — уравнение с разделяющимися переменными. Интегрируем его:

Дифференциальные уравнения

Подставляя функцию Дифференциальные уравнения в уравнение, найдем уравнение для функции Дифференциальные уравнения:

Дифференциальные уравнения

Отсюда следует Дифференциальные уравнения следовательно

Дифференциальные уравнения

Теперь находим общее решение заданного уравнения Дифференциальные уравнения:

Дифференциальные уравнения

Линейные дифференциальные уравнения второго порядка

Дифференциальным уравнением второго порядка называется уравнение, связывающее между собой независимую переменную, функцию, ее первую и вторую производную

Дифференциальные уравнения

Здесь F — функция четырех аргументов. Если уравнение (3.35) разрешить относительно Дифференциальные уравнения, то получим

Дифференциальные уравнения

где f — функция трех переменных х, у и Дифференциальные уравнения. Решением дифференциального уравнения () называется функция Дифференциальные уравнения которая будучи подставлена в уравнение, обращает его в тождество Дифференциальные уравнения Общее решение содержит две произвольные постоянные Дифференциальные уравнения Для дифференциального уравнения (3.36) задача Коши формулируется так: среди всех решений уравнения найти решение Дифференциальные уравнения удовлетворяющее условиям

Дифференциальные уравнения

где Дифференциальные уравнения заданные числа. Частное решение единственно, если функция Дифференциальные уравнениянепрерывна по аргументам х, у и Дифференциальные уравнения, и имеет непрерывные производные по аргументам у и Дифференциальные уравнения.

На практике чаще всего используются так называемые линейные уравнения. Линейным дифференциальным уравнением второго порядка называется уравнение вида

Дифференциальные уравнения

Если Дифференциальные уравнения то уравнение называется однородным, в противном случае уравнение называется неоднородным. Общее решение однородного уравнения является линейной комбинацией двух различных частных решений Дифференциальные уравнения

Дифференциальные уравнения

Тем самым, для того, чтобы найти общее решение линейного уравнения, достаточно найти два различных частных решения. Общее решение неоднородного уравнения складывается из общего решения однородного и частного решения неоднородного уравнения Дифференциальные уравнения

Такие частные решения легко находятся, если коэффициенты уравнения Дифференциальные уравнения есть постоянные величины, т. е. Дифференциальные уравнения

Дифференциальные уравнения

Решением однородного уравнения с постоянными коэффициентами Дифференциальные уравнения может быть только экспоненциальная функция вида Дифференциальные уравнения так как только функция Дифференциальные уравнения сохраняет свой вид при дифференцировании: Дифференциальные уравнения причем Дифференциальные уравнения Тогда

Дифференциальные уравнения

Это обычное квадратное уравнение, которое называется характеристическим. В зависимости от того, чему равен дискриминант уравнения Дифференциальные уравнения различают три случая:

Дифференциальные уравнения следовательно, есть два разных значенияДифференциальные уравнения Общее решение уравнения имеет вид

Дифференциальные уравнения

Дифференциальные уравнения следовательно, есть только одно решение уравнения Дифференциальные уравнения Непосредственной подстановкой можно проверить, что в качестве второго решения можно выбрать Дифференциальные уравнения а общим решением будет Дифференциальные уравнения

Дифференциальные уравнения в этом случае Дифференциальные уравнения

Дифференциальные уравнения

Общее решение уравнения в этом случае имеет вид

Дифференциальные уравнения

Пример:

Найти решение уравнения

Дифференциальные уравнения

Решение:

Характеристическое уравнение имеет вид

Дифференциальные уравнения

Общее решение имеет вид Дифференциальные уравнения

Пример:

Найти решение уравнения

Дифференциальные уравнения

Решение:

Характеристическое уравнение имеет вид

Дифференциальные уравнения

Общее решение имеет вид Дифференциальные уравнения

Пример:

Найти решение уравнения

Дифференциальные уравнения

Решение:

Характеристическое уравнение имеет вид

Дифференциальные уравнения

Общее решение имеет вид

Дифференциальные уравнения

Частное решение Дифференциальные уравнениядля неоднородного уравнения легко находится, если функция f равна:

Дифференциальные уравнения в этом случае Дифференциальные уравнения число В находится подстановкой в уравнение.

Дифференциальные уравнения в этом случае Дифференциальные уравнения числа D и Е находятся подстановкой в уравнение.

Дифференциальные уравненияв этом случае

Дифференциальные уравнения

Пример:

Найти решение уравнения

Дифференциальные уравнения

Решение:

Общее решение имеет вид Дифференциальные уравнения

Ищем Дифференциальные уравнения

Подставим в уравнение

Дифференциальные уравнения

Общее решениеДифференциальные уравнения

Как найти дифференциальные уравнения — подробная инструкция

Семейство функций:

Рассмотрим уравнение

Дифференциальные уравнения

в котором С обозначает любое действительное число, или, как говорят, параметр. Если придадим С определенное значение, например С= 1, то получим уравнение Дифференциальные уравнения, которое имеет своим графиком параболу, изображенную на рис. 108.

Дифференциальные уравнения

Если положить Дифференциальные уравнения то получим новую параболу, уравнение которой будет Дифференциальные уравненияПри С = 0 будем иметь у = 0, т. е. ось Ох. Таким образом, уравнение (1) определяет не одну функцию, а, как говорят, семейство функций, зависящее от одной произвольной постоянной (или параметра). Поскольку каждая функция имеет своим графиком некоторую кривую, то уравнение (1) определяет семейство кривых. На рис. 108 изображены кривые, принадлежащие семейству и соответствующие

Дифференциальные уравнения

Теперь найдем производную от функции (1)

Дифференциальные уравнения

и рассмотрим систему уравнений (1) и (2). Исключим из них произвольное постоянное (параметр) С. Решая уравнение (2) относительно С и подставляя полученное выражение в уравнение (1), получим:

Дифференциальные уравнения

или

Дифференциальные уравнения

Это уравнение связывает х, у и у’ для любой кривой семейства, определенного уравнением (1).

Уравнение (3) называют дифференциальным уравнением семейства кривых (1). Этому уравнению удовлетворяет любая функция семейства (1). Это надо понимать так: если возьмем уравнение любой функции из семейства (1), найдем производную и подставим в УI . уравнение (3), то получим тождество

Дифференциальные уравнения

Рассматривая дифференциальное уравнение (3), можно найти некоторые свойства всех кривых семейства (1).Например, в первой четверти х и у положительны; поэтому выражение Дифференциальные уравнения в первой четверти всегда больше нуля и, следовательно, производная всегда положительна. Отсюда можно заключить, что все функции рассматриваемого семейства в первой четверти возрастают. Значит, рассмотрение дифференциального уравнения семейства позволило нам сделать заключение не об одной кривой, а о всем семействе сразу.

Теперь рассмотрим уравнение

Дифференциальные уравнения

где С1 и С2—произвольные постоянные, или параметры. Если взять, например, значения С1 = 1 и С2 = 0, тогда получим у = cos x. Эта кривая изображена на рис. 109.

Дифференциальные уравнения

Если С1= 0, С2 = 1, то получаем у = sin х. Эта кривая также изображена на рис. 109.

Вообще, для каждой пары значений С1 и С2 получается определенная кривая (на рис. 109) изображены кривые, соответствующие С1 = 1 и С2 = 0; С1 = 0 и С2 = 1; С1 = 1 и С2= 1). Будем говорить, что уравнение (4) определяет семейство функций, зависящее от двух произвольных постоянных (или двух параметров), а так как каждой функции соответствует кривая, то будем также говорить, что уравнение (4) определяет семейство кривых. Найдем первую и вторую производные от функции, определяемой уравнением (4):

Дифференциальные уравнения

Исключим из уравнений (4), (5) и (6) параметры С1 и С2 [в этом примере С1 и С2 проще всего исключаются при помощи сложения уравнений (4) и (6)]. После исключения будем иметь

Дифференциальные уравнения

Уравнение (7) является дифференциальным уравнением семейства (4). Из уравнения (7) находим, что y» = —у; отсюда видно, что в первой и второй четвертях, т. е. при y > 0, вторая производная отрицательна. А если у» < 0, то (см. гл. VIII) кривая выпукла. Таким образом, опять из рассмотрения дифференциального уравнения мы выяснили, что все кривые семейства, если они расположены над осью Ох, выпуклы, а если ниже оси Ох, вогнуты.

Приведенные примеры показывают связь между дифференциальными уравнениями и семействами кривых.

Основные определения

Дифференциальным уравнением называется уравнение, в которое входят: независимое переменное, неизвестная функция и производные от этой неизвестной функции. При этом в уравнение обязательно должны входить производные, а независимое переменное и сама искомая функция явно могут не входить. Например,

Дифференциальные уравнения

являются дифференциальными уравнениями, хотя второе и третье уравнения не содержат явно неизвестной функции, а в четвертое не входит независимое переменное.

Наивысший порядок производной, входящей в дифференциальное уравнение, называется порядком дифференциального уравнения.

Приведем примеры дифференциальных уравнений первого порядка:

Дифференциальные уравнения

Примерами дифференциальных уравнений второго порядка могут служить следующие:

Дифференциальные уравнения

Уравнение ху'» —уу’ = 0 есть дифференциальное уравнение третьего порядка. Уравнение Дифференциальные уравнения — пятого порядка. Примером дифференциального уравнения порядка п может служить уравнение

Дифференциальные уравнения

Решить, или проинтегрировать, дифференциальное уравнение это значит найти такую функцию, которая, будучи подставлена в дифференциальное уравнение, обращает его в тождество. Такая функция называется решением дифференциального уравнения.

Например, функция Дифференциальные уравнения является решением дифференциального уравнения ху»— у’ = 0. Действительно, у’= 2х, у» = 2; подставляя у’ , у» в уравнение ху»— у’ = 0 , получаем Дифференциальные уравнения , т. е. тождество.

Если решение дифференциального уравнения содержит столько независимых произвольных постоянных, каков порядок уравнения, то такое решение называется общим решением дифференциального уравнения.

Поясним, что называют независимыми произвольными постоянными:

Постоянные, входящие в решение, называются независимыми, если они входят в решение так, что нельзя заменить никакую комбинацию двух или нескольких из них при помощи введения нового постоянного и тем самым уменьшить число постоянных.

Например, если имеем

Дифференциальные уравнения

то сюда входят три постоянных C1, С2 и С3. Однако можно заданное уравнение переписать в виде

Дифференциальные уравнения

Если теперь обозначим С2 + С3 через С, то последнее уравнение перепишется так: Дифференциальные уравнения В этом уравнении постоянных только два С1 и С. Таким образом, в первоначальном уравнении постоянные C1, С2 и С3 не были независимыми. Также, если рассмотрим уравнение

Дифференциальные уравнения

то его можно переписать в виде

Дифференциальные уравнения

обозначив С1 — С2 через С4, а С2 + С3 через C5, получим

Дифференциальные уравнения

Следовательно, в уравнении

Дифференциальные уравнения

, постоянные C1, С2 и С3 не являются независимыми.

Пример:

Дифференциальные уравнения имеет общее решение

Дифференциальные уравнения

Решение, которое получается из общего при определенных значениях произвольных постоянных, называется частным решением. Например, если положить в предыдущем примере С1 = 0 и С2 = 1, то получим y = sin x, это частное решение.

Геометрический смысл общего решения: общее решение дифференциального уравнения является семейством кривых, зависящим от произвольных постоянных в числе, равном порядку дифференциального уравнения. Частное решение имеет своим графиком какую-нибудь из кривых, входящих в указанное семейство. Эти кривые называются интегральными кривыми.

Часто встречается задача, в которой нужно определить частное решение по начальным данным, или начальным условиям. Эта задача состоит в следующем:

Дано дифференциальное уравнение у’ = f(х, у); требуется найти решение y = Дифференциальные уравнения(x) , которое при х = а принимало бы значение Дифференциальные уравнения (а) = b, или, что то же, найти решение, график которого проходил бы через заданную точку (а, b). Координаты этой точки и являются начальными данными, или начальными условиями. Например, дано уравнение Дифференциальные уравнения; его общим решением является у = Сх (так как у’ = С, то при подстановке в уравнение получаем тождество). Надо выделить из общего решения частное, принимающее значение y = 6 при х = 2 (это начальные данные). Полагая в уравнении у = Сх х = 2, а у = 6, получаем 6 = С2, из этого уравнения определяем С. Находим, что С= 3. Таким образом, у = 3х является искомым частным решением.

Дифференциальные уравнения возникают часто при решении геометрических и физических задач, так как установить соотношение между дифференциалами (или производными) обычно легче, чем между самими величинами. Это объясняется тем, что, пользуясь дифференциалами, можно допускать ошибки, являющиеся бесконечно малыми высшего порядка по сравнению с приращением независимого переменного, как мы это видели в гл. IX и XII и увидим в пр. 4 следующего параграфа.

Дифференциальные уравнения первого порядка

Из всех дифференциальных уравнений первого порядка мы рассмотрим только один тип, именно уравнения с разделяющимися переменными. Дифференциальным уравнением с разделяющимися переменными называется уравнение вида

Дифференциальные уравнения

Чтобы проинтегрировать, или, что то же, решить это уравнение, переписывают его в таком виде:

Дифференциальные уравнения

После этого ищут неопределенные интегралы от левой и правой частей уравнения (2) и приравнивают результаты. Произвольные постоянные, которые при этом получаются, обычно переносят в одну из частей равенства и их разность обозначают одной буквой. Покажем на примерах, как это делается.

Пример:

Рассмотрим уравнение Дифференциальные уравнения. Разделяя переменные, перепишем его так: у dу = х dх. Интегрируя правую и левую части уравнения, получаем

Дифференциальные уравнения

Обозначая Дифференциальные уравнения , найдем Дифференциальные уравнения откуда

Дифференциальные уравнения

Это и есть общее решение данного уравнения.

Пример:

Рассмотрим дифференциальное уравнениеДифференциальные уравнения Заменим производную отношением

дифференциалов Дифференциальные уравненияи разделим переменные:

Дифференциальные уравнения

Проинтегрируем левую часть по у, правую по x и приравняем результаты:

Дифференциальные уравнения

Это равенство и дает общее решение.

Интересно отметить, что общие решения могут иметь различный вид, поэтому сразу не скажешь, что они дают одну и ту же связь между х и у. Например, возьмем только что полученное общее решение и положим С = aгctgC1, тогда

Дифференциальные уравнения

Теперь возьмем тангенсы от левой и правой частей

Дифференциальные уравнения

Заметив, что tg arctg y = у, и применив формулу тангенса суммы, получим

Дифференциальные уравнения

или

Дифференциальные уравнения

Последнее равенство также дает общее решение уравнения (*).

Пример:

Закон охлаждения тела в воздухе формулируется следующим образом:

Скорость охлаждения тела в воздухе пропорциональна разности температур тела и воздуха, т. е.

Дифференциальные уравнения

где t — время, и — температура тела, и0 — температура воздуха, которая во время опыта считается неизменной, Дифференциальные уравнения— коэффициент пропорциональности.

Тело в начале опыта имело температуру 100° С. Через 20 минут его температура стала 60° С. Узнать, через сколько минут температура тела станет равной 30° С, если известно, что температура воздуха во время опыта оставалась равной 20° С.

В нашей задаче и0 = 20, поэтому сформулированный закон запишется

Дифференциальные уравнения

Это—дифференциальное уравнение первого порядка с разделяющимися переменными. Перепишем его, разделив переменные:

Дифференциальные уравнения

Интегрируя левую и правую части, будем иметь

Дифференциальные уравнения

или, потенцируя,

Дифференциальные уравнения

Используем начальные условия: подставим в равенство (5) t = 0 и и = 100, получим Дифференциальные уравнения. Отсюда Дифференциальные уравнения= 80, и уравнение (5) принимает вид

Дифференциальные уравнения

Полученное уравнение еще не позволяет вычислить искомое время, так как в нем содержится неизвестный коэффициент Дифференциальные уравнения. Для его определения используем результаты наблюдения, т. е. что при t = 20 температура и = 60. Подставляя эти данные в уравнение (6), получим

Дифференциальные уравнения

откуда Дифференциальные уравнения, следовательно, уравнение (6) можно записать в виде

Дифференциальные уравнения

Теперь можно получить ответ на поставленный в задаче вопрос: положим в (7) и = 30, тогда

Дифференциальные уравнения

Решая это уравнение, находим

Дифференциальные уравнения
Дифференциальные уравнения

Таким образом, тело за 60 минут охладится до 30° С.

Пример:

Истечение жидкости из сосуда. Если жидкость вытекает из сосуда через малое отверстие в дне сосуда, то скорость v истечения в данный момент определяется формулой Дифференциальные уравнения Здесь h —расстояние свободной поверхности жидкости от отверстия,Дифференциальные уравнения— ускорение силы тяжести и Дифференциальные уравнения— коэффициент, определяемый опытным путем и зависящий от вязкости жидкости и формы отверстия. Например, если жидкость— вода, а отверстие—круг радиуса 0,1, то коэффициент Дифференциальные уравнения = 0,6. Значит, в этом случае формула будет выглядеть так:

Дифференциальные уравнения

Пользуясь приведенной формулой, решим задачу.

Задача:

Вычислить время T, за которое вода вытечет из сосуда, имеющего форму полушара радиуса R = 1м если известно, что в начальный момент вода заполняла весь сосуд. Отверстие, через которое вытекает вода, является кругом радиуса 0,1 м (рис. 110, а).

Дифференциальные уравнения

Решение:

Выберем оси координат так, как показано на рис. 110, я, и рассмотрим промежуточный момент времени t(0 < t < Т). В этот момент вода уже не будет занимать всего сосуда и ее поверхность будет находиться на расстоянии х от начала координат. Расстояние свободной поверхности воды от отверстия будет равно h = 1— х, скорость истечения

Дифференциальные уравнения

Рассмотрим, что произойдет с водой за малый промежуток времени dt, протекший с момента t . За этот промежуток времени уровень воды понизится на . С другой стороны, за этот же промежуток времени вода вытечет через отверстие в виде цилиндрической струйки. На чертеже изображены вытекший слой высоты (рис. 110, в) и струйка (рис. 110, в). Объем слоя и струйки равны между собой.

Подсчитаем каждый из этих объемов. Подсчеты будем вести так же, как в гл. XII, с точностью до бесконечно малых высшего порядка. Так как dx мало, то можно считать, что скорость истечения за промежуток времени dt остается неизменной, такой же, какой она была в момент времени t. Значит, будем считать, что скорость истечения в указанном промежутке времени равна

Дифференциальные уравнения

Поэтому длина струйки (высота цилиндра) равна скорости, умноженной на время, т. е.

Дифференциальные уравнения

Объем струйки получим, умножив ее длину (высоту цилиндра) на площадь отверстия (площадь основания цилиндра), которая равна

Дифференциальные уравнения

Таким образом, объем струйки равен

Дифференциальные уравнения

Теперь подсчитаем объем слоя как объем цилиндра (см. гл. XII, § 1), высота которого равна , а радиус основания МК Выразим МК через х из прямоугольного треугольника ОМК. Применяя теорему Пифагора, получим

Дифференциальные уравнения

Поэтому площадь круга равна

Дифференциальные уравнения

а объем слоя

Дифференциальные уравнения

Приравнивая объем струйки объему слоя, получим

Дифференциальные уравнения

Это — дифференциальное уравнение, связывающее расстояние свободной поверхности воды от начала координат, время истечения и их дифференциалы. Уравнение (**) можно переписать в виде

Дифференциальные уравнения

Здесь переменные разделены, т. е. левая часть зависит только от t а правая — только от х. Интегрируя, получим

Дифференциальные уравнения

Для вычисления интеграла, стоящего в правой части, сделаем замену переменного, положив 1—х = z. Отсюда x = 1— z, dх = — dz, и интеграл перепишется в следующем виде:

Дифференциальные уравнения

Делая последовательные преобразования и интегрируя, получаем

Дифференциальные уравнения

Следовательно,

Дифференциальные уравнения

Это уравнение, хотя и дает связь между величинами t й х, но для расчетов им воспользоваться нельзя, так как в него еще входит произвольное постоянное С, величина которого неизвестна.

Однако вспомним, что нами не было использовано условие, что в начальный момент времени t = 0 вода занимала весь сосуд, т. е. ее поверхность проходила через начало координат. Значит, при t = 0 величина х равнялась нулю. Подставляя t = 0 и x = 0 в уравнение (***), получим

Дифференциальные уравнения

откуда

Дифференциальные уравнения

Найденное значение С подставляем в (***):

Дифференциальные уравнения

Итак, мы получили связь между х и t. Так как в момент, когда вся вода вытечет, x = 1, то время истечения воды из сосуда найдем, подставляя x = 1 в уравнение (****); получим, что T = 35,2сек.

Некоторые дифференциальные уравнения, встречающиеся в механик

Если точка движется по оси Ох под влиянием силы F и если t обозначает время, а х— расстояние от начала координат, то скорость v точки определяется первой производной Дифференциальные уравненияа ускорение а —второй производной, т. е. Дифференциальные уравнения. Связь между силой, массой точки и ее ускорением устанавливается законом Ньютона: действующая сила равна произведению массы на ускорение, т. е.

Дифференциальные уравнения

или

Дифференциальные уравнения

или

Дифференциальные уравнения

Приведем примеры дифференциальных уравнений, полученных при помощи этого закона.

Пример:

Точка движется под действием силы, пропорциональной скорости точки и направленной против движения. В начальный момент точка находилась в начале координат и скорость точки была равна v0. Найти закон движения, т. е. связь между расстоянием точки от начала координат и временем.

В этом примере величина силы F равна Дифференциальные уравненияv, а так как сила направлена против движения, то

Дифференциальные уравнения

Применяя закон в форме (2), получаем дифференциальное уравнение первого порядка

Дифференциальные уравнения

Разделяем переменные:

Дифференциальные уравнения

Интегрируем левую и правую части и приравниваем результаты:

Дифференциальные уравнения

Для удобства записи положим С1 = lnС:

Дифференциальные уравнения

потенцируя, имеем

Дифференциальные уравнения

Это — общее решение задачи.

Так как в начальный момент t = 0 скорость равнялась v, то, подставляя t = 0 и v = v0 в уравнение (4), найдем значение С:

Дифференциальные уравнения

следовательно, уравнение (4) примет вид

Дифференциальные уравнения

Это — частное решение. Однако задача еще не решена, так как зависимость х от t не найдена. В силу того, что Дифференциальные уравнения уравнение (5) можно переписать в виде

Дифференциальные уравнения

Это тоже дифференциальное уравнение первого порядка. Разделяем переменные:

Дифференциальные уравнения

Интегрируем отдельно левую и правую части и, приравнивая результаты, получаем

Дифференциальные уравнения

Так как в начальный момент времени точка находилась в начале координат, то, подставляя в (8) значения t = 0 и x = 0, имеем

Дифференциальные уравнения

Отсюда

Дифференциальные уравнения

При этом значении С2 из общего решения (8) получаем частное

Дифференциальные уравнения

или

Дифференциальные уравнения

Это и есть решение задачи.

Приведем конкретный пример, сводящийся к предыдущей задаче.

Пример:

Моторная лодка, вес которой 245 кг, идет прямолинейно и равномерно со скоростью 10 м\сек. В некоторый момент, который будем считать начальным, двигатель выключается и движение лодки замедляется за счет трения о воду. Через одну секунду после выключения двигателя лодка имела скорость 8 м\сек. Нужно найти скорость лодки через 5 сек и расстояние, пройденное за это время.

Примечание:

Сила трения лодки о воду пропорциональна скорости и направлена против движения. Коэффициент пропорциональности находится из опыта.

Применяя обозначения предыдущей задачи, имеем v0 = 10 м|сек. Так как вес лодки 245 кг, то ее масса равна Дифференциальные уравнения, а считая, что Дифференциальные уравнения, получим 25. Подставляя эти данные в равенство (5), находим

Дифференциальные уравнения

Эта формула еще непригодна для вычислений, так как в ней к неизвестно. Но мы еще не использовали то, что через 1 сек скорость лодки была 8 м\сек. Подставив эти данные, полученные из наблюдений, в равенство (5′), мы сможем найти Дифференциальные уравнения. Сделаем это:

Дифференциальные уравнения

откуда

Дифференциальные уравнения

Логарифмируя обе части этого выражения, найдем

Дифференциальные уравнения

откуда

Дифференциальные уравнения

Итак, коэффициент пропорциональности Дифференциальные уравнения найден. После этого равенство (5′) примет вид

Дифференциальные уравнения

Отсюда можно определить искомую скорость, подставляя t = 5 сек:

Дифференциальные уравнения

Подставляя найденное значение Дифференциальные уравнения и данные значения т, v0 и t = 5 в равенство (9), найдем путь, пройденный за пять секунд:

Дифференциальные уравнения

Пример:

Рассмотрим движение точки под влиянием силы

Дифференциальные уравнения

Возьмем закон Ньютона в форме (3), тогда

Дифференциальные уравнения

Это дифференциальное уравнение второго порядка. Проверим что выражение

Дифференциальные уравнения

является его общим решением. Для этого найдем

Дифференциальные уравнения

и

Дифференциальные уравнения

Подставляя вторую производную и саму функцию в уравнение, получим

Дифференциальные уравнения

Таким образом, функция (11), подставленная в уравнение (10), обращает его в тождество. Значит, функция (11) является решением уравнения (10). Поскольку эта функция содержит две произвольные постоянные С1 и С2 (ведь это любые числа), то она является общим решением уравнения (10).

Общее решение можно записать и в другом виде, а именно:

Дифференциальные уравнения

Положим

Дифференциальные уравнения

тогда

Дифференциальные уравнения

или

Дифференциальные уравнения

Коэффициент А называется амплитудой,Дифференциальные уравнения — частой, Дифференциальные уравнения— фазой. Вспомнив гл. IV, § 1, видим, что любым решением уравнения (10) является синусоида, т. е. колебательное движение. Уравнение (10) называется уравнением гармонических колебаний.

Движение точки на плоскости

Система дифференциальных уравнений:

Пусть точка Р с массой т движется на плоскости под влиянием силы F, которая меняется и по величине и по направлению. Для изучения этого движения точку Р проектируют на оси координат и рассматривают движения эти^ проекций по осям. Таким образом, вопрос движения точки на плоскости сводится к рассмотрению двух движений по осям координат.

Обозначим угол, образуемый силой F с положительным направлением оси Ох, через а (этот угол является переменной величиной). Тогда проекция силы F на ось Ох (обозначим ее X) будет

Дифференциальные уравнения

а проекция силы F на ось Оу (обозначим ее Y) будет

Дифференциальные уравнения

Проекции скорости v на оси Ох и Оу обозначим соответственно Дифференциальные уравненияи Дифференциальные уравненияпроекции точки Р на оси Ох и ОуРх и Рy. Имеем Рх(х, 0), Ру(0).

В соответствии с § 4 можно записать:

Дифференциальные уравнения

или

Дифференциальные уравнения

Таким образом, движение точки на плоскости определяется двумя дифференциальными уравнениями Дифференциальные уравненияи Дифференциальные уравнения. Иногда удобнее пользоваться уравнениями Дифференциальные уравнения и Дифференциальные уравнения. Уравнения Дифференциальные уравнения и Дифференциальные уравнения называются системой дифференциальных уравнений движения точки на плоскости

Пример:

Рассмотрим движение материальной точки Р под влиянием силы тяжести F в безвоздушном пространстве, если известно, что точка брошена под углом а к горизонту из начала координат с начальной скоростью Дифференциальные уравнения.

Выберем оси координат так, чтобы ось Ох была горизонтальной (рис. 111).

Дифференциальные уравнения

Во время движения на точку Р действует только сила тяжести F, равная mg, где т— масса точки, a g—ускорение силы тяжести. Сила тяжести направлена в отрицательном направлении оси Оу. Никакие другие силы на точку не действуют.

Проекция силы F на ось Ох равна нулю, а на ось Оу — равна (—mg). Поэтому уравнения Дифференциальные уравнения и Дифференциальные уравнения для нашего случая напишутся так:

Дифференциальные уравнения

Это—система двух дифференциальных уравнений второго порядка. Однако удобнее воспользоваться уравнениями Дифференциальные уравнения и Дифференциальные уравнения, т. е. системой дифференциальных уравнений первого порядка. Эта система для рассматриваемой задачи такова:

Дифференциальные уравнения
Дифференциальные уравнения

Из уравнения (5Х) имеем: Дифференциальные уравнениязначит, Дифференциальные уравнениявсе время сохраняет постоянное значение, а так как в начальный момент времени

Дифференциальные уравнения

, то и всегда

Дифференциальные уравнения

Но Дифференциальные уравнения поэтому уравнение (6) перепишем в виде dx

Дифференциальные уравнения

Интегрируя, находим

Дифференциальные уравнения

Известно, что при t = 0 абсцисса равна нулю, т. е. х = 0. Подставляя эти значения в равенство (8), находим

Дифференциальные уравнения

Уравнение (8) получает вид

Дифференциальные уравнения

Из уравнения (5t) находим:

Дифференциальные уравнения

или

Дифференциальные уравнения

Интегрируя, получаем, что

Дифференциальные уравнения

Так как при t = 0 Дифференциальные уравнения то из (10)

Дифференциальные уравнения

Уравнение (10) принимает вид

Дифференциальные уравнения

или

Дифференциальные уравнения

Интегрируя, наконец, последнее уравнение, получим, что

Дифференциальные уравнения

Так как при t = 0 ордината тоже равна нулю, то, подставляя значения t = 0 и y = 0 в (И), находим, что C3 = 0. После этого уравнение (11) уже принимает окончательный вид:

Дифференциальные уравнения

Уравнения (9) и (12) определяют движение точки, брошенной под углом к горизонту, под действием силы тяжести. Эти уравнения были уже получены из других соображений в гл. III.

Дифференциальные уравнения первого порядка

Дифференциальное уравнение — это уравнение, содержащее
неизвестные функции, независимые переменные и производные неизвестных функций.

Уравнение вида

F(x,y,y’) = 0, (10.1)

где х — независимая переменная, у — искомая функция, у’ — ее
производная, называется дифференциальным уравнением первого порядка.

Порядок дифференциального уравнения определяется наивысшим
порядком производной неизвестной функции, входящей в уравнение.

Если уравнение (10.1) можно разрешить относительно у’ то
оно принимает вид

у’= f(х,у).

Решением дифференциального уравнения первого порядка называется
функция Дифференциальные уравнения в промежутке (a,b) (как конечном, так и
бесконечном), которая при подстановке в уравнение обращает его в
тождество. Вместо термина «решение» часто используют выражение
«интеграл дифференциального уравнения». Это связано с тем, что, во-
первых, сама задача является обобщением задачи интегрирования
функций и, во-вторых, решение действительно нередко находят путем интегрирования функций или, как принято говорить, путем квадратур.

Решением дифференциального уравнения Дифференциальные уравнения является
интеграл Дифференциальные уравнения Таким образом, это дифференциальное
уравнение имеет бесконечное множество решений, которое
выражается следующим образом.

Существует общее решение у(х, С) дифференциального
уравнения, которое зависит не только от независимой переменной х,
но еще и от произвольного параметра С, называемого постоянной
интегрирования.
Подставляя вместо С конкретные частные
значения, получают различные частные решения.

Условия

Дифференциальные уравнения при Дифференциальные уравнения

в силу которых функция Дифференциальные уравнения принимает заданное значение Дифференциальные уравнения
в заданной точке Дифференциальные уравнения называют начальными условиями решения.

Теорема существования (Коши)

Если f(х, у) непрерывна в
некоторой области около точки Дифференциальные уравнения то существует по крайней
мере одно решение уравнения

у’ = f(х,у), (10.3)

принимающее при Дифференциальные уравнения значение Дифференциальные уравнения определенное и
непрерывное в некотором интервале около Дифференциальные уравнения Если, кроме того, в этой области выполняется условие Липшица, т.е.

Дифференциальные уравнения

причем N не зависит от х, Дифференциальные уравнения и Дифференциальные уравнения то это решение единственное и
является непрерывной функцией от Дифференциальные уравнения

Геометрический смысл теоремы существования и
единственности состоит в том, что через каждую точку Дифференциальные уравнения проходит одна и только одна интегральная кривая уравнения (10.3).

Дифференциальные уравнения с разделяющимися переменными

Если дифференциальное уравнение путем умножения или деления на какое-либо выражение можно привести к виду

Дифференциальные уравнения

где известная функция А зависит только от х, а В — только от у, то оно
называется дифференциальным уравнением с разделяющимися
переменными.

Последнее уравнение можно записать и в иной форме:

Дифференциальные уравнения или Дифференциальные уравнения

где каждый символ интеграла означает какую-либо первообразную
для своей подынтегральной функции, а С — постоянную
интегрирования.

Пример:

Найти общее решение дифференциального уравнения Дифференциальные уравнения

Решение:

Разделение переменных дает Дифференциальные уравнения

Интегрируя, получим

Дифференциальные уравнения

Последнее выражение эквивалентно уравнению Дифференциальные уравненияПолагая
Дифференциальные уравнения окончательно получим: Дифференциальные уравнения

Пример:

Найти частное решение дифференциального уравнения

Дифференциальные уравнения

по начальным условиям Дифференциальные уравнения при Дифференциальные уравнения

Решение:

Общее решение исследуемого уравнения имеет вид (см.
пример 10.1) Дифференциальные уравнения Подставив в него начальные условия, найдем Дифференциальные уравнения откуда С = 1. Таким образом, искомое частное решение: Дифференциальные уравнения

Пример:

Используя метод разделения переменных, найти
общие решения дифференциальных уравнений:

а) Дифференциальные уравненияб)Дифференциальные уравненияв) Дифференциальные уравнения

Решение:

а) Дифференциальные уравнения

Решим интегралы, введя замену 1 + x=t. Тогда

Дифференциальные уравнения

Таким образом, интегрируя дифференциальное уравнение, найдем

Дифференциальные уравнения

Введем замену Дифференциальные уравнения Окончательно получим:

Дифференциальные уравнения

б)Дифференциальные уравнения

Введя замену Дифференциальные уравнения найдем:

Дифференциальные уравнения

Введя замену Дифференциальные уравнения вычислим:

Дифференциальные уравнения

Таким образом, Дифференциальные уравнения или Дифференциальные уравнения

в) Дифференциальные уравнения

Левый интеграл уравнения был найден в предыдущей задаче, а
правый определяется подстановкой Дифференциальные уравнения т.е.

Дифференциальные уравнения

Представив подынтегральное выражение в виде

Дифференциальные уравнения

найдем

Дифференциальные уравнения

Таким образом, Дифференциальные уравнения или Дифференциальные уравнения

Пример:

Используя метод разделения переменных, найти
частные решения дифференциальных уравнений, удовлетворяющие
указанным начальным условиям:

а) Дифференциальные уравнения Дифференциальные уравнения при Дифференциальные уравнения

б) Дифференциальные уравнения Дифференциальные уравненияпри Дифференциальные уравнения

в) Дифференциальные уравненияДифференциальные уравнения при Дифференциальные уравнения

г) Дифференциальные уравненияДифференциальные уравнения при Дифференциальные уравнения

Решение:

а) Дифференциальные уравнения

Для расчета значения С подставим в полученное выражение начальные условия: Дифференциальные уравнения Отсюда С =-2 . Частное решение имеет вид Дифференциальные уравнения или Дифференциальные уравнения

б) Дифференциальные уравнения

в) Дифференциальные уравнения

г)Дифференциальные уравнения

Пример:

Функция спроса q от цены р имеет вид Дифференциальные уравнения где Дифференциальные уравнения

Тогда функция выручки от продаж R от цены (этот результат будет получен позже) определяется дифференциальным уравнением

Дифференциальные уравнения

Найти функцию выручки от цены.

Решение:

Дифференциальное уравнение иначе можно записать
в виде Дифференциальные уравнения
Проинтегрировав данное уравнение, получим:

Дифференциальные уравнения

где Дифференциальные уравнения — постоянная интегрирования.

Перепишем полученное уравнение в виде

Дифференциальные уравнения

Определим

Дифференциальные уравнения

Отсюда следует

Дифференциальные уравнения

Таким образом, функция выручки от цены является параболой.
Точки пересечения с осью Ор находим из уравнения

Дифференциальные уравнения

Решения этого уравнения р = 0 и Дифференциальные уравнения являются искомыми
точками пересечения. Симметрия параболы позволяет определить
координаты экстремальной точки. По оси абсцисс координата экстремальной точки Дифференциальные уравнения а по оси координат — Дифференциальные уравнения

Значение выручки в экстремальной точке

Дифференциальные уравнения

С другой стороны, по определению выручки

Дифференциальные уравнения

Приравнивая правые части двух последних выражений, найдем

Дифференциальные уравнения

Подставив полученное значение в формулу для выручки, получим

Дифференциальные уравнения

График функциональной зависимости выручки от цены товара показан на рис. 10.1.

Дифференциальные уравнения

Из рисунка следует, что для цены на товар, заключенной между 0 и Дифференциальные уравнения, повышение цены приведет к повышению выручки от продаж продавца. Понижение цены приведет к снижению выручки. Для
интервала цен от Дифференциальные уравнения до Дифференциальные уравненияповышение цены приведет к снижению выручки, понижение цены — к ее повышению. В частности, из графика рис. 10.1 следует, что при цене на товар, равной Дифференциальные уравнения, незначительное снижение и повышение цены
приводят к небольшому снижению выручки от продаж продавца. ►

Линейные дифференциальные уравнения

Линейным дифференциальным уравнением первого порядка
называется уравнение вида

y’+p(x)y = f(x). (10.4)

Термином «линейный» здесь отмечается, что уравнение линейно,
т.е. первой степени, относительно совокупности символов у и у’.

Линейное уравнение (10.4) называется однородным, если f(x) = 0.
Если Дифференциальные уравнения то уравнение (10.4) называется неоднородным.

Найдем сначала решение однородного уравнения. Тогда,
предполагая, что Дифференциальные уравнения получим:

Дифференциальные уравнения или Дифференциальные уравнения

Интегрируя, получим:

Дифференциальные уравнения

где Дифференциальные уравнения — какая-либо из первообразных функций для р(х).

Потенцирование дает

Дифференциальные уравнения

Пусть теперь Дифференциальные уравнения Введем новую неизвестную функцию
u(х) с помощью формулы

Дифференциальные уравнения

Это преобразование получено заменой в решении (10.5)
однородного уравнения постоянной С неизвестной функцией u(х).
Такое преобразование называется изменением, или вариацией
постоянной.
Продифференцируем последнее выражение:

Дифференциальные уравнения

Так как Дифференциальные уравнения то эту формулу можно переписать в виде

Дифференциальные уравнения

Подставляя у’, выраженное из последней формулы, и (10.6) в
(10.4), получим уравнение для искомой функции u(х):

Дифференциальные уравнения или Дифференциальные уравнения

Интегрируя, найдем

Дифференциальные уравнения

Здесь символ интеграла обозначает только одну какую-либо из
первообразных функций для своей подынтегральной функции.
Выражение для u(х) построено из известных функций р(х) и f(x)
исключительно с помощью квадратур, т.е. процессов
обыкновенного интегрирования. Для первоначальной неизвестной у получается следующее выражение:

Дифференциальные уравнения

Пример:

Найти общее решение дифференциального уравнения

у’ + ху = -х. (10.7)

Решение:

Решим сначала однородное уравнение у’ + ху = 0 .
Разделяя переменные Дифференциальные уравнения и интегрируя, находим

Дифференциальные уравнения

Введем новую неизвестную функцию u(х) с помощью формулы

Дифференциальные уравнения

Дифференцируя, получим

Дифференциальные уравненияили Дифференциальные уравнения

Учитывая (10.7) и (10.8), находим

Дифференциальные уравнения

Разделяя переменные Дифференциальные уравнения и интегрируя, получим

Дифференциальные уравнения

Окончательный результат определяем, подставив полученное
выражение в (10.8):

Дифференциальные уравнения

Правильность результата легко проверить дифференцированием. ►

Пример:

Найти общие решения линейных уравнений:

а) Дифференциальные уравненияб) Дифференциальные уравнения

Решение:

а) Дифференциальные уравнения Однородное уравнение имеет вид

Дифференциальные уравнения или Дифференциальные уравнения

Его решение Дифференциальные уравнения или Дифференциальные уравнения

Общее решение неоднородного уравнения будем искать в виде
Дифференциальные уравнения Дифференцируя, имеем Дифференциальные уравненияили
Дифференциальные уравнения откуда Дифференциальные уравнения или Дифференциальные уравнения

Интегрируя, получим Дифференциальные уравненияОбщее решение исследуемого
уравнения имеет вид

Дифференциальные уравнения

б) Дифференциальные уравнения Однородное уравнение имеет вид

Дифференциальные уравнения

Ищем решение в виде

Дифференциальные уравнения

Уравнение Бернулли

Рассмотрим несколько более общее дифференциальное уравнение, называемое уравнением Бернулли:

Дифференциальные уравнения

где n — любое постоянное число.

Линейное уравнение является частным случаем уравнения Бернулли при n = 0. При n = 1 получаем уравнение с разделяющимися
переменными:

Дифференциальные уравнения или Дифференциальные уравнения

Будем полагать, что в уравнении Бернулли п отлично от нуля и
единицы. Решить это уравнение можно так же, как и линейное,
методом вариации постоянной. Найдем сначала решение
однородного уравнения. Тогда, предполагая, что Дифференциальные уравнения получим Дифференциальные уравненияили Дифференциальные уравнения

Интегрируя, получим

Дифференциальные уравнения

где Дифференциальные уравнения — какая-либо из первообразных функций для р(х).

Потенцирование дает

Дифференциальные уравненияи Дифференциальные уравнения

При определении общего решения уравнения Бернулли введем
новую неизвестную функцию v(x) с помощью формулы

Дифференциальные уравнения

Это преобразование получено заменой в решении (10.10)
однородного уравнения постоянной С неизвестной функцией v(x).
Продифференцируем выражение (10.11) и учтем, что Дифференциальные уравнения

Дифференциальные уравнения

Подставив последнюю формулу и (10.11) в (10.9), получим

Дифференциальные уравнения

Разделив переменные, найдем

Дифференциальные уравнения или Дифференциальные уравнения

Интегрируя, получим:

Дифференциальные уравнения

или

Дифференциальные уравнения

где под символом интеграла подразумевается какая-либо одна первообразная.

Для определения первоначальной неизвестной у возведем правую и левую части (10.11) в степень 1-n:

Дифференциальные уравнения

Подставив в полученное выражение (10.12), найдем окончательное решение уравнения Бернулли:

Дифференциальные уравнения

или

Дифференциальные уравнения

Метод вариации постоянной может быть применен во многих других случаях.

Пример:

Найти общее решение уравнения Бернулли

Дифференциальные уравнения

Решение:

Решим сначала однородное уравнение у’ — ху = 0 . Разделяя переменные Дифференциальные уравнения и интегрируя, находим

Дифференциальные уравнения

Введем новую неизвестную функцию v(x) с помощью формулы

Дифференциальные уравнения

Дифференцируя, получим:

Дифференциальные уравнения

Подставив последнюю формулу и (10.15) в (10.14), найдем

Дифференциальные уравнения

Интегрируя, получим

Дифференциальные уравнения

или

Дифференциальные уравнения

Окончательный результат определяем, подставив полученное
выражение в (10.15):

Дифференциальные уравнения

Пример:

Найти общие решения уравнений Бернулли:

а) Дифференциальные уравнения б)Дифференциальные уравнения

Решение:

а) Дифференциальные уравнения Решим сначала однородное
уравнение

Дифференциальные уравнения

Введем новую неизвестную функцию v(x) с помощью формулы

Дифференциальные уравнения

Дифференцируя, получим:

Дифференциальные уравнения

Подставив последнюю формулу и (10.15) в исходное уравнение,
найдем

Дифференциальные уравнения

Интегрируя, получим:

Дифференциальные уравнения

или

Дифференциальные уравнения

Окончательный результат определяем, подставив полученное
выражение в (10.16):

Дифференциальные уравнения

б) Дифференциальные уравнения Разделив правую и левую части на х, найдем:

Дифференциальные уравнения

Решим сначала однородное уравнение Дифференциальные уравнения

Дифференциальные уравнения

Введем неизвестную функцию v(x):

Дифференциальные уравнения

Дифференцируя, получим:

Дифференциальные уравнения

Подставив последнюю формулу и (10.18) в (10.17), найдем:

Дифференциальные уравнения

Интегрируя, получим:

Дифференциальные уравнения

Окончательный результат определяем, подставив данное выражение
в (10.18):

Дифференциальные уравнения

Дифференциальные уравнения второго порядка

Уравнение вида

F(x, у, у’, у») = 0,

где х — независимая переменная, у — искомая функция у’ и у» — ее
производные, называется дифференциальным уравнением второго порядка.

Обычно изучаются уравнения, разрешенные относительно
второй производной:

у»= f(x,y,y’).

Решением дифференциального уравнения второго порядка
называется функция у(х), которая при подстановке в уравнение обращает его в тождество. График решения называется интегральной кривой.

Условия

Дифференциальные уравнения при Дифференциальные уравнения

называются начальными условиями.

Пример:

Найти общие и частное решения уравнения у» = 2
при начальных условиях Дифференциальные уравнения при Дифференциальные уравнения

Решение:

Последовательно интегрируя, находим сначала первую производную:

Дифференциальные уравнения

а затем общее решение:

Дифференциальные уравнения

Подставляя значения начальных условий в выражение общего
решения и его производную, получим систему линейных уравнений:

Дифференциальные уравнения

откуда находим Дифференциальные уравненияи Дифференциальные уравнения

Таким образом, искомым частным решением является функция

Дифференциальные уравнения

Линейные дифференциальные уравнения с постоянными коэффициентами

Линейное дифференциальное уравнение с постоянными коэффициентами может быть записано в виде

Дифференциальные уравнения

где Дифференциальные уравнения постоянные коэффициенты.

Если в уравнении (10.19) правая часть равна нулю, то уравнение называется однородным, если не равна нулю — неоднородным.

Линейное однородное дифференциальное уравнение с постоянными коэффициентами имеет вид

Дифференциальные уравнения

Рассмотрим метод интегрирования однородного дифференциального уравнения с постоянными коэффициентами, предложенный Эйлером.

Частные решения линейного однородного дифференциального
уравнения с постоянными коэффициентами могут быть найдены в
виде Дифференциальные уравнения где k — константа. Подставив в (10.20) Дифференциальные уравнения и сокращая на не обращающийся в ноль множитель Дифференциальные уравнения получим:

Дифференциальные уравнения

Это уравнение n-й степени определяет те значения k, при которых Дифференциальные уравнения является решением исходного линейного однородного дифференциального уравнения с постоянными коэффициентами (10.20). Если все корни Дифференциальные уравнения характеристического уравнения (10.21) различны, то найдено n независимых решений Дифференциальные уравнения уравнения (10.20). Тогда общее решение исходного линейного однородного дифференциального уравнения с постоянными коэффициентами (10.20) имеет вид

Дифференциальные уравнения

где Дифференциальные уравнения — произвольные постоянные.

Пример:

Решить уравнение у»=3у’ + 2у = 0 .

Решение:

Характеристическое уравнение имеет вид Дифференциальные уравнения Его корни Дифференциальные уравнения

Следовательно, общее решение: Дифференциальные уравнения

Пример:

Решить уравнение у»‘ — у’ = 0.

Решение:

Характеристическое уравнение имеет вид Дифференциальные уравнения Его корни Дифференциальные уравнения

Общее решение — Дифференциальные уравнения

Комплексные корни характеристического уравнения (10.21) могут появляться лишь сопряженными парами, например:

Дифференциальные уравнения

Эти корни соответствуют комплексным решениям:

Дифференциальные уравнения

Комплексные решения могут быть заменены двумя действительными решениями, являющимися действительной и мнимой частями одного из представленных решений, т.е.

Дифференциальные уравнения

Пример:

Решить уравнение у» + 4у’ + 5у = 0.

Решение:

Характеристическое уравнение имеет вид Дифференциальные уравнения Его корни Дифференциальные уравнения

Следовательно, общее решение — Дифференциальные уравнения

Пример:

Решить уравнение Дифференциальные уравнения

Решение:

Характеристическое уравнение имеет вид

Дифференциальные уравнения

Его корни: Дифференциальные уравнения

Следовательно, общее решение — Дифференциальные уравнения

Если среди корней характеристического уравнения имеется корень Дифференциальные уравнения кратности Дифференциальные уравнения, то Дифференциальные уравнения— частных решений будут иметь вид

Дифференциальные уравнения

Пример:

Решить уравнение у'» — Зу» + 3у’ — у = 0 .

Решение:

Характеристическое уравнение имеет вид

Дифференциальные уравнения или Дифференциальные уравнения

Это уравнение имеет трехкратный корень Дифференциальные уравнения Частные решения: Дифференциальные уравнения

Следовательно, общее решение — Дифференциальные уравнения

Если среди корней характеристического уравнения имеется комплексный корень Дифференциальные уравнения кратности Дифференциальные уравнения, то Дифференциальные уравнения частных решений можно записать в виде

Дифференциальные уравнения

Отделяя действительную и мнимую части, получим Дифференциальные уравнения частных действительных решений:

Дифференциальные уравнения

Пример:

Решить уравнение Дифференциальные уравнения

Решение:

Характеристическое уравнение имеет вид Дифференциальные уравнения или Дифференциальные уравнения

Это уравнение имеет двукратные корни Дифференциальные уравнения и Дифференциальные уравнения Частные решения: Дифференциальные уравнения

Следовательно, общее решение —

Дифференциальные уравнения

Рассмотрим методы решения неоднородных линейных дифференциальных уравнений с постоянными коэффициентами. Если в уравнении (10.19) коэффициент Дифференциальные уравнениято его можно записать в виде:

Дифференциальные уравнения

Теорема:

Общее решение неоднородного линейного дифференциального уравнения (10.22) равно сумме общего решения Дифференциальные уравнения соответствующего однородного уравнения

Дифференциальные уравнения

и какого-либо частного решения Дифференциальные уравнения неоднородного уравнения:

Дифференциальные уравнения

При решении неоднородных линейных дифференциальных уравнений с постоянными коэффициентами во многих случаях удается подобрать их частные решения и тем самым свести задачу к интегрированию соответствующего однородного уравнения.

Пусть, например, правая часть является многочленом степени s. Тогда уравнение (10.22) принимает вид:

Дифференциальные уравнения

Если Дифференциальные уравнения то существует частное решение уравнения (10.24), тоже имеющее вид многочлена степени s:

Дифференциальные уравнения

Коэффициенты в уравнении (10.25) находят, подставляя (10.25) в (10.24) и приравнивая коэффициенты при одинаковых степенях х в левой и правой частях. Таким образом, получим для определения коэффициентов Дифференциальные уравнения где i = 0,1, 2,…, s, всегда разрешимую систему линейных уравнений:

Дифференциальные уравнения

Дифференциальные уравнения и т.д.

Предположим теперь, что Дифференциальные уравнения

Тогда уравнение (10.24) принимает вид:

Дифференциальные уравнения

Частное решение уравнения (10.26) —

Дифференциальные уравнения

Пример:

Решить уравнение Дифференциальные уравнения

Решение:

Характеристическое уравнение соответствующего
однородного уравнения имеет вид: Дифференциальные уравнения Данное уравнение имеет двукратные корни Дифференциальные уравнения и Дифференциальные уравнения Частные решения однородного уравнения: Дифференциальные уравнения а его общее решение —

Дифференциальные уравнения

Частное решение неоднородного уравнения имеет вид:

Дифференциальные уравнения

Найдем вторую производную Дифференциальные уравнения и подставим результат в исходное уравнение:

Дифференциальные уравнения

Отсюда находим Дифференциальные уравнения

Общее решение неоднородного уравнения —

Дифференциальные уравнения

Пример:

Решить уравнение у» + у’ = х — 2 .

Решение:

Характеристическое уравнение соответствующего однородного уравнения имеет вид: Дифференциальные уравнения Корни данного уравнения: Дифференциальные уравнения и Дифференциальные уравнения Частные решения однородного уравнения: Дифференциальные уравненияДифференциальные уравнения а его общее решение — Дифференциальные уравнения

Частное решение неоднородного уравнения имеет вид:

Дифференциальные уравнения

Найдем первую и вторую производные: Дифференциальные уравнения и подставим результат в исходное уравнение: Дифференциальные уравнения

Отсюда находим Дифференциальные уравнения

Общее решение неоднородного уравнения —

Дифференциальные уравнения

Рассмотрим теперь линейное неоднородное уравнение вида:

Дифференциальные уравнения

Если p не является корнем характеристического уравнения, то
частное решение надо искать в виде:

Дифференциальные уравнения

Если же р является корнем характеристического уравнения
кратности Дифференциальные уравнения (этот случай называют особым, или резонансным), то
частное решение имеет вид:

Дифференциальные уравнения

Пример:

Решить уравнение Дифференциальные уравнения

Решение:

Характеристическое уравнение соответствующего
однородного уравнения имеет вид: Дифференциальные уравнения Корни данного уравнения: Дифференциальные уравнения и Дифференциальные уравнения Частные решения однородного уравнения: Дифференциальные уравнения а его общее решение —

Дифференциальные уравнения

Частное решение неоднородного уравнения имеет вид

Дифференциальные уравнения

Найдем вторую производную:

Дифференциальные уравнения

Подставим результат в исходное уравнение:

Дифференциальные уравнения

Отсюда находим Дифференциальные уравнения

Общее решение неоднородного уравнения —

Дифференциальные уравнения

Пример:

Решить уравнение:Дифференциальные уравнения

Решение:

Характеристическое уравнение соответствующего
однородного уравнения имеет вид: Дифференциальные уравнения Это уравнение имеет
двукратные корни Дифференциальные уравнения и Дифференциальные уравненияЧастные решения однородного
уравнения: Дифференциальные уравнения а его общее решение —

Дифференциальные уравнения

Частное решение неоднородного уравнения необходимо искать в виде:

Дифференциальные уравнения

Найдем первую и вторую производные:

Дифференциальные уравнения

Подставим результат в исходное уравнение:

Дифференциальные уравнения

Отсюда находим

Дифференциальные уравнения

Общее решение неоднородного уравнения —

Дифференциальные уравнения

Решение заданий и задач по предметам:

Дополнительные лекции по высшей математике:

  1. Тождественные преобразования алгебраических выражений
  2. Функции и графики
  3. Преобразования графиков функций
  4. Квадратная функция и её графики
  5. Алгебраические неравенства
  6. Неравенства
  7. Неравенства с переменными
  8. Прогрессии в математике
  9. Арифметическая прогрессия
  10. Геометрическая прогрессия
  11. Показатели в математике
  12. Логарифмы в математике
  13. Исследование уравнений
  14. Уравнения высших степеней
  15. Уравнения высших степеней с одним неизвестным
  16. Комплексные числа
  17. Непрерывная дробь (цепная дробь)
  18. Алгебраические уравнения
  19. Неопределенные уравнения
  20. Соединения
  21. Бином Ньютона
  22. Число е
  23. Непрерывные дроби
  24. Функция
  25. Исследование функций
  26. Предел
  27. Интеграл
  28. Двойной интеграл
  29. Тройной интеграл
  30. Интегрирование
  31. Неопределённый интеграл
  32. Определенный интеграл
  33. Криволинейные интегралы
  34. Поверхностные интегралы
  35. Несобственные интегралы
  36. Кратные интегралы
  37. Интегралы, зависящие от параметра
  38. Квадратный трехчлен
  39. Производная
  40. Применение производной к исследованию функций
  41. Приложения производной
  42. Дифференциал функции
  43. Дифференцирование в математике
  44. Формулы и правила дифференцирования
  45. Дифференциальное исчисление
  46. Дифференциальные уравнения первого порядка
  47. Дифференциальные уравнения высших порядков
  48. Дифференциальные уравнения в частных производных
  49. Тригонометрические функции
  50. Тригонометрические уравнения и неравенства
  51. Показательная функция
  52. Показательные уравнения
  53. Обобщенная степень
  54. Взаимно обратные функции
  55. Логарифмическая функция
  56. Уравнения и неравенства
  57. Положительные и отрицательные числа
  58. Алгебраические выражения
  59. Иррациональные алгебраические выражения
  60. Преобразование алгебраических выражений
  61. Преобразование дробных алгебраических выражений
  62. Разложение многочленов на множители
  63. Многочлены от одного переменного
  64. Алгебраические дроби
  65. Пропорции
  66. Уравнения
  67. Системы уравнений
  68. Системы уравнений высших степеней
  69. Системы алгебраических уравнений
  70. Системы линейных уравнений
  71. Системы дифференциальных уравнений
  72. Арифметический квадратный корень
  73. Квадратные и кубические корни
  74. Извлечение квадратного корня
  75. Рациональные числа
  76. Иррациональные числа
  77. Арифметический корень
  78. Квадратные уравнения
  79. Иррациональные уравнения
  80. Последовательность
  81. Ряды сходящиеся и расходящиеся
  82. Тригонометрические функции произвольного угла
  83. Тригонометрические формулы
  84. Обратные тригонометрические функции
  85. Теорема Безу
  86. Математическая индукция
  87. Показатель степени
  88. Показательные функции и логарифмы
  89. Множество
  90. Множество действительных чисел
  91. Числовые множества
  92. Преобразование рациональных выражений
  93. Преобразование иррациональных выражений
  94. Геометрия
  95. Действительные числа
  96. Степени и корни
  97. Степень с рациональным показателем
  98. Тригонометрические функции угла
  99. Тригонометрические функции числового аргумента
  100. Тригонометрические выражения и их преобразования
  101. Преобразование тригонометрических выражений
  102. Комбинаторика
  103. Вычислительная математика
  104. Прямая линия на плоскости и ее уравнения
  105. Прямая и плоскость
  106. Линии и уравнения
  107. Прямая линия
  108. Уравнения прямой и плоскости в пространстве
  109. Кривые второго порядка
  110. Кривые и поверхности второго порядка
  111. Числовые ряды
  112. Степенные ряды
  113. Ряды Фурье
  114. Преобразование Фурье
  115. Функциональные ряды
  116. Функции многих переменных
  117. Метод координат
  118. Гармонический анализ
  119. Вещественные числа
  120. Предел последовательности
  121. Аналитическая геометрия
  122. Аналитическая геометрия на плоскости
  123. Аналитическая геометрия в пространстве
  124. Функции одной переменной
  125. Высшая алгебра
  126. Векторная алгебра
  127. Векторный анализ
  128. Векторы
  129. Скалярное произведение векторов
  130. Векторное произведение векторов
  131. Смешанное произведение векторов
  132. Операции над векторами
  133. Непрерывность функций
  134. Предел и непрерывность функций нескольких переменных
  135. Предел и непрерывность функции одной переменной
  136. Производные и дифференциалы функции одной переменной
  137. Частные производные и дифференцируемость функций нескольких переменных
  138. Дифференциальное исчисление функции одной переменной
  139. Матрицы
  140. Линейные и евклидовы пространства
  141. Линейные отображения
  142. Дифференциальные теоремы о среднем
  143. Теория устойчивости дифференциальных уравнений
  144. Функции комплексного переменного
  145. Преобразование Лапласа
  146. Теории поля
  147. Операционное исчисление
  148. Системы координат
  149. Рациональная функция
  150. Интегральное исчисление
  151. Интегральное исчисление функций одной переменной
  152. Дифференциальное исчисление функций нескольких переменных
  153. Отношение в математике
  154. Математическая логика
  155. Графы в математике
  156. Линейные пространства
  157. Первообразная и неопределенный интеграл
  158. Линейная функция
  159. Выпуклые множества точек
  160. Система координат